Download as pdf or txt
Download as pdf or txt
You are on page 1of 126

UNIVERSITY OF MINDANAO

College of Engineering
Program: Computer Engineering

Physically Distanced but Academically Engaged

Self-Instructional Manual (SIM) for Self-Directed Learning (SDL)

Engr. Stephen Paul Alagao


Engr. Maryjean A. Bantan
Engr. Luomar Jake Cabatas
Engr. Rey Albert Cabotaje
Engr. Jessie Calo
Engr. Krisca Lynge C. Donayre
Engr. Kimberly M. Nepa
Engr. Reynaldo Sahagun Jr.
Engr. Consuelo Tan

CEE 103 – Engineering Calculus 2

THIS SIM/SDL MANUAL IS A DRAFT VERSION ONLY; NOT FOR


REPRODUCTION AND DISTRIBUTION OUTSIDE OF ITS INTENDED USE.
THIS IS INTENDED ONLY FOR THE USE OF THE STUDENTS WHO ARE
OFFICIALLY ENROLLED IN THE COURSE/SUBJECT.
EXPECT REVISIONS OF THE MANUAL.

1
TABLE OF CONTENTS

PAGE
Cover Page ……………………………………………………………………………………………….. 1
Table of Contents………………………………………………………………………………………. 2
Course Outline…………………………………………………………………………………………... 3
Course Outline Policy………………………………………………………………………………… 3
Course Information…………………………………………………………………………………… 6

Topic/ Activity
Unit Learning Outcomes- Unit 1…………………………………………………………………. 10
Big Picture in Focus: ULO-1a…………………………………………………………………..….. 10
Metalanguage…………………………………………………………………………………... 10
Essential Knowledge………………………………………………………………………… 10
Self-Help………………………………………………………………………………………….. 14
Let Us Check…………………………………………………………………………………….. 14
Let Us Analyze…………………………………………………………………………………. 14
In a Nutshell…………………………………………………………………………………….. 14
Q & A List ………………………………………………………………………………………… 15
Keywords Index………………………………………………………………………………. 15
Big Picture in Focus: ULO-1b…………………………………………………………………..….. 15
Metalanguage…………………………………………………………………………………... 15
Essential Knowledge………………………………………………………………………… 16
Self-Help…………………………………………………………………………………………. 21
Let Us Check…………………………………………………………………………………….. 21
Let Us Analyze…………………………………………………………………………………. 22
In a Nutshell…………………………………………………………………………………….. 22
Q & A List ………………………………………………………………………………………… 22
Keywords Index………………………………………………………………………………. 22
Big Picture in Focus: ULO-2a…………………………………………………………………..….. 23
Metalanguage…………………………………………………………………………………... 23

2
Essential Knowledge………………………………………………………………………… 23
Self-Help…………………………………………………………………………………………. 26
Let Us Check…………………………………………………………………………………….. 26
Let Us Analyze…………………………………………………………………………………. 26
In a Nutshell…………………………………………………………………………………….. 26
Q & A List ………………………………………………………………………………………… 27
Keywords Index………………………………………………………………………………. 27
Big Picture in Focus: ULO-2b…………………………………………………………………..….. 27
Metalanguage…………………………………………………………………………………... 27
Essential Knowledge………………………………………………………………………… 28
Self-Help………………………………………………………………………………………….. 33
Let Us Check…………………………………………………………………………………….. 34
Let Us Analyze…………………………………………………………………………………. 34
In a Nutshell…………………………………………………………………………………….. 34
Q & A List ………………………………………………………………………………………… 35
Keywords Index………………………………………………………………………………. 35
Big Picture in Focus: ULO-3a…………………………………………………………………..….. 35
Metalanguage…………………………………………………………………………………... 35
Essential Knowledge………………………………………………………………………… 36
Self-Help………………………………………………………………………………………….. 41
Let Us Check…………………………………………………………………………………….. 42
Let Us Analyze…………………………………………………………………………………. 42
In a Nutshell…………………………………………………………………………………….. 42
Q & A List ………………………………………………………………………………………… 42
Keywords Index………………………………………………………………………………. 42
Big Picture in Focus: ULO-4a…………………………………………………………………..….. 43
Metalanguage…………………………………………………………………………………... 43
Essential Knowledge………………………………………………………………………… 43
Self-Help………………………………………………………………………………………….. 54
Let Us Check…………………………………………………………………………………….. 55
Let Us Analyze…………………………………………………………………………………. 55

3
In a Nutshell…………………………………………………………………………………….. 56
Q & A List ………………………………………………………………………………………… 56
Keywords Index………………………………………………………………………………. 56
Big Picture in Focus: ULO-5a…………………………………………………………………..….. 57
Metalanguage…………………………………………………………………………………... 57
Essential Knowledge………………………………………………………………………… 57
Self-Help………………………………………………………………………………………….. 69
Let Us Check…………………………………………………………………………………….. 69
Let Us Analyze…………………………………………………………………………………. 70
In a Nutshell…………………………………………………………………………………….. 70
Q & A List ………………………………………………………………………………………… 70
Keywords Index………………………………………………………………………………. 71
Big Picture in Focus: ULO-6a…………………………………………………………………..….. 71
Metalanguage…………………………………………………………………………………... 71
Essential Knowledge………………………………………………………………………… 72
Self-Help………………………………………………………………………………………….. 85
Let Us Check…………………………………………………………………………………….. 85
Let Us Analyze…………………………………………………………………………………. 85
In a Nutshell…………………………………………………………………………………….. 86
Q & A List ………………………………………………………………………………………… 86
Keywords Index………………………………………………………………………………. 86
Big Picture in Focus: ULO-6b…………………………………………………………………..….. 87
Metalanguage…………………………………………………………………………………... 87
Essential Knowledge………………………………………………………………………… 87
Self-Help………………………………………………………………………………………….. 92
Let Us Check…………………………………………………………………………………….. 92
Let Us Analyze…………………………………………………………………………………. 92
In a Nutshell…………………………………………………………………………………….. 93
Q & A List ………………………………………………………………………………………… 93
Keywords Index………………………………………………………………………………. 93
Big Picture in Focus: ULO-7a…………………………………………………………………..….. 93
4
Metalanguage…………………………………………………………………………………... 94
Essential Knowledge………………………………………………………………………… 94
Self-Help………………………………………………………………………………………….. 105
Let Us Check…………………………………………………………………………………….. 105
Let Us Analyze…………………………………………………………………………………. 105
In a Nutshell…………………………………………………………………………………….. 106
Q & A List ………………………………………………………………………………………… 106
Keywords Index………………………………………………………………………………. 106
Big Picture in Focus: ULO-8a…………………………………………………………………..….. 106
Metalanguage…………………………………………………………………………………... 107
Essential Knowledge………………………………………………………………………… 107
Self-Help………………………………………………………………………………………….. 112
Let Us Check…………………………………………………………………………………….. 112
Let Us Analyze…………………………………………………………………………………. 113
In a Nutshell…………………………………………………………………………………….. 113
Q & A List ………………………………………………………………………………………… 113
Keywords Index………………………………………………………………………………. 113
Big Picture in Focus: ULO-8b…………………………………………………………………..….. 114
Metalanguage…………………………………………………………………………………... 114
Essential Knowledge………………………………………………………………………… 114
Self-Help………………………………………………………………………………………….. 123
Let Us Check…………………………………………………………………………………….. 123
Let Us Analyze…………………………………………………………………………………. 123
In a Nutshell…………………………………………………………………………………….. 124
Q & A List ………………………………………………………………………………………… 124
Keywords Index………………………………………………………………………………. 124

5
Course Outline: CEE 103 – Engineering Calculus 2

Course Coordinator: Engr. Jay Al S. Gallenero


Email: jgallenero@umindanao.edu.ph
Student Consultation: You can leave a message using the blackboard LMS
Mobile:
Phone: (082) 2961084 loc.133
Effectivity Date: August 2020
Mode of Delivery: Blended (On-Line with face to face or virtual sessions)
Time Frame: 108 Hours
Student Workload: Expected Self-Directed Learning
Requisites: CEE 101 – ENGINEERING CALCULUS 1
Credit: 5

Course Outline Policy

Areas of Concern Details


Contact and Non-contact Hours This 5-unit course self-instructional manual is designed
for blended learning mode of instructional delivery with
scheduled face to face or virtual sessions. The expected
number of ho666666urs will be 90 including the face to
face or virtual sessions. The face to face sessions shall
include the summative assessment tasks (exams) since
this course is crucial in the licensure examination for
engineers.
Assessment Task Submission Submission of assessment tasks shall be on 3rd, 5th, 7th and
9th week of the term. The assessment paper shall be
attached with a cover page indicating the title of the
assessment task (if the task is performance), the
name of the course coordinator, date of submission and
name of the student. The document should be emailed
to the course coordinator. It is also expected that you
already paid your tuition and other fees before the
submission of the assessment task.

If the assessment task is done in real time through the


features in the Blackboard Learning Management
System, the schedule shall be arranged ahead of time
by the course coordinator.
Turnitin Submission (if necessary) To ensure honesty and authenticity, all assessment
tasks are required to be submitted through Turnitin with
a maximum similarity index of 30% allowed. This means

6
that if your paper goes beyond 30%, the students will
either opt to redo her/his paper or explain in writing
addressed to the course coordinator the reasons for the
similarity. In addition, if the paper has reached more
than 30% similarity index, the student may be called for
a disciplinary action in accordance with the University’s
OPM on Intellectual and Academic Honesty.

Please note that academic dishonesty such as


cheating and commissioning other students or people
to complete the task for you have severe punishments
(reprimand, warning, expulsion).
Penalties for Late The score for an assessment item submitted after the
Assignments/Assessments designated time on the due date, without an approved
extension of time, will be reduced by 5% of the possible
maximum score for that assessment item for each day
or part day that the assessment item is late.

However, if the late submission of assessment


paper has a valid reason, a letter of explanation
should be submitted and approved by the course
coordinator. If necessary, you will also be required
to present/attach evidences.
Return of Assignments/ Assessment tasks will be returned to you two (2)
Assessments weeks after the submission. This will be returned by
email or via Blackboard portal.

For group assessment tasks, the course coordinator


will require some or few of the students for online or
virtual sessions to ask clarificatory questions to
validate the originality of the assessment task
submitted and to ensure that all the group members
are involved.
Assignment Resubmission Resubmission of assignment will not be allowed in this
class.
Re-marking of Assessment You should request in writing addressed to the
Papers and Appeal program coordinator your intention to appeal or
contest the score given to an assessment task. The
letter should explicitly explain the reasons/points to
contest the grade. The program coordinator shall
communicate with the students on the approval and
disapproval of the request.

If disapproved by the course coordinator, you can


elevate your case to the program head or the dean with
the original letter of request. The final decision will come
from the dean of the college.

7
Grading System All culled from BlackBoard sessions and
traditional contact

Course discussions/exercises –
30% 1st formative assessment –
10%
2nd formative assessment – 10%
3rd formative assessment – 10%

All culled from on-campus/onsite sessions


(TBA):

Final exam – 40%

Submission of the final grades shall follow the


usual University system and procedures.
Preferred Referencing Style APA 6th Edition.
Student Communication You are required to create a umindanao email
account which is a requirement to access the
BlackBoard portal. Then, the course coordinator
shall enroll the students to have access to the
materials and resources of the course. All
communication formats: chat, submission of
assessment tasks, requests etc. shall be through the
portal and other university recognized platforms.

You can also meet the course coordinator in


person through the scheduled face to face
sessions to raise your issues and concerns.

For students who have not created their student


email, please contact the course coordinator or
program head.
Contact Details of the Dean Dr. Charlito L. Cañesares
Email: charlitocanesares@umindanao.edu.ph
Phone: 082-296-1084
Contact Details of the Program Engr. Randy E. Angelia
Head Email: randy_angelia@umindanao.edu.ph
Phone: 082-296-1084/09423595648
Students with Special Needs Students with special needs shall communicate with the
course coordinator about the nature of his or her special
needs. Depending on the nature of the need, the
course coordinator with the approval of the program
coordinator may provide alternative assessment tasks
or extension of the deadline of submission of
assessment tasks. However, the alternative
assessment tasks should still be in the service of
achieving the desired course learning outcomes.

8
Instructional Help Desk Contact CEE BLACKBOARD ADMINISTRATOR
Details Engr. Jethron J. Adtoon
Email: jadtoon@umindanao.edu.ph
Phone: 09055267834

CEE
Frida Santa O. Bacani
Email: cee@umindanao.edu.ph
Mobile: 09562082442
Phone: 082-2272902

Well-being Welfare Support Help LIC


Desk Contact Details Brigada E. Bacani
Email: library@umindanao.edu.ph
Mobile: 0951-376-6681

GSTC
Ronadora E. Deala, RPsy, RPm, RGC, LPT
Email: ronadora_deala@umindanao.edu.ph
09212122846

Silvino P. Josol
Email: gstcmain@umindanao.edu.ph
09060757721

Course Information – see/download course syllabus in the Black Board LMS

CC’s Voice: Hello students! Welcome to this course, CEE 103: Engineering Calculus 2,
the integral calculus. By now, I am confident that you really wanted to
become an engineer and that you have visualized yourself already
working in the field of engineering of your choice. This course is one of the
most important courses in the field of engineering. The knowledge and
concepts in this course were used to derive important engineering formulas
that are being used in the field of practice.

CO CO1. Demonstrate understanding on the concept and importance of


fundamental integration formula, power, logarithm, exponential,
trigonometric, transformation of trigonometric functions, inverse
trigonometric, integration by parts, integration by substitution, integration of
rational fractions for both definite and indefinite integral and improper and
multiple integral.

CO2. Solve related problems such as plane areas, volume of solid of


revolution, centroid of plane areas, centroid of solid of revolution, moment of
inertia, work and force using integration.

Let us begin!

9
Big Picture

Week 1-3: Unit Learning Outcomes (ULO): At the end of the unit, you are expected to

a. define the basic principles and concepts of integration and derive some
fundamental integration formula from standard differentiation formulas,
and;
b. determine the antiderivatives or the indefinite integrals of a given
functions using the fundamental integration formulas.

Big Picture in Focus: ULOa. Define the basic principles and


concepts of integration and derive some fundamental integration
formula from standard differentiation formulas.

Metalanguage

In this section, the fundamental concepts in Integral Calculus will be discussed to


demonstrate ULOa and serves as the foundation in understanding the subject. Please
refer to these definitions in case you will encounter difficulty in understanding educational
concepts.

1. Integral Calculus. It is the branch of calculus concerned with the determination of


integrals and their application to the solution of differential equations, the
determination of areas and volumes.
2. Integration. It is the reverse process of differentiation.
3. Integrand. It refers to the function to be integrated and immediately follows the
integral sign.
4. ∫. It is the integral symbol or sign. It is an elongated S denoting sum.
5. Indefinite Integral. It is an integral with no restrictions imposed on its independent
variables. It is also called antiderative or primitive integral.
6. Constant of Integration. An arbitrary constant added to the antiderative of a
function.

Essential Knowledge

To perform the aforesaid big picture (unit learning outcomes) for the first three (3)
weeks of the course, you need to review the fundamental concepts in Integral Calculus
that will be laid down in the succeeding pages. Please note that you are not limited
to exclusively refer to these resources. Thus, you are expected to utilize other books,
research articles and other resources that are available in the university’s library e.g.
ebrary, search.proquest.com etc.

10
Definition of Integration.
Integration is the inverse operation to differentiation. In differentiation, we solve
for the differential of a given function whereas in integration, we solve for the
function corresponding to a given differential. The resulting function is called the
integral of the differential.

Indefinite Integral
The collection of all the possible antiderivatives of a given function is called the
indefinite integral. The indefinite integral comprises of the antiderivative and the
constant of integration (𝑪) . The presence of this constant of integration (in
indefinite integration) introduces a family of functions which have the same
derivative in all points in their domain.

If 𝑭(𝒙) is a function whose derivative 𝑭′ (𝒙) = 𝒇(𝒙) on a certain interval of the x


axis, then 𝑭(𝒙) is called an antiderivative or indefinite integral of 𝒇(𝒙) . The
indefinite integral of a given function is not unique; for example, 𝒙𝟐 , 𝒙𝟐 − 𝟔, 𝒙𝟐 +
𝟑 are all indefinite integrals of 𝒇(𝒙) = 𝟐𝒙. All indefinite integrals of 𝒇(𝒙) = 𝟐𝒙 are
then included in 𝑭(𝒙) = 𝒙𝟐 + 𝑪.

The Notation
The symbol ∫ 𝒇(𝒙)𝒅𝒙 is used to indicate the indefinite integral of 𝒇(𝒙). Thus, we
write ∫ 𝟐𝒙 𝒅𝒙 = 𝒙𝟐 + 𝑪. In the expression ∫ 𝒇(𝒙)𝒅𝒙, the function 𝒇(𝒙) is called the
integrand and is mathematically expressed as:

The Concept of Integration


• For example, find the indefinite integral for 𝒇(𝒙) = 𝒙𝟓 .
Base on the definition if 𝑭(𝒙) is such a function we should have:

(𝑭(𝒙)) = 𝒙𝟓
𝒙𝟔
Obviously, the function satisfies the above equation but other functions such
𝟔
𝒙𝟔 𝒙𝟔
as + 𝟏, − 𝟑, and etc. can be considered as an answer so we can write
𝟔 𝟔
the answer generally as:
𝒙𝟓
∫ 𝒙𝟒 𝒅𝒙 = +𝑪
𝟓

11
• Using the definition of the indefinite integral, we can find the integral of a
simple function directly:
• ∫ 𝟎 𝒅𝒙 = 𝑪
• ∫ 𝟏 𝒅𝒙 = 𝒙 + 𝑪
𝒙𝒏+𝟏
• ∫ 𝒙𝒏 𝒅𝒙 = + 𝑪 (𝒏 ≠ −𝟏)
𝒏+𝟏
𝟏
• ∫ 𝒙−𝟏 𝒅𝒙 = ∫ 𝒙 𝒅𝒙 = 𝒍𝒏 |𝒙| + 𝑪 (𝒙 ≠ 𝟎)
𝒂𝒙
• ∫ 𝒂𝒙 𝒅𝒙 = ∫ 𝒍𝒏 𝒂 + 𝑪 (𝒂 ≠ 𝟏, 𝒂 > 𝟎)
• ∫ 𝒄𝒐𝒔 𝒙 𝒅𝒙 = 𝒔𝒊𝒏 𝒙 + 𝑪
• ∫ 𝒔𝒊𝒏 𝒙 𝒅𝒙 = −𝒄𝒐𝒔 𝒙 + 𝑪

Rules of Integration
1. The derivative of the indefinite integral is the integrand:

(∫ 𝒇(𝒙)𝒅𝒙) = 𝒇(𝒙)

2. The differential of the indefinite integral is equal to the element of the integration:

𝒅 (∫ 𝒇(𝒙)𝒅𝒙) = 𝒇(𝒙)𝒅𝒙

3. The indefinite integral of a differential of a function is equal to that function plus a


constant:

∫ 𝒅(𝑭(𝒙)) = 𝑭(𝒙) + 𝑪

4. If 𝒂 ≠ 𝟎 and is a constant, then:


∫ 𝒂𝒇(𝒙)𝒅𝒙 = 𝒂 ∫ 𝒇(𝒙)𝒅𝒙
➢ A constant coefficient goes in and comes out of the integral sign.

5. The indefinite integral of the sum or difference of two integrable functions is equal
to the sum or difference of their individual indefinite integral:
∫[𝒇(𝒙) ± 𝒈(𝒙)]𝒅𝒙 = ∫ 𝒇(𝒙)𝒅𝒙 ± ∫ 𝒈(𝒙)𝒅𝒙

6. If ∫ 𝒇(𝒙)𝒅𝒙 = 𝑭(𝒙) + 𝑪, then:


𝟏
∫ 𝒇(𝒂𝒙 + 𝒃)𝒅𝒙 = 𝑭(𝒂𝒙 + 𝒃 ) + 𝑪
𝒂
And if 𝒃 = 𝟎, then
𝟏
∫ 𝒇(𝒂𝒙 )𝒅𝒙 = 𝑭(𝒂𝒙 ) + 𝑪
𝒂

12
o Using the last rule, we can easily calculate some integrals without applying a
specific method:

Example:
𝒅𝒙
a. ∫ 𝒙−𝒂 = 𝒍𝒏 |𝒙 − 𝒂| + 𝑪
−𝒄𝒐𝒔 (𝒂𝒙 )
b. ∫ 𝒔𝒊𝒏(𝒂𝒙 )𝒅𝒙 = +𝑪
𝒂

The Substitution Method of Integration


o If the integrand is in the form of 𝒇(𝒈(𝒙))𝒈′(𝒙) 𝒅𝒙, and substituting 𝒖 = 𝒈(𝒙),
then we will have
∫ 𝒇(𝒈(𝒙))𝒈′(𝒙) 𝒅𝒙 = ∫ 𝒇(𝒖)𝒖′ 𝒅𝒙 = ∫ 𝒇(𝒖) 𝒅𝒖
o And if ∫ 𝒇(𝒖) 𝒅𝒖 = 𝑭(𝒖) + 𝑪, then:
∫ 𝒇(𝒈(𝒙))𝒈′(𝒙) 𝒅𝒙 = 𝑭(𝒈(𝒙)) + 𝑪

Example 1:
𝒙
Find the indefinite integral ∫ 𝒅𝒙.
𝟏+𝒙𝟐

Solution:

Let 𝒖 = 𝟏 + 𝒙𝟐 , then 𝒅𝒖 = 𝟐𝒙𝒅𝒙, and we will have:


𝟏

𝒙
𝒅𝒙 = ∫ 𝟐 𝒅𝒖 = 𝟏 ∫ 𝒅𝒖 = 𝟏 𝒍𝒏 |𝒖| + 𝑪
𝟏 + 𝒙𝟐 𝒖 𝟐 𝒖 𝟐
𝟏 𝟏
since 𝒖 = 𝟏 + 𝒙𝟐 , thus, 𝟐 𝒍𝒏 |𝒖| + 𝑪 = 𝒍𝒏 |𝟏 + 𝒙𝟐 | + 𝑪
𝟐

Example 2:

𝒅𝒙
Find ∫ , ( 𝒙 > 𝟎)
𝒙𝒍𝒏 𝒙

Solution:
𝒅𝒙
Let 𝒖 = 𝒍𝒏 𝒙, then 𝒅𝒖 = , and:
𝒙
𝒅𝒙 𝒙𝒅𝒖 𝒅𝒖
∫ 𝒙𝒍𝒏 𝒙 = ∫ 𝒙𝒖
= ∫ 𝒖
= 𝒍𝒏 |𝒖| + 𝑪 = 𝒍𝒏 |𝒍𝒏 (𝒙)| + 𝑪

Note that, having success with this method requires finding a relevant substitution, which
comes after lots of practice.

13
Self-Help: You can also refer to the sources below to help you further
understand the lesson.

* Hass, J., Weir, M., Thomas Jr, G. (2012). University Calculus: early transcendental.
Philippines : Pearson Education South Asia.
* Stewart, J. (2000). Calculus: Concepts and Contexts. Pacipic Grove, CA: Brook/Cole.

Let’s Check.

Activity 1. Evaluate the following expressions using the fundamental principles of


integration you learned in this section. Show the step by step process of your solution.

1. ∫(3 − 2𝑥 − 𝑥 4 ) 𝑑𝑥 𝑑𝑥
5. ∫ 3𝑥 + 1
𝑑𝑥
2. ∫ (𝑥− 1)3 3𝑥𝑑𝑥
6. ∫ 𝑥 2 +2
3. ∫( 𝑥 2 − 1)𝑥 𝑑𝑥 3
7. ∫ 𝑐𝑜𝑠 5 𝑥 𝑑𝑥
𝑥 𝑑𝑥
4. ∫ (𝑥 2 + 1)3
8. ∫(𝑐𝑜𝑠 2𝑥 − 2𝑠𝑖𝑛3𝑥)𝑑𝑥

Let’s Analyze.

Activity 1. In this activity, you are required to integrate again the following expressions
and show the details of your work.

5𝑥 − 1
1. ∫( 𝑎𝑥 2 + 𝑏)2⁄3 𝑥 𝑑𝑥 3. ∫ 2𝑥 + 3 𝑑𝑥
𝑥+𝑎 𝑥+𝑏
2. ∫ (𝑠𝑖𝑛 𝑚
+ 𝑐𝑜𝑠 𝑛
) 𝑑𝑥 4. ∫( 𝑐𝑜𝑠 𝑥 − 𝑠𝑖𝑛 𝑥)2 𝑑𝑥

In a Nutshell

The importance of learning the basic concept of integral and its formulas are undeniable.
In this portion of the unit, you will be required to state your arguments or synthesis
relevant to the topics presented. I will supply the first two items and you will continue the
rest.

1. The reverse process of differentiation is the integration process.


2. Identifying the appropriate formula to use for a given problem is essential.
14
Your Answer:

3. __________________________________________________________________________

4. __________________________________________________________________________

5. __________________________________________________________________________

Q&A List

Questions/Issues Answer
1.
2.
3.
4.
5.

Keywords Index

This section lists down the keywords that help students for recall. Keywords are similar
to Metalanguage but without the definitions and descriptions. This section includes
concepts (a word or phrase), ideas, theories, names of people, and other vital terms to
remember. Technically speaking, all those included in the Metalanguage should be part
of the Keywords. However, Keywords can also include other important concepts or ideas
not stipulated in the Metalanguage. This section also helps in the review.

• Integrand • Antidifferentiation
• Integral • Constant of Integration
• Differential • Chain Rule

Big Picture in Focus: ULOb. Determine the antiderivatives or the


indefinite integrals of a given functions using the fundamental
integration formulas.

Metalanguage
For you to demonstrate ULOb, you will need to familiarize with the essential terms
and symbols relevant to the discussion in this section. Please refer to these definitions in
case you will encounter these terms and symbols. Also, please note that you will be
15
required to refer to the previous principles found in ULOa section.

Terms/Symbols Meaning
∫ 𝑓(𝑥)𝑑𝑥 Integral of 𝑓 with respect to x

𝑓(𝑥) 𝑖𝑛 ∫ 𝑓(𝑥)𝑑𝑥 Integrand

𝑥 𝑖𝑛 ∫ 𝑓(𝑥)𝑑𝑥 Variable of integration


Integrate Find the integral
An integral of 𝑓 A function 𝐹 such that 𝐹 ′(𝑥) = 𝑓(𝑥)
Integration The process of finding the integral
Any real number 𝐶, considered as constant in the
Constant of Integration
function

Essential Knowledge

Fundamental Integration Formulas.

Since integration is the inverse of differentiation, to integrate any given function we must
reduce it to one or more of the differentials of the elementary functions, expressed by the
fundamental formula of Differential Calculus. Formulas below follow immediately from the
standard differentiation formulas.

A. Basic Integrals

𝑑
1. ∫ 𝑑𝑢 [𝑓(𝑢)]𝑑𝑢 = 𝑓(𝑢) + 𝐶

2. ∫[𝑓(𝑢) + g(𝑢)] 𝑑𝑢 = ∫ 𝑓(𝑢)𝑑𝑢 + ∫ g(𝑢)𝑑𝑢


3. ∫ 𝑑𝑢 = 𝑢 + 𝐶
4. ∫ 𝑎𝑑𝑢 = 𝑎 ∫ 𝑑𝑢 = 𝑎𝑢 + 𝐶 , (𝑎 is constant)
𝑢𝑛+1
5. ∫ 𝑢𝑛 𝑑𝑢 = + 𝐶 , (n ≠ 1)
𝑛+1
𝑑𝑢
6. ∫ = 𝑙𝑛 |𝑢| + 𝐶
𝑢

B. Logarithmic Functions

𝑎𝑢
7. ∫ 𝑎𝑢 𝑑𝑢 = + 𝐶 , (a > 1, 𝑎 ≠ 1 )
𝑙𝑛 𝑎

8. ∫ 𝑙𝑛𝑢 𝑑𝑢 = 𝑢 𝑙𝑛 |𝑢| − 𝑢 + 𝐶

16
C. Trigonometric Functions

9. ∫ 𝑠𝑖𝑛 𝑢 𝑑𝑢 = − 𝑐𝑜𝑠 𝑢 + 𝐶
10. ∫ 𝑐𝑜𝑠 𝑢 𝑑𝑢 = 𝑠𝑖𝑛 𝑢 + 𝐶
11. ∫ 𝑡𝑎𝑛 𝑢 𝑑𝑢 = 𝑙𝑛 |𝑠𝑒𝑐| + 𝐶
12. ∫ 𝑐𝑜𝑡 𝑢 𝑑𝑢 = 𝑙𝑛 |𝑠𝑖𝑛| + 𝐶
13. ∫ 𝑠𝑒𝑐 𝑢 𝑑𝑢 = 𝑙𝑛 |𝑠𝑒𝑐 𝑢 + 𝑡𝑎𝑛 𝑢| + 𝐶
14. ∫ 𝑐𝑠𝑐 𝑢 𝑑𝑢 = 𝑙𝑛 |𝑐𝑠𝑐 𝑢 − 𝑐𝑜𝑡 𝑢| + 𝐶
15. ∫ 𝑠𝑒𝑐 2 𝑢 𝑑𝑢 = 𝑡𝑎𝑛 𝑢 + 𝐶
16. ∫ 𝑐𝑠𝑐 2 𝑢 𝑑𝑢 = −𝑐𝑜𝑡 𝑢 + 𝐶
17. ∫ 𝑠𝑒𝑐 𝑢 𝑡𝑎𝑛 𝑢 𝑑𝑢 = 𝑠𝑒𝑐 𝑢 + 𝐶
18. ∫ 𝑐𝑠𝑐 𝑢 𝑐𝑜𝑡 𝑢 𝑑𝑢 = −𝑐𝑠𝑐 𝑢 + 𝐶

Proof of formula 5 and 6.

To derive formula 5,
Since 𝑑 (𝑢𝑛+1 ) = (𝑛 + 1)𝑢𝑛 𝑑𝑢,
therefore 𝑢𝑛+1 = ∫(𝑛 + 1)𝑢𝑛 𝑑𝑢 = (𝑛 + 1) ∫ 𝑢𝑛 𝑑𝑢.
𝑢𝑛+1
Hence ∫ 𝑢𝑛 𝑑𝑢 = 𝑛+1

Formula 6 follows directly from


𝑑𝑢
𝑑(𝑙𝑛|𝑢|) = 𝑢

Note: Formula 5 applies to all values of 𝑛 except 𝒏 = −1. For this value it gives
𝑢−1+1 𝑢0
∫ 𝑢−1 𝑑𝑢 = −1 + 1
= 0
= ∞ , formula 6 provides for this failing case.

Example:

Integrate the following expressions.

For Equations 1 – 3: Basic Integrals

1. Evaluate ∫(𝑥 + 3)11 𝑑𝑥.

17
Solution:

∫(𝑥 + 3)11 𝑑𝑥= ∫ 𝑢11 𝑑𝑢 with the use of substitution method, we can replace 𝑥 + 3 with 𝑢. Then
𝑑𝑥 = 𝑑𝑢. And use formula 5 above.
𝑢11 + 1 1
= ∫ 𝑢11 𝑑𝑢 = + 𝐶 = 𝑢12 + 𝐶 then substitute 𝑢 with 𝑥 + 3
11+1 12
1
= 12 (𝑥 + 3)12 + 𝐶

1
2. Evaluate ∫(𝑥 2 + 3)2 𝑥 𝑑𝑥.

Solution:
1
If we apply formula 5, letting 𝑢 = 𝑥 2 + 1, and 𝑛 = 2; then 𝑑𝑢 = 2𝑥 𝑑𝑥. We must then
1
introduce a factor 2 before the 𝑥 𝑑𝑥, and consequently its reciprocal 2 on the left of ∫, thus

1 1
1
∫(𝑥 2 + 3)2 𝑥 𝑑𝑥 = 2 ∫(𝑥 2 + 3)2 2𝑥 𝑑𝑥
3
1 (𝑥 2 +1)2
=2 3 +𝐶
2
3
2
(𝑥 +1)2
= +𝐶
3

Another solution: By Substitution Method

Let 𝑢 = 𝑥 2 + 1 and 𝑢 = 2𝑑𝑥 , thus

1 1
𝑑𝑢
∫(𝑥 2 + 3)2 𝑥 𝑑𝑥 = ∫ 𝑢2 𝑥 2𝑥
1
1
= 2∫𝑢 2 𝑑𝑢 then use formula 5
3
1 𝑢2
=2 3 +𝐶
2
3
1 (𝑥 2 +1)2
=2 3 + 𝐶
2
3
(𝑥 2 +1)2
= + C
3

18
𝑥2
3. Evaluate ∫ 4 𝑑𝑥.
√𝑥 3 + 2

Solution:

𝑥2 1
∫4 𝑑𝑥 = 3 ∫(𝑥 3 + 2)−1/4 (3𝑥 2 )𝑑𝑥
√𝑥 3 + 2
−1/4+1
1 (𝑥 3 +2)
= +𝐶
3 −1/4+1
1 4
= ( ) (𝑥 3 + 2)3/4 + 𝐶
3 3
4
= (𝑥 3 + 2)3/4 + 𝐶
9

Note: It can also be evaluated by making the substitution 𝑢 = 𝑥 3 + 2, and 𝑑𝑢 = 3𝑑𝑥

For Equations 4 – 6: Logarithmic Functions


𝑑𝑥
4. Evaluate ∫ .
𝑥

Solution:
𝑑𝑥
∫ = 𝑙𝑛|𝑥| + 𝐶 by formula 6
𝑥

𝑑𝑥
5. Evaluate the integral ∫ 2𝑥−3.

Solution:
𝑑𝑥 1 2𝑑𝑥
∫ 2𝑥−3 = 2 ∫ 2𝑥−3
1
= 2 𝑙𝑛|2𝑥 − 3| + 𝐶

Another solution: By Substitution Method

Let 𝑢 = 2𝑥 − 3, 𝑑𝑢 = 2𝑑𝑥, thus

𝑑𝑥 𝑑𝑢
∫ 2𝑥−3 = ∫ 2𝑢
1
= 2 𝑙𝑛|𝑢| + 𝐶
1
= 2 𝑙𝑛|2𝑥 − 3| + 𝐶 Substituting back the value of u.

6. Evaluate ∫ 𝑎2𝑥 𝑑𝑥.

19
Solution:

1
∫ 𝑎2𝑥 𝑑𝑥 = 2 ∫ 𝑎2𝑥 (2𝑑𝑥)
1 𝑎2𝑥
= 2 𝑙𝑛 𝑎 + 𝐶

For Equations 7 – 10: Trigonometric Functions

Proof:
−𝑠𝑖𝑛 𝑢
∫ 𝑡𝑎𝑛 𝑢 𝑑𝑢 = − ∫ 𝑑𝑢 = − 𝑙𝑛 |𝑐𝑜𝑠 𝑢| + 𝐶 = 𝑙𝑛|𝑐𝑜𝑠|−1 + 𝐶 = 𝑙𝑛 |𝑠𝑒𝑐| + 𝐶
𝑐𝑜𝑠 𝑢
𝑐𝑜𝑠 𝑢
∫ 𝑐𝑜𝑡 𝑢 𝑑𝑢 = ∫ 𝑠𝑖𝑛 𝑢 𝑑𝑢 = 𝑙𝑛 |𝑠𝑖𝑛 𝑢| + 𝐶
𝑠𝑒𝑐 𝑢(𝑡𝑎𝑛 𝑢+𝑠𝑒𝑐 𝑢) 𝑠𝑒𝑐 𝑢 𝑡𝑎𝑛 𝑢 𝑑𝑢+ 𝑠𝑒𝑐 2 𝑢 𝑑𝑢
∫ 𝑠𝑒𝑐 𝑢 𝑑𝑢 = ∫ 𝑑𝑢 = ∫ = 𝑙𝑛 |𝑠𝑒𝑐 𝑢 + 𝑡𝑎𝑛 𝑢| + 𝐶
𝑡𝑎𝑛 𝑢 +𝑠𝑒𝑐 𝑢 𝑡𝑎𝑛 𝑢+ 𝑠𝑒𝑐 𝑢

7. Evaluate ∫ 𝑐𝑜𝑠 3𝑥 𝑑𝑥.

Solution:
1
∫ 𝑐𝑜𝑠 3𝑥 𝑑𝑥 = 3 ∫ 𝑐𝑜𝑠3𝑥(3𝑑𝑥)
1
= 3 𝑠𝑖𝑛3𝑥 + 𝐶

8. Evaluate ∫ 𝑠𝑖𝑛2 𝑥 𝑐𝑜𝑠 𝑥 𝑑𝑥.

Solution:

By substitution: let 𝑢 = 𝑠𝑖𝑛 𝑥, 𝑑𝑢 = 𝑐𝑜𝑠𝑥 𝑑𝑥, thus


𝑑𝑢
∫ 𝑠𝑖𝑛2 𝑥 𝑐𝑜𝑠 𝑥 𝑑𝑥 = ∫ 𝑢2 𝑐𝑜𝑠 𝑥 𝑐𝑜𝑠 𝑥
2 𝑢3
= ∫ 𝑢 𝑑𝑢 = +𝐶
3
𝑠𝑖𝑛3 𝑥
= +𝐶
3

9. Evaluate ∫(1 + 𝑡𝑎𝑛 𝑥)2 𝑑𝑥.

Solution:

∫(1 + 𝑡𝑎𝑛 𝑥)2 𝑑𝑥 = ∫(1 + 2𝑡𝑎𝑛 𝑥 + 𝑡𝑎𝑛2 𝑥 ) 𝑑𝑥


= ∫(𝑠𝑒𝑐 2 𝑥 + 2𝑡𝑎𝑛 𝑥 ) 𝑑𝑥
= 𝑡𝑎𝑛 𝑥 + 2ln |𝑠𝑒𝑐 𝑥| + 𝐶

10. Evaluate ∫ 𝑒 3𝑐𝑜𝑠2𝑥 𝑠𝑖𝑛 2𝑥 𝑑𝑥.

20
Solution:
1
∫ 𝑒 3𝑐𝑜𝑠2𝑥 𝑠𝑖𝑛 2𝑥 𝑑𝑥 = - 6 ∫ 𝑒 3𝑐𝑜𝑠2𝑥 (−6𝑠𝑖𝑛2𝑥𝑑𝑥)

𝑒 3𝑐𝑜𝑠2𝑥
=− +𝐶
6

Another solution: By Substitution Method

Let 𝑢 = 3𝑐𝑜𝑠2𝑥, 𝑑𝑢 = −6𝑠𝑖𝑛2𝑥𝑑𝑥, thus

𝑑𝑢
∫ 𝑒 3𝑐𝑜𝑠2𝑥 𝑠𝑖𝑛 2𝑥 𝑑𝑥 = ∫ 𝑒 𝑢 𝑠𝑖𝑛 2𝑥 (− 6𝑠𝑖𝑛2𝑥)
1
= - 6 ∫ 𝑒 𝑢 𝑑𝑢
1
= - 6 𝑒𝑢 + 𝐶
𝑒 3𝑐𝑜𝑠2𝑥
=- +𝐶
6

Self-Help: You can also refer to the sources below to help you further
understand the lesson.

* Hass, J., Weir, M., Thomas Jr., G. (2012). University Calculus: Early Transcendental.
Philippines: Pearson Education South Asia.
* Stewart, J. (2000). Calculus: Concepts and Contexts. Pacific Grove, CA: Brook/Cole.
* Ayres Jr., F. & Mendelson, E. (1990). Schaum’s Outline of Theory and Problems of
Differential and Integral Calculus. USA: The McGraw-Hill Companies, Inc.

Let’s Check

Activity 1. After learning to solve integral equations from the examples discussed you
will now be able to evaluate the following indefinite integral using the fundamental integral
formulas introduced above.

1. ∫ 𝒙𝟔 𝒅𝒙 𝒙 𝒅𝒙
8. ∫ 𝒙𝟐 −𝟏
𝟑
2. ∫ √𝒛 𝒅𝒛 (𝒙𝟐 − 𝒂𝟐 ) 𝒅𝒙
9. ∫ 𝒙𝟑 −𝟑𝒂𝟐 𝒙
3. ∫(𝟏 − 𝒙 ) √𝒙 𝒅𝒙
𝟏
𝟐 10. ∫ 𝒔𝒊𝒏 𝟐 𝒙 𝒅𝒙
4. ∫(𝟑𝒙 + 𝟒) 𝒅𝒙
𝟖𝒙𝟐 11. ∫ 𝒙 𝒄𝒐𝒕 𝒙𝟐 𝒅𝒙
5. ∫ 𝟑 𝒅𝒙
(𝒙𝟑 +𝟐) 𝒔𝒊𝒏 𝒚 𝒅𝒚
12. ∫
𝟑 𝒄𝒐𝒔𝟐 𝒚
6. ∫ √𝟏 − 𝒙𝟐 𝒙𝒅𝒙
13. ∫(𝒕𝒂𝒏 𝟐𝒙 + 𝒔𝒆𝒄 𝟐𝒙)𝟐 𝒅𝒙
𝒅𝒙
7. ∫
𝒙+𝟐 14. ∫ 𝒄𝒔𝒄 𝒙 𝒅𝒙
21
Let’s Analyze

Activity 1. In this activity you are required to integrate again the following functions
using the fundamental integration formulas and techniques learned previously.

1. ∫(2𝑥 9 − 3𝑥 6 + 12𝑥 4 − 3)𝑑𝑥 (𝑦−1)3


3. ∫ 𝑑𝑦
𝑦
2. ∫ (𝑥 2 3 3
− 2) 𝑥 𝑑𝑥
4. ∫(𝑎𝑏 2 )𝑥 𝑑𝑥

In a Nutshell
In integration, most of the fundamental formulas follow immediately from the standard
differentiation formulas. However, there are functions which do not have an equivalent
integral but can be manipulated mathematically in such a way that it will have a
corresponding integral. It is very important that the students master these formulas and
the techniques being used in integration as these are the foundation in understanding
and mastering the subject.

Q&A List

Questions/Issues Answer
1.
2.
3.
4.
5.

Keywords Index

This section lists down the keywords that help students for recall. Keywords are similar
to Metalanguage but without the definitions and descriptions. This section includes
concepts (a word or phrase), ideas, theories, names of people, and other vital terms to
remember. Technically speaking, all those included in the Metalanguage should be part
of the Keywords. However, Keywords can also include other important concepts or ideas
not stipulated in the Metalanguage. This section also helps in the review.

• Logarithmic Functions • Trigonometric Functions

22
Big Picture

Week 4-5: Unit Learning Outcomes (ULO): At the end of the unit, you are expected to

a. determine the antiderivatives of the indefinite integrals of a given


functions for exponential, inverse trigonometric and hyperbolic
functions, and;
b. determine the principles and fundamental theorems of definite integral
of a continuous function over a closed interval.

Big Picture in Focus: ULOa. Determine the antiderivatives of the


indefinite integrals of a given functions for exponential, inverse
trigonometric and hyperbolic functions.

Metalanguage

This section is a continuation of the previous unit learning outcomes which you will
need to familiarize the essential terms and symbols relevant to the discussion and also
note that you will be required to refer to the previous principles of the previous ULOs.

Essential Knowledge

Fundamental Integration Formulas (Continuation)

Since integration is the inverse of differentiation, to integrate any given function we must
reduce it to one or more of the differentials of the elementary functions, expressed by the
fundamental formula of Differential Calculus. Formulas below follow immediately from the
standard differentiation formulas.

D. Exponential Functions

8. ∫ 𝑒 𝑢 𝑑𝑢 = 𝑒 𝑢 + 𝐶

E. Inverse Trigonometric Functions


𝑑𝑢 𝑢
9. ∫ √𝑎2 = 𝑎𝑟𝑐𝑠𝑖𝑛 + 𝐶
− 𝑢2 𝑎
𝑑𝑢 1 𝑢
10.∫ 𝑎2 + 𝑢2 = 𝑎𝑟𝑐𝑡𝑎𝑛 + 𝐶
𝑎 𝑎

23
𝑑𝑢 1 𝑢
11. ∫ = 𝑎 𝑎𝑟𝑐𝑠𝑒𝑐 + 𝐶
𝑢√𝑢2 − 𝑎2 𝑎

F. Hyperbolic Functions

12. ∫ 𝑠𝑖𝑛ℎ 𝑢 𝑑𝑢 = 𝑐𝑜𝑠ℎ 𝑢 + 𝐶


13. ∫ 𝑐𝑜𝑠ℎ 𝑢 𝑑𝑢 = 𝑠𝑖𝑛ℎ 𝑢 + 𝐶
14.∫ 𝑡𝑎𝑛ℎ 𝑢 𝑑𝑢 = 𝑙𝑛|𝑐𝑜𝑠ℎ 𝑢| + 𝐶
15. ∫ 𝑐𝑜𝑡ℎ 𝑢 𝑑𝑢 = 𝑙𝑛|𝑠𝑖𝑛ℎ 𝑢| + 𝐶
16. ∫ 𝑠𝑒𝑐ℎ2 𝑢 𝑑𝑢 = 𝑡𝑎𝑛ℎ 𝑢 + 𝐶
17. ∫ 𝑐𝑠𝑐ℎ2 𝑢 𝑑𝑢 = −𝑐𝑜𝑡ℎ 𝑢 + 𝐶
18. ∫ 𝑠𝑒𝑐ℎ 𝑢 𝑡𝑎𝑛ℎ 𝑢 𝑑𝑢 = −𝑠𝑒𝑐ℎ 𝑢 + 𝐶
19. ∫ 𝑐𝑠𝑐ℎ 𝑢 𝑐𝑜𝑡ℎ 𝑢 𝑑𝑢 = −𝑐𝑠𝑐ℎ 𝑢 + 𝐶

For Equations 10 - 12: Exponential Functions

11. Evaluate ∫ 𝑒 −𝑥 𝑑𝑥.

Solution:

By substitution: let 𝑢 = −𝑥, 𝑑𝑢 = −𝑑𝑥, thus

∫ 𝑒 −𝑥 𝑑𝑥 = − ∫ 𝑒 𝑢 𝑑𝑢
= − 𝑒𝑢 + 𝐶
= − 𝑒 −𝑥 + 𝐶

𝑒 1/𝑥
12. Evaluate ∫ 𝑑𝑥.
𝑥2

Solution:

𝑒 1/𝑥 𝑑𝑥
∫ 𝑑𝑥 = − ∫ 𝑒 1/𝑥 (− 𝑥 2 )
𝑥2
= − 𝑒 1/𝑥 + 𝐶
1 𝑑𝑥
Note: It can also be solved by substitution. Let 𝑢 = 𝑥 , 𝑑𝑢 = − 𝑥2

For Equations 13 – 15: Inverse Trigonometric Functions:


𝑑𝑥
13. Evaluate ∫ √1 .
− 𝑥2

Solution:
24
𝑑𝑥
∫ √1 − 𝑥2 = 𝑎𝑟𝑐𝑠𝑖𝑛 𝑥 + 𝐶 where 𝑎2 = 1, 𝑎 = 1 , 𝑢2 = 𝑥 2 , 𝑢 = 𝑥 , 𝑑𝑢 = 𝑑𝑥

𝑑𝑥
14. Evaluate ∫ .
𝑥√ 𝑥 4 − 1

Solution:

Since 𝑢2 = 𝑥 4 , 𝑢 = 𝑥 2 𝑎𝑛𝑑 𝑑𝑢 = 2𝑥𝑑𝑥, so to normalize, multiply both the numerator and


denominator by 2x, thus the resulting expression becomes

𝑑𝑥 1 𝑥𝑑𝑥
∫ 𝑥√ 𝑥 4− 1 = 2∫
𝑥 2 √(𝑥 2 )2 −1
1
= 2 𝑎𝑟𝑐𝑠𝑒𝑐 𝑥 2 + 𝐶

𝑑𝑥
15. Evaluate ∫ 𝑒 𝑥 + 𝑒 −𝑥 .

Solution:

Since there is no equivalent formula for this expression, it needs to be manipulated first
so that the fundamental integral formulas listed above can be applied, hence

𝑑𝑥 𝑒 𝑥 𝑑𝑥
∫ 𝑒 𝑥 + 𝑒 −𝑥 = ∫ 𝑒 2𝑥 + 1
= 𝑎𝑟𝑐𝑡𝑎𝑛 𝑒 𝑥 + 𝐶

For Equations 16 - 17: Hyperbolic Functions:

16. Evaluate ∫ 𝑐𝑜𝑠ℎ (4𝑥 + 3) 𝑑𝑥.

Solution:

By substitution method, let 𝑢 = 4𝑥 + 3 and 𝑑𝑢 = 4𝑑𝑥, thus

1
∫ 𝑐𝑜𝑠ℎ (4𝑥 + 3) 𝑑𝑥 = 4 ∫ 𝑐𝑜𝑠ℎ (4𝑥 + 3) 4𝑑𝑥 by normalization
1
= 𝑠𝑖𝑛ℎ (4𝑥 + 3) + 𝐶
4

17. Evaluate ∫ 𝑥𝑡𝑎𝑛ℎ 𝑥 2 𝑑𝑥.

Solution:
1 1
∫ 𝑥𝑡𝑎𝑛ℎ 𝑥 2 𝑑𝑥 = ∫ 2𝑥𝑡𝑎𝑛ℎ 𝑥 2 𝑑𝑥
2
=
2
𝑙𝑛 |𝑐𝑜𝑠ℎ 𝑥 2 | + 𝐶

25
Self-Help: You can also refer to the sources below to help you further
understand the lesson.

* Hass, J., Weir, M., Thomas Jr., G. (2012). University Calculus: Early Transcendental.
Philippines: Pearson Education South Asia.
* Stewart, J. (2000). Calculus: Concepts and Contexts. Pacific Grove, CA: Brook/Cole.
* Ayres Jr., F. & Mendelson, E. (1990). Schaum’s Outline of Theory and Problems of
Differential and Integral Calculus. USA: The McGraw-Hill Companies, Inc.

Let’s Check

Activity 1. After learning to solve integral equations for exponential, inverse trigonometric
and hyperbolic functions from the examples discussed you will now be able to evaluate
the following indefinite integral using the fundamental integral formulas introduced above.

1. ∫ 𝒆𝟑𝒙 𝒅𝒙 4. ∫
𝒅𝒙
√𝟐𝟓 −𝟏𝟔𝒙𝟐
𝒅𝒙
2. ∫ 𝒆𝒙 +𝟏 (𝒙+𝟑)𝒅𝒙
5. ∫
√𝟏−𝒙𝟐
𝒅𝒙
3. ∫ (𝒙+𝟏)
𝒙√ 𝒙𝟐 − 𝟏
6. ∫ 𝒙𝟐 − 𝟒𝒙 + 𝟖 𝒅𝒙

Let’s Analyze

Activity 1. In this activity you are required to integrate again the following functions using
the fundamental integration formulas and techniques learned previously.

𝑑𝑥
1. ∫(𝑒 𝑠𝑖𝑛 𝑥 𝑐𝑜𝑠 𝑥 − 𝑎𝑐𝑜𝑠 2𝑥 𝑠𝑖𝑛 2𝑥)𝑑𝑥 6. ∫ 2𝑥 2 − 3𝑥+ 5
2. ∫(𝒆𝒕𝒂𝒏 𝒙 𝒔𝒆𝒄 𝒙 − 𝒆𝒔𝒆𝒄𝒙 𝒕𝒂𝒏𝒙)𝒔𝒆𝒄 𝒙 𝒅𝒙 2−𝑥
𝑑𝑥
7. ∫ 4𝑥 2 +4𝑥 −3 𝑑𝑥
3. ∫ 𝑥+2
𝑥√𝑎2 𝑥 2 − 16 8. ∫ √4𝑥− 𝑑𝑥
3𝑥−2 𝑥2
4. ∫ 𝑥 2 + 9 𝑑𝑥 𝑑𝑥
3𝑥−2
9. ∫ 𝑐𝑠𝑐 2𝑥 − 𝑐𝑜𝑡 2𝑥
5. ∫ √9− 𝑑𝑥 𝑠𝑒𝑐 𝑥 𝑡𝑎𝑛𝑥 𝑑𝑥
𝑥2 10. ∫ 𝑑𝑥
𝑎 + 𝑏𝑠𝑒𝑐 𝑥

In a Nutshell
In integration, most of the fundamental formulas follow immediately from the standard
differentiation formulas. However, there are functions which do not have an equivalent
integral but can be manipulated mathematically in such a way that it will have a
26
corresponding integral. It is very important that the students master these formulas and
the techniques being used in integration as these are the foundation in understanding
and mastering the subject.

Q&A List

Questions/Issues Answer
1.
2.
3.
4.
5.

Keywords Index

This section lists down the keywords that help students for recall. Keywords are similar
to Metalanguage but without the definitions and descriptions. This section includes
concepts (a word or phrase), ideas, theories, names of people, and other vital terms to
remember. Technically speaking, all those included in the Metalanguage should be part
of the Keywords. However, Keywords can also include other important concepts or ideas
not stipulated in the Metalanguage. This section also helps in the review.

• Exponential Functions • Hyperbolic Functions


• Inverse Trigonometric Functions

Big Picture in Focus: ULOb. Determine the principles and


fundamental theorems of definite integral of a continuous function
over a closed interval.

Metalanguage

In this section, the concept of definite integral will be discussed to demonstrate


ULOb and to be utilized in the application of Integral Calculus to different mathematical
problems. Please refer to these definitions (terms/symbols) in case you will encounter
difficulty in understanding educational concepts.

1. Definite integral. It is an integral that is defined by the limit values a and b of the
independent variable.
2. Upper and lower limits. The boundaries of integration.

27
3. Riemann Sums. The theory of limits of finite approximations that was made
precise by the German mathematician Bernhard Riemann.
4. 𝒃 − 𝒂. The width of the interval [𝑎, 𝑏].
5. Lim. It means “Limit”.
6. →. Symbol indicating that one quantity approaches another.
7. → +∞. It means increases endlessly or “approaches infinity”
8. Sigma notation. Denoted as 𝛴. It enables us to write a sum with many terms in the
compact form.
𝒃
9. ∫𝒂 𝒇(𝒙)𝒅𝒙. This is read as, “the integral from a to b of 𝑓(𝑥)𝑑𝑥.”
10. ]𝒃𝒂 . Symbol for evaluation over interval from a to b.

Essential Knowledge
To perform the aforesaid big picture (unit learning outcomes) for the next two (2)
weeks of the course, you need to review the concepts and theorems of definite integral
that will be laid down in the succeeding pages. Please note that you are not limited
to exclusively refer to these resources. Thus, you are expected to utilize other books
and other resources that are available in the university’s library e.g. ebrary,
search.proquest.com etc.

Definite Integral

• The definite integral is the key tool in calculus for defining and calculating quantities
important to mathematics and science, such as areas, volumes, and lengths of curved
paths. (Stewart, 2000)

• The basis for formulating definite integrals is the construction of appropriate finite
sums. Although we need to define precisely what we mean by the area of a general
region in the plane, or the average value of a function over a closed interval, we do
have intuitive ideas of what these notions mean. So, in this section we begin our
approach to integration by approximating these quantities with finite sums. We also
consider what happens when we take more and more terms in the summation
process. In subsequent sections we look at taking the limit of these sums as the
number of terms goes to infinity, which then leads to precise definitions of the
quantities being approximated here. (Stewart, 2000)

• For example, we are going to find the area under the curve 𝒚 = 𝒇(𝒙) over the interval
[𝑎, 𝑏] (as shown in figure 1). It is not easy to find the area of a region with curve sides.
One way to calculate this area is to divide the area into 𝑛 equal sub-interval such as
[𝑎, 𝑥1 ] , [𝑥1 , 𝑥2 ] , … , [𝑥𝑛−1 , 𝑏] , (each with the length equal to ∆𝑥 ) and construct
rectangles (see figure 1). (Stewart, 2000)

28
Figure 1 Figure 2

• The width of the interval [𝑎, 𝑏] is 𝑏 − 𝑎, so the width of each of the n strips is

𝒃− 𝒂
∆𝒙 =
𝒏

Let’s approximate the ith strip Si by a rectangle with width ∆𝒙 and height 𝑓(𝑥𝑖 ), which is
the value of 𝑓 at the right-hand endpoint (see Figure 2). Then the area of the ith rectangle
is 𝑓(𝑥𝑖 )∆𝒙 . What we think of intuitively as the area of is approximated by the sum of the
areas of these rectangles, which is

𝑹𝒏 = 𝒇(𝒙𝟏 )∆𝒙 + 𝒇(𝒙𝟐 )∆𝒙 + 𝒇(𝒙𝟏 )∆𝒙 ⋯ + 𝒇(𝒙𝒏 )∆𝒙

Figure 3 shows this approximation for, n = 4, 8, and 12. Notice that this approximation
appears to become better and better as the number of strips increases, that is, as 𝒏 → ∞.
Therefore, we define the area of the region in the following way.

The area A of the region S that lies under the graph of the continuous function 𝒇 is the
limit of the sum of the areas of approximating rectangles:

𝑨 = 𝒍𝒊𝒎 𝑹𝒏 = 𝒍𝒊𝒎[𝒇(𝒙𝟏 )∆𝒙 + 𝒇(𝒙𝟐 )∆𝒙 + 𝒇(𝒙𝟏 )∆𝒙 ⋯ + 𝒇(𝒙𝒏 )∆𝒙]


𝒏→∞ 𝒏→∞

Figure 3

• Obviously, the area under the curve can be estimated as 𝑹 = ∑𝒏𝒊=𝟏 𝒇(𝒙∗𝒊 ) (∆𝒙) (which
is called the Riemann Sum) and our approximation of this sum gets better and better
29
if the number of sub-interval goes to infinity, which is equivalent to say ∆𝒙 → 𝟎, in this

case the value of the area approaches to a limit:


𝒏

𝑺 = 𝒍𝒊𝒎 ∑ 𝒇(𝒙∗𝒊 )(∆𝒙)


𝒏→∞
𝒊=𝟏

• We call this sum as a definite integral of y = 𝑓(𝑥) over the interval [𝑎,𝑏] and it can be
𝒙 𝒃
shown as ∫𝒙 𝒏=𝒃 𝒇(𝒙)𝒅𝒙 or simply ∫𝒂 𝒇(𝒙)𝒅𝒙. Therefore:
𝟎=𝒂

𝒃 𝒏

∫ 𝒇(𝒙)𝒅𝒙 = 𝒍𝒊𝒎 ∑ 𝒇(𝒙∗𝒊 )(∆𝒙)


𝒂 𝒏→∞
𝒊=𝟏

• The component parts in the integral symbol also have names: (Stewart, 2000)

• If 𝑓(𝑥) takes both positive and negative values, the area under the curve and confined
by the x-axis and lines 𝑥 = 𝑎 and 𝑥 = 𝑏 (see figure 4) is the sum of areas above the x-
axis minus the sum of areas under the x-axis, i.e.:

𝒃
∫ 𝒇(𝒙)𝒅𝒙 = 𝑷𝒐𝒔𝒊𝒕𝒊𝒗𝒆 𝑨𝒓𝒆𝒂 − 𝑵𝒆𝒈𝒂𝒕𝒊𝒗𝒆 𝑨𝒓𝒆𝒂
𝒂

Figure 4

30
Properties of Definite Integrals

1. If 𝑎 = 𝑏, then the area under the curve is zero

𝒃
∫ 𝒇(𝒙)𝒅𝒙 = 𝟎
𝒂

2. If 𝑎 and 𝑏 exchange their position the new definite integral is the negative of the
previous integral:
𝒃 𝒂
∫ 𝒇(𝒙)𝒅𝒙 = − ∫ 𝒇(𝒙)𝒅𝒙
𝒂 𝒃

3. If 𝑦 = k over the interval [𝑎,𝑏], then:

𝒃 𝒃 𝒃
∫𝒂 𝒇(𝒙)𝒅𝒙 = ∫𝒂 𝒌 𝒅𝒙 = 𝒌 ∫𝒂 𝒅𝒙 = 𝒌(𝒃 − 𝒂)

where k is any constant

4. If 𝑓(𝑥) ≥ 0 and it is continuous over the interval [𝑎,𝑏] and the interval can be
divided into sub-intervals such as [𝑎, 𝑥1 ], [𝑥1 , 𝑥2 ] …, [𝑥𝑛−1 , 𝑏], then:

𝒃 𝒙𝟏 𝒙𝟐 𝒃
∫ 𝒇(𝒙)𝒅𝒙 = ∫ 𝒇(𝒙)𝒅𝒙 + ∫ 𝒇(𝒙)𝒅𝒙 + ⋯ + ∫ 𝒇(𝒙)𝒅𝒙
𝒂 𝒂 𝒙𝟏 𝒙𝒏 −𝟏

5. If 𝒇(𝒙) and 𝒈(𝒙) are both continuous over the interval [𝑎,𝑏], then:

𝒃 𝒃 𝒃
∫ [𝒇(𝒙) + 𝒈(𝒙)]𝒅𝒙 = ∫ 𝒇(𝒙)𝒅𝒙 + ∫ 𝒈(𝒙)𝒅𝒙
𝒂 𝒂 𝒂

6. If 𝑭(𝒙) is any anti-derivative (primitive) function of 𝒇(𝒙), then:

𝒃
∫ 𝒇(𝒙)𝒅𝒙 = 𝑭(𝒃) − 𝑭(𝒂)
𝒂

The Substitution Rule for Definite Integrals

•If 𝒈′ is continuous on [𝒂, 𝒃] and 𝒇 is continuous on the range of 𝒖 = 𝒈(𝒙), then


𝒃 𝒈(𝒃)
∫ 𝒇(𝒈(𝒙)) 𝒈′(𝒙) 𝒅𝒙 = ∫ 𝒇(𝒖) 𝒅𝒖
𝒂 𝒈(𝒂)

31
Example:

Use the fundamental theorem of definite integral to evaluate the following:


1
1. ∫−1(2x 2 − x 3 )dx.

Solution:
1
1 2x3 x4
∫−1(2x 2 − x 3 )dx =[ 3
− 4
] Substitute the limits into the resulting antiderivative
−1
2(1)3 (1)4 2(−1)3 (−1)4
=[ − ]−[ − ]
3 4 3 4
2 1 2 1
= (3 − 4) − (− 3 − 4)
4
=3

𝟒 𝒅𝒙
2. ∫𝟏
√𝒙

Solution:

4 𝑑𝑥 4
∫1 = [ 2√𝑥 ]1
√𝑥
= 2 (√4 − √1 )
=2

1 2
3. ∫0 𝑥(1 − √𝑥) dx

Solution:

1 2 1
∫0 𝑥(1 − √𝑥) dx = ∫0 x(1 − 2√𝑥 + 𝑥)dx
1
= ∫0 (x − 2𝑥 3/2 + x 2 )dx
1
x2 x5/2 x3
= [2 − 5/2
+ 3 0
]
1
= 30

10 𝑑𝑥
4. ∫6 𝑥−2

Solution:

10 𝑑𝑥
∫6 = [𝑙𝑛 |𝑥 − 2|]10
6
𝑥−2
= 𝑙𝑛 |10 − 2| − 𝑙𝑛 |6 − 2|
= 𝑙𝑛 8 − 𝑙𝑛 4
32
8
= 𝑙𝑛 4
= 𝑙𝑛 2

𝜋/2 𝑐𝑜𝑠 𝑥 𝑑𝑥
5. ∫0 2𝑠𝑖𝑛 𝑥+1

Solution:
𝜋/2 𝑐𝑜𝑠 𝑥 𝑑𝑥 1 ⁄2
∫0 = 2 [𝑙𝑛 |2𝑠𝑖𝑛 𝑥 + 1|]𝜋0
2𝑠𝑖𝑛 𝑥+1
1
= 2 (𝑙𝑛 3 – 𝑙𝑛 1 )
1
= 2 𝑙𝑛 3

Note: When evaluating definite integral of a trigonometric functions, calculator should be


in “radian mode”.

Odd and Even Functions

A function is an even function when f(-x) = f(x). Consequently, a function is an odd function
is an odd function if f(-x) = -f(x).

Example:

Identify whether the given functions below are even or odd.

1. f(x) = cos (-2x), is an even function, since from trigonometry identity and definition of
an even function cos –a = cos a.

2. f(x) = 2x is an odd function since f(-x) = -2x by the definition of an odd function.

Self-Help: You can also refer to the sources below to help you further
understand the lesson.

* Stewart, J. (2000). Calculus: Concepts and Contexts. Australia: Cengage.


* Downing, Douglas (2009). Mathematics Terms 3rd Edition. New York: Barron’s
Educational Series Inc.
* Feliciano and Uy, F. (1983). Differential and Integral Calculus. Manila: Merriam and
Webster Bookstore, Inc.
* Peterson, Thurman. (1964). Calculus with Analytic Geometry. Philippines: Harper and
Row Publishers, Incorporated.

33
Let’s Check

Activity 1. Evaluate the following definite integrals by using the fundamental theorem of
Integral Calculus.
2 5
1. ∫0 (2𝑥 − 1)9 𝑑𝑥 2. ∫0 √6 + 4𝑥 𝑑𝑥

1 𝜋/4
3. ∫0 𝑥 3 (2 + 3𝑥 4 )3 𝑑𝑥 4. ∫0 𝑒 𝑠𝑖𝑛2𝑥 𝑐𝑜𝑠2𝑥𝑑𝑥

5 𝑒 √2𝑥 3 𝑑𝑥
5. ∫1 𝑑𝑥 6. ∫1
√2𝑥 5𝑥+2

7 𝑑𝑥 𝑒3 𝑑𝑥
7. ∫2 3 8. ∫𝑒
√(1+3𝑥)2 𝑥√𝑙𝑛 𝑥

𝑎 𝜋/4 𝑠𝑖𝑛 𝑥
9. ∫0 𝑥√𝑎2 − 𝑥 2 10. ∫0 1+𝑐𝑜𝑠2 𝑥

Let’s Analyze

Activity 1. Evaluate the definite integral. Show your solutions neatly and legibly.
2 4
1. ∫−1(3𝑥 2 − 2𝑥 3 )𝑑𝑥 2. ∫1 (𝑥 2 − 1)√𝑥 𝑑𝑥

𝜋⁄2 𝑠𝑖𝑛 2𝑥 𝜋/4


3. ∫0 𝑑𝑥 4. ∫𝜋/6 (2 + 𝑠𝑒𝑐)2 𝑑𝑥
3+𝑐𝑜𝑠 (2𝑥)

2 2
5. ∫1 (2𝑥 3 + 𝑥)√2𝑥 2 + 1 𝑑𝑥 6. ∫1 (1 + 3𝑥 )2 𝑑𝑥

𝜋⁄4 1+𝑠𝑖𝑛 𝑥 𝜋⁄3 𝑠𝑒𝑐 2 3𝑥


7. ∫0 𝑑𝑥 8. ∫𝜋⁄4 𝑑𝑥
𝑐𝑜𝑠 (𝑥) 2+4𝑡𝑎𝑛 3𝑥

𝜋 𝑑𝑥 3 (3𝑥−2)
9. ∫0 10. ∫0
4𝑥 2 −4𝑥+5 𝑥 2 +2𝑥+17

In a Nutshell.

A continuous function over a closed interval has a definite integral, which is the limit of
any Riemann sum for the function. Definite integral can be link to the concept of area.

34
Q&A List

Questions/Issues Answer
1.
2.
3.
4.
5.

Keywords Index

This section lists down the keywords that help students for recall. Keywords are similar
to Metalanguage but without the definitions and descriptions. This section includes
concepts (a word or phrase), ideas, theories, names of people, and other vital terms to
remember. Technically speaking, all those included in the Metalanguage should be part
of the Keywords. However, Keywords can also include other important concepts or ideas
not stipulated in the Metalanguage. This section also helps in the review.

• Lower and Upper Limit • Sub-interval


• Interval • Sigma Notation
• Riemann Sum • Area
• Finite Sum • Definite Integral

Big Picture

Week 6-7: Unit Learning Outcomes (ULO): At the end of the unit, you are expected to

a. Familiarize and practice the different integration techniques.

Big Picture in Focus: ULOa. Familiarize and practice the different


integration techniques.

Metalanguage

In this section, additional integration techniques will be discussed to achieve


ULOa. To do this, you need to know first the essential terms and symbols that you will
encounter in the following discussion for a quicker understanding of the topics to be
discussed.

35
Integration Techniques

These techniques are used when one cannot evaluate the integral of a function by direct
application of the formulas given in the first module. They can be evaluated after a suitable
transformation has been made. The techniques below will orient you on the elementary
method for changing the form of the integral.

Essential Knowledge

Integration by Parts

Integration by parts corresponds to the product rule for differentiation. Recall that
the product rule states that if 𝒇 and 𝒈 are differentiable functions, then

𝒅
[𝒇(𝒙)𝒈(𝒙)] = 𝒇(𝒙)𝒈′(𝒙) + 𝒈(𝒙)𝒇′(𝒙)
𝒅𝒙

If we get the antiderivative of the result above, then

∫[𝒇(𝒙)𝒈′(𝒙) + 𝒈(𝒙)𝒇′(𝒙)] 𝒅𝒙 = 𝒇(𝒙)𝒈(𝒙)


or
∫ 𝒇(𝒙)𝒈′(𝒙) 𝒅𝒙 + ∫ 𝒈(𝒙)𝒇′(𝒙) 𝒅𝒙 = 𝒇(𝒙)𝒈(𝒙)

Rearranging the equation, we have

∫ 𝒇(𝒙)𝒈′(𝒙) 𝒅𝒙 = 𝒇(𝒙)𝒈(𝒙) − ∫ 𝒈(𝒙)𝒇′(𝒙) 𝒅𝒙

The resulting equation above is the formula for Integration by Parts. It would be easier
to remember if we let 𝒖 = 𝒇(𝒙) and 𝒗 = 𝒈(𝒙) . Then the differentials are 𝒅𝒖 = 𝒇′(𝒙)𝒅𝒙
and 𝒅𝒗 = 𝒈′(𝒙)𝒅𝒙, thus
∫ 𝒇(𝒙)𝒈′(𝒙) 𝒅𝒙 = 𝒇(𝒙)𝒈(𝒙) − ∫ 𝒈(𝒙)𝒇′(𝒙) 𝒅𝒙

∫ 𝒖𝒅𝒗 = 𝒖𝒗 − ∫ 𝒗 𝒅𝒖

NOTE: The aim for using integration by parts is to obtain a simpler integral than the
original given. In general, in using the function for 𝒖 and 𝒗, choose 𝒖 = 𝒇(𝒙) to be a
function that becomes simpler when differentiated (or at least not more complicated) as
long as 𝒅𝒗 = 𝒈′(𝒙)𝒅𝒙 can be readily integrated to give 𝒗. Thus, in using integration by
parts consider the following rules:

36
Two general rules can be stated:

1. The part selected as 𝒅𝒗 must be readily integrable.


2. ∫ 𝒗 𝒅𝒖 must not be more complex than ∫ 𝒖 𝒅𝒗 .

Example:

Evaluate the following by integration by parts.

1. ∫ 𝑥 𝑠𝑖𝑛 𝑥 𝑑𝑥

Solution:

Let 𝑓(𝑥) = 𝑥 and 𝑔′(𝑥) = 𝑠𝑖𝑛 𝑥 𝑑𝑥,


then 𝑓′(𝑥) = 1 and 𝑔(𝑥) = ∫ 𝑔′(𝑥) = ∫ 𝑠𝑖𝑛 𝑥 𝑑𝑥 = −𝑐𝑜𝑠 𝑥

Thus,

∫ 𝑥 𝑠𝑖𝑛 𝑥 𝑑𝑥 = 𝑓(𝑥)𝑔(𝑥) − ∫ 𝑔(𝑥)𝑓′(𝑥) 𝑑𝑥 = 𝑥(−𝑐𝑜𝑠 𝑥) − ∫(−𝑐𝑜𝑠 𝑥) 𝑑𝑥


= −𝑥𝑐𝑜𝑠 𝑥 + ∫ 𝑐𝑜𝑠 𝑥 𝑑𝑥
= −𝑥𝑐𝑜𝑠 𝑥 + 𝑠𝑖𝑛 𝑥 + 𝐶

The answer can also be checked by differentiating it, the result should be 𝒙 𝒔𝒊𝒏 𝒙.

Another Method:

Let 𝑢 = 𝑥 and 𝑑𝑣 = 𝑠𝑖𝑛 𝑥 𝑑𝑥,


then 𝑑𝑢 = 𝑑𝑥 and 𝑣 = ∫ 𝑑𝑣 = ∫ 𝑠𝑖𝑛 𝑥 𝑑𝑥 = −𝑐𝑜𝑠 𝑥

Thus,

∫ 𝑥 𝑠𝑖𝑛 𝑥 𝑑𝑥 = 𝑢𝑣 − ∫ 𝑣 𝑑𝑢
= 𝑥(−𝑐𝑜𝑠 𝑥) − ∫(−𝑐𝑜𝑠 𝑥) 𝑑𝑥
= −𝑥𝑐𝑜𝑠 𝑥 + ∫ 𝑐𝑜𝑠 𝑥 𝑑𝑥
= −𝑥𝑐𝑜𝑠 𝑥 + 𝑠𝑖𝑛 𝑥 + 𝐶

In the first example we started with∫ 𝑥 𝑠𝑖𝑛 𝑥 𝑑𝑥 and expressed it in terms of the simpler
integral ∫ 𝑐𝑜𝑠 𝑥 𝑑𝑥. If we had chosen 𝑢 = 𝑠𝑖𝑛 𝑥 and 𝑑𝑣 = 𝑥𝑑𝑥 , 𝑑𝑢 = 𝑐𝑜𝑠 𝑥𝑑𝑥 and 𝑣 =
𝑥2
2
, so integration by parts gives
𝑥2 1
∫ 𝑥 𝑠𝑖𝑛 𝑥 𝑑𝑥 = 𝑠𝑖𝑛 𝑥 ( 2 ) − 2 ∫ 𝑥 2 𝑐𝑜𝑠𝑥 𝑑𝑥

37
The resulting integral ∫ 𝑥 2 𝑐𝑜𝑠𝑥 𝑑𝑥 is a more complicated than the one we started with.
Thus, in solving integral using integration by parts, it is very important that we take note
of the general rules stated above.

2. ∫ 𝑥 2 𝑒 𝑥 𝑑𝑥

Solution:

Since 𝑥 2 becomes simpler when differentiated whereas 𝑒 𝑥 remains unchanged when


differentiated or integrated which satisfies the two general rules stated above, thus;

Let 𝑢 = 𝑥2 𝑑𝑣 = 𝑒 𝑥 𝑑𝑥
then 𝑑𝑢 = 2𝑥𝑑𝑥 𝑣 = ∫ 𝑑𝑣 = ∫ 𝑒 𝑥 𝑑𝑥 = 𝑒 𝑥

By integration by parts, we have

∫ 𝑥 2 𝑒 𝑥 𝑑𝑥 = 𝑥 2 𝑒 𝑥 − ∫ 𝑒 𝑥 (2𝑥𝑑𝑥) = 𝑥 2 𝑒 𝑥 − 2 ∫ 𝑥𝑒 𝑥 𝑑𝑥

As you can see the resulting integral ∫ 𝑥𝑒 𝑥 𝑑𝑥 is simpler than the original integral which
satisfies the condition stated in the rule above. Solving further, by integration by parts
again,

Let 𝑢=𝑥 𝑑𝑣 = 𝑒 𝑥 𝑑𝑥
then 𝑑𝑢 = 𝑑𝑥 𝑣 = ∫ 𝑑𝑣 = ∫ 𝑒 𝑥 𝑑𝑥 = 𝑒 𝑥

Thus,

∫ 𝑥 2 𝑒 𝑥 𝑑𝑥 = 𝑥𝑒 𝑥 − ∫ 𝑒 𝑥 𝑑𝑥
= 𝑥𝑒 𝑥 − 𝑒 𝑥 + 𝐶

Substituting this to the result above, we get

∫ 𝑥 2 𝑒 𝑥 𝑑𝑥 = 𝑥 2 𝑒 𝑥 − 2 ∫ 𝑥𝑒 𝑥 𝑑𝑥
= 𝑥 2 𝑒 𝑥 − 2[𝑥𝑒 𝑥 − 𝑒 𝑥 + 𝐶]
= 𝑥 2 𝑒 𝑥 − 2𝑥𝑒 𝑥 + 2𝑒 𝑥 + 𝐶1 where 𝑪𝟏 = −𝟐𝑪

3. ∫ 𝑥 𝐴𝑟𝑐𝑡𝑎𝑛 𝑥 𝑑𝑥

Solution:

Let 𝑢 = 𝐴𝑟𝑐𝑡𝑎𝑛 𝑥 𝑑𝑣 = 𝑥 𝑑𝑥
𝑑𝑥 𝑥2
then 𝑑𝑢 = 1+𝑥 2 𝑣 = ∫ 𝑥 𝑑𝑥 = 2

Thus,

38
𝑥2 𝑥2 𝑑𝑥
∫ 𝑥 𝐴𝑟𝑐𝑡𝑎𝑛 𝑥 𝑑𝑥 = 2
𝐴𝑟𝑐𝑡𝑎𝑛 𝑥 ∫ 2
(1+𝑥 2 )
𝑥2 1 𝑥2 𝒙𝟐
= 𝐴𝑟𝑐𝑡𝑎𝑛 𝑥 − 2 ∫ (1+𝑥 2 ) 𝑑𝑥 Simplify
𝟏+𝒙𝟐
by long division
2
𝑥2 1 1
= 2
𝐴𝑟𝑐𝑡𝑎𝑛 𝑥 − 2 ∫ (1 − 1+𝑥 2
) 𝑑𝑥
𝑥2 1 1
= 2
𝐴𝑟𝑐𝑡𝑎𝑛 𝑥 − 𝑥 + 𝐴𝑟𝑐𝑡𝑎𝑛 𝑥 + 𝐶
2 2

Transformation of Trigonometric Functions

There are four cases in solving functions using integration by trigonometric functions or
by trigonometric integral which will be discussed in details in this section.

Case I: ∫ 𝑠𝑖𝑛𝑚 𝑥 𝑐𝑜𝑠 𝑛 𝑥 𝑑𝑥; where either m or n is a positive odd integer.


Use: 𝑐𝑜𝑠 2 𝑥 = 1 − 𝑠𝑖𝑛2 𝑥

Case II: ∫ 𝑡𝑎𝑛𝑛 𝑥 𝑑𝑥 or ∫ 𝑐𝑜𝑡 𝑛 𝑥 𝑑𝑥; where n is an integer


Use: 𝑡𝑎𝑛2 𝑥 = 𝑠𝑒𝑐 2 𝑥 − 1
𝑐𝑜𝑡 2 𝑥 = 𝑐𝑠𝑐 2 𝑥 − 1

Case III: ∫ 𝑡𝑎𝑛𝑚 𝑥 𝑠𝑒𝑐 𝑛 𝑥 𝑑𝑥 or ∫ 𝑐𝑜𝑡 𝑚 𝑥 𝑐𝑠𝑐 𝑛 𝑐 𝑑𝑥; where n is a positive even integer
Use: 𝑠𝑒𝑐 2 𝑥 = 1 + 𝑡𝑎𝑛2 𝑥
𝑐𝑠𝑐 2 𝑥 = 1 + 𝑐𝑜𝑡 2 𝑥

Case IV: ∫ 𝑠𝑖𝑛𝑚 𝑥 𝑐𝑜𝑠 𝑛 𝑥 𝑑𝑥; where both m and n are positive even integers
1
Use: 𝑠𝑖𝑛2 𝑥 = 2 (1 − cos 2𝑥)
1
𝑐𝑜𝑠 2 𝑥 = 2 (1 + cos 2𝑥)
1
sin 𝑥 cos 𝑥 = 2 sin 2𝑥

Case 1 Examples:

1. Evaluate ∫ 𝑐𝑜𝑠 3 𝑦 𝑠𝑖𝑛3 𝑦 𝑑𝑦.

Solution:

∫ 𝑐𝑜𝑠 3 𝑦 𝑠𝑖𝑛3 𝑦 𝑑𝑦 = ∫ 𝑐𝑜𝑠 2 𝑦 cos 𝑦 𝑠𝑖𝑛3 𝑦 𝑑𝑦


= ∫(1 − 𝑠𝑖𝑛2 𝑦)𝑠𝑖𝑛3 𝑦 cos 𝑦 𝑑𝑦
= ∫ 𝑠𝑖𝑛3 𝑦 cos 𝑦 𝑑𝑦 − ∫ 𝑠𝑖𝑛5 𝑦 cos 𝑦 𝑑𝑦
𝑠𝑖𝑛4 𝑦 𝑠𝑖𝑛6 𝑦
= − +𝐶
4 6

2. Evaluate ∫ 𝑐𝑜𝑠 4 2𝑥 𝑠𝑖𝑛3 2𝑥 𝑑𝑥.

Solution:

39
∫ 𝑐𝑜𝑠 4 2𝑥 𝑠𝑖𝑛3 2𝑥 𝑑𝑥 = ∫ 𝑐𝑜𝑠 4 2𝑥 𝑠𝑖𝑛2 2𝑥 sin 2𝑥 𝑑𝑥
= ∫ 𝑐𝑜𝑠 4 2𝑥 (1 − 𝑐𝑜𝑠 2 2𝑥) sin 2𝑥 𝑑𝑥
= ∫ 𝑐𝑜𝑠 4 2𝑥 sin 2𝑥 𝑑𝑥 − ∫ 𝑐𝑜𝑠 6 2𝑥 sin 2𝑥 𝑑𝑥
1 1
= − 10 𝑐𝑜𝑠 5 2𝑥 + 14 𝑐𝑜𝑠 7 2𝑥 + 𝐶

Case 2 Examples:

1. Evaluate ∫ 𝑡𝑎𝑛3 𝜃 𝑑𝜃.

Solution:

∫ 𝑡𝑎𝑛3 𝜃 𝑑𝜃 = ∫ 𝑡𝑎𝑛2 𝜃 tan 𝜃 𝑑𝜃


= ∫(𝑠𝑒𝑐 2 𝜃 − 1) tan 𝜃 𝑑𝜃
= ∫ tan 𝜃 𝑠𝑒𝑐 2 𝜃 𝑑𝜃 − ∫ tan 𝜃 𝑑𝜃 let u = tan 𝜃; du = 𝑠𝑒𝑐 2 𝜃 𝑑𝜃
𝑡𝑎𝑛2 𝜃
= + ln(cos 𝜃) + 𝐶
2

2. Evaluate ∫ 𝑐𝑜𝑡 4 𝑦 𝑑𝑦.

Solution:

∫ 𝑐𝑜𝑡 4 𝑦 𝑑𝑦 = ∫(𝑐𝑠𝑐 2 𝑦 − 1) 𝑐𝑜𝑡 2 𝑦 𝑑𝑦


= ∫ 𝑐𝑜𝑡 2 𝑦 𝑐𝑠𝑐 2 𝑦 𝑑𝑦 − ∫ 𝑐𝑜𝑡 2 𝑦 𝑑𝑦
𝑐𝑜𝑡 3 𝑦
=− − ∫(𝑐𝑠𝑐 2 𝑦 − 1) 𝑑𝑦
3
𝑐𝑜𝑡 3 𝑦
=− − ∫ 𝑐𝑠𝑐 2 𝑦 𝑑𝑦 + ∫ 𝑑𝑦
3
𝑐𝑜𝑡 3 𝑦
=− + cot 𝑦 + 𝑦 + 𝐶
3

Case 3 Examples:

1. Evaluate ∫ 𝑠𝑒𝑐 4 𝑦 𝑡𝑎𝑛4 𝑦 𝑑𝑦.

Solution:

∫ 𝑠𝑒𝑐 4 𝑦 𝑡𝑎𝑛4 𝑦 𝑑𝑦 = ∫(1 + 𝑡𝑎𝑛2 𝑦) 𝑠𝑒𝑐 2 𝑦 𝑡𝑎𝑛4 𝑦 𝑑𝑦


= ∫ 𝑡𝑎𝑛4 𝑦 𝑠𝑒𝑐 2 𝑦 𝑑𝑦 + ∫ 𝑡𝑎𝑛6 𝑦 𝑠𝑒𝑐 2 𝑦 𝑑𝑦
𝑡𝑎𝑛5 𝑦 𝑡𝑎𝑛7 𝑦
= + +𝐶
5 7

2. Evaluate ∫ 𝑐𝑜𝑡 5 𝑥 𝑐𝑠𝑐 6 𝑥 𝑑𝑥.

Solution:

∫ 𝑐𝑜𝑡 5 𝑥 𝑐𝑠𝑐 6 𝑥 𝑑𝑥 = ∫ 𝑐𝑜𝑡 5 𝑥 (1 + 𝑐𝑜𝑡 2 𝑥)2 𝑐𝑠𝑐 2 𝑥 𝑑𝑥


40
= ∫ 𝑐𝑜𝑡 5 𝑥 (1 + 2𝑐𝑜𝑡 2 𝑥 + 𝑐𝑜𝑡 4 𝑥) 𝑐𝑠𝑐 2 𝑥 𝑑𝑥
= ∫ 𝑐𝑜𝑡 5 𝑥 𝑐𝑠𝑐 2 𝑥 𝑑𝑥 + 2 ∫ 𝑐𝑜𝑡 7 𝑥 𝑐𝑠𝑐 2 𝑥 𝑑𝑥 + ∫ 𝑐𝑜𝑡 9 𝑥 𝑐𝑠𝑐 2 𝑥 𝑑𝑥
1 1 1
= − 6 𝑐𝑜𝑡 6 𝑥 − 4 𝑐𝑜𝑡 8 𝑥 − 10 𝑐𝑜𝑡10 𝑥 + 𝐶

Case 4 Examples:

1. Evaluate ∫ 𝑠𝑖𝑛2 𝑦 𝑐𝑜𝑠 4 𝑦 𝑑𝑦.

Solution:

∫ 𝑠𝑖𝑛2 𝑦 𝑐𝑜𝑠 4 𝑦 𝑑𝑦 = ∫ 𝑠𝑖𝑛2 𝑦 𝑐𝑜𝑠 2 𝑦 𝑐𝑜𝑠 2 𝑦 𝑑𝑦


= ∫(sin 𝑦 cos 𝑦)2 𝑐𝑜𝑠 2 𝑦 𝑑𝑦
1
= ∫(2 𝑠𝑖𝑛2𝑦)2 𝑐𝑜𝑠 2 𝑦 𝑑𝑦
1
= 4 ∫ 𝑠𝑖𝑛2 2𝑦 𝑐𝑜𝑠 2 𝑦 𝑑𝑦
1 1
= 4 ∫ 𝑠𝑖𝑛2 2𝑦 [2 (1 + cos 2𝑦)] 𝑑𝑦
1 1
= 8 ∫ 𝑠𝑖𝑛2 2𝑦 𝑑𝑦 + 8 ∫ 𝑠𝑖𝑛2 2𝑦 cos 2𝑦 𝑑𝑦
1 1 1
= 8 ∫ 2 (1 − cos 4𝑦) 𝑑𝑦 + 8 ∫ 𝑠𝑖𝑛2 2𝑦 cos 2𝑦 𝑑𝑦
1 1 1
= 16 ∫ 𝑑𝑦 − 16 ∫ cos 4𝑦 𝑑𝑦 + 8 ∫ 𝑠𝑖𝑛2 2𝑦 cos 2𝑦 𝑑𝑦
1 1 1 1 1 𝑠𝑖𝑛3 2𝑦
= 16 𝑦 − 16 (4) sin 4𝑦 + 8 (2) +𝐶
3
1 1 1
= 16 𝑦 − 64 sin 4𝑦 + 48 𝑠𝑖𝑛3 2𝑦 + 𝐶

Self-Help: You can also refer to the sources below to help you further
understand the lesson.

* Stewart, J. (2000). Calculus: Concepts and Contexts. Australia: Cengage.


* Downing, Douglas (2009). Mathematics Terms 3rd Edition. New York: Barron’s
Educational Series Inc.
* Feliciano and Uy, F. (1983). Differential and Integral Calculus. Manila: Merriam and
Webster Bookstore, Inc.
* Peterson, Thurman. (1964). Calculus with Analytic Geometry. Philippines: Harper and
Row Publishers, Incorporated.

41
Let’s Check

Activity 1. Solve the following integrals using the integration by parts technique.

1. ∫ 𝑥𝑒 𝑥 𝑑𝑥 𝑦 𝑑𝑦
4. (𝑦−1)4
2. ∫ 𝑦 cos 4𝑦 𝑑𝑦
5. 𝑥 2 𝑒 −𝑥 𝑑𝑥
3 2)
3. ∫ 𝑥 sin(𝑥 𝑑𝑥
6. ∫ 𝑦 ln 𝑦 𝑑𝑦

Let’s Analyze

Activity 1. Evaluate the following integrals using trigonometric transformation. Show


your solutions neatly and legibly.

1. ∫ 𝑠𝑖𝑛2 𝑥 𝑐𝑜𝑠 3 𝑥 𝑑𝑥 3. ∫ 𝑡𝑎𝑛3 3𝑥 𝑠𝑒𝑐 4 3𝑥 𝑑𝑥


𝑥
2. ∫ 𝑐𝑜𝑠 3 3 𝑑𝑥 4. ∫ cot 3𝑥 𝑐𝑠𝑐 4 3𝑥 𝑑𝑥

In a Nutshell

The importance of learning different techniques in integration is very important in helping


you solve complicated problems. In this portion of the unit, you will be required to state
your arguments or synthesis relevant to the topics presented. I will supply the first two
items and you will continue the rest.

1. The integration by parts is the simplest technique to use.


2. Not all techniques can make the integration process easy.

Your Turn:

3. __________________________________________________________________________

4. __________________________________________________________________________

5. __________________________________________________________________________

Q&A List

Questions/Issues Answer
1.
2.
3.
4.
5.

42
Keywords Index

This section lists down the keywords that help students for recall. Keywords are similar
to Metalanguage but without the definitions and descriptions. This section includes
concepts (a word or phrase), ideas, theories, names of people, and other vital terms to
remember. Technically speaking, all those included in the Metalanguage should be part
of the Keywords. However, Keywords can also include other important concepts or ideas
not stipulated in the Metalanguage. This section also helps in the review.

• Integration by parts • Integration by Trigonometric


Transformation
.

Big Picture

Week 8-9: Unit Learning Outcomes (ULO): At the end of the unit, you are expected to

a. practice additional integration techniques and apply the concepts


learned from the definite integral to solve for the limit of an improper
integrals.

Big Picture in Focus: ULOa. Practice additional integration


techniques and apply the concepts learned from the definite integral
to solve for the limit of an improper integrals.

Metalanguage
In this section, additional integration techniques will be discussed to achieve
ULOa. To do this, you need to know first the essential terms and symbols that you will
encounter in the following discussion for a quicker understanding of the topics to be
discussed.

Essential Knowledge

To perform the aforesaid big picture (unit learning outcomes) for the next two (2)
weeks of the course, you need to review the concepts and theorems of definite integral
as well as the concept of solving areas under the curve that is related to the topics that
will be laid down in the succeeding pages. Please note that you are not limited to
exclusively refer to these resources. Thus, you are expected to utilize other books and
other resources that are available in the university’s library e.g. ebrary,
search.proquest.com etc.
43
Trigonometric Substitutions

There are some problems that you cannot integrate directly however you can change the
variable of equations involving trigonometric functions called trigonometric substitution.
Below are the trigonometric substitutions that you can use for a given radical.

If the integrand involves: Substitute by:


𝑎2 − 𝑥 2 𝑥 = 𝑎 sin 𝜃
𝑎2 + 𝑥 2 𝑥 = a tan 𝜃
𝑥 2 − 𝑎2 𝑥 = 𝑎 sec 𝜃

Examples:
𝑑𝑥
1. Evaluate ∫ 𝑥 2 (𝑎2+𝑥 2 ) using trigonometric substitution.

Solution:

By trigonometric substitution: let 𝑥 = a tan 𝜃 ; 𝑑𝑥 = 𝑎 𝑠𝑒𝑐 2 𝜃 𝑑𝜃

𝑑𝑥 𝑎 𝑠𝑒𝑐 2 𝜃 𝑑𝜃
∫ 𝑥 2 (𝑎2 +𝑥 2) = ∫ (𝑎 tan 𝜃)2[𝑎2 +(𝑎 tan 𝜃)2 ]
𝑎 𝑠𝑒𝑐 2 𝜃 𝑑𝜃
= ∫ 𝑎2 𝑡𝑎𝑛2 𝜃[𝑎2+𝑎2 𝑡𝑎𝑛2 𝜃]
1 𝑠𝑒𝑐 2 𝜃 𝑑𝜃
= 𝑎 ∫ 𝑡𝑎𝑛2 𝜃 [𝑎2(1+𝑡𝑎𝑛2 𝜃)]
1 𝑠𝑒𝑐 2 𝜃 𝑑𝜃
= 𝑎3 ∫ 𝑡𝑎𝑛2 𝜃 𝑠𝑒𝑐 2𝜃
1
= 𝑎3 ∫ 𝑐𝑜𝑡 2 𝜃 𝑑𝜃
1
= 𝑎3 ∫(𝑐𝑠𝑐 2 𝜃 − 1) 𝑑𝜃
1 1
= − 𝑎3 cot 𝜃 − 𝑎3 𝜃 + 𝐶

𝑥 𝑜𝑝𝑝
From: 𝑥 = a tan 𝜃 where tan 𝜃 = =
𝑎 𝑎𝑑𝑗

x √𝑥 2 + 𝑎2 𝑎
cot 𝜃 = 𝑥
𝑥
𝜃 𝜃 = 𝑡𝑎𝑛−1 𝑎
a

1 𝑎 1 𝑥
= − 𝑎3 (𝑥 ) − 𝑎3 𝐴𝑟𝑐𝑡𝑎𝑛 +𝐶
𝑎
1 1 𝑥
= − 𝑎2 𝑥 − 𝑎3 𝐴𝑟𝑐𝑡𝑎𝑛 +𝐶
𝑎

44
𝑣 2 𝑑𝑣
2. Evaluate ∫ (𝑎2−𝑣2 )3/2.

Solution:

By trigonometric substitution: let 𝑣 = 𝑎 sin 𝜃 ; 𝑑𝑣 = 𝑎 cos 𝜃 𝑑𝜃


𝑣 2 𝑑𝑣 (a sin 𝜃)2 𝑎 cos 𝜃 𝑑𝜃
∫ (𝑎2−𝑣2 )3/2 = ∫ [𝑎2 −(𝑎 sin 𝜃)2 ]3/2
𝑎2 𝑠𝑖𝑛2 𝜃 𝑎 cos 𝜃 𝑑𝜃
=∫ [𝑎2 −𝑎2 𝑠𝑖𝑛2 𝜃]3/2
𝑎3 𝑠𝑖𝑛2 𝜃 cos 𝜃 𝑑𝜃
=∫ 𝑎3 [1−𝑠𝑖𝑛2 𝜃]3/2
𝑠𝑖𝑛2 𝜃 cos 𝜃 𝑑𝜃
=∫ 𝑐𝑜𝑠3 𝜃
𝑠𝑖𝑛2 𝜃 𝑑𝜃
=∫ 𝑐𝑜𝑠2 𝜃
(1−𝑐𝑜𝑠2 𝜃)𝑑𝜃
=∫ 𝑐𝑜𝑠2 𝜃
2
= ∫ 𝑠𝑒𝑐 𝜃 𝑑𝜃 − ∫ 𝑑𝜃
= tan 𝜃 − 𝜃 + 𝐶
𝑣 𝑜𝑝𝑝
From: 𝑣 = 𝑎 sin 𝜃 where sin 𝜃 = 𝑎 = ℎ𝑦𝑝

v a 𝑣
tan 𝜃 = √𝑎2
−𝑣 2
−1 𝑣
𝜃 𝜃= 𝑠𝑖𝑛 𝑎
√𝑎2 − 𝑣 2

𝑣 𝑣
= √𝑎2 − 𝐴𝑟𝑐𝑠𝑖𝑛 𝑎 + 𝐶
−𝑣 2

Integration of Rational Functions

In the evaluation of integrals, it is essential to have the skill and knowledge of doing the
reverse procedure. This procedure is called the partial fraction. The process is based in
this theorem that any proper rational fraction may be resolved into sum of partial fractions
subject to its classification.

𝑝𝑜𝑙𝑦𝑛𝑜𝑚𝑖𝑎𝑙 𝑖𝑛 𝑥 𝑓(𝑥)
Rational Fraction = 𝑝𝑜𝑙𝑦𝑛𝑜𝑚𝑖𝑎𝑙 𝑖𝑛 𝑥 = 𝑔(𝑥)

There are also two types of rational fraction. These are:


• Proper Fraction – where the degree of g(x) is greater than the degree of f(x)
• Improper Fraction – where the degree of f(x) is greater than the degree of g(x)
𝑥3
Example: 𝑥 2+2𝑥+1
45
Things we need to note in integrating:
1. To integrate an improper rational fraction, convert first to proper rational fraction
by division.
2. To integrate a proper rational fraction that cannot be integrated using the basic
integration formulas and convert to partial fractions.

There are four classifications under the Integration of Rational Functions. Namely:
• Distinct Linear Factor
• Repeated Linear Factor
• Distinct Quadratic Factor
• Repeated Quadratic Factor

Distinct Linear Factor

If the exponent of the numerator is less than exponent of the denominator can be broken
up into real linear factors (none of which is repeated). We then rewrite the given fractions
as a sum of fractions whose numerators are constants and whose respective
𝐴
denominators are the factors of the original denominator such as 𝑥+𝑏 , where A is a
constant number and is not equal to 0.

Examples:
(2𝑥+11)𝑑𝑥
1. ∫ 𝑥 2 +𝑥−6

Solution:
(2𝑥+11)𝑑𝑥 (2𝑥+11)𝑑𝑥
∫ 𝑥 2 +𝑥−6
= ∫ (𝑥+3)(𝑥−2)

2𝑥+11 𝐴 𝐵
= (𝑥+3) + 𝑥−2 multiplying both sides by (x+3)(x-2)
𝑥 2 +𝑥−6
2𝑥 + 11 = 𝐴(𝑥 − 2) + 𝐵(𝑥 + 3)

If x = 2: If x = -3:
2(2) + 11 = 𝐴(2 − 2) + 𝐵(2 + 3) 2(−3) + 11 = 𝐴(−3 − 2) + 𝐵(−3 + 3)
15 = 5𝐵 5 = −5𝐴
B=3 A = -1
−1 3
2𝑥 + 11 = (𝑥+3) + (𝑥−2)
(2𝑥+11)𝑑𝑥 𝑑𝑥 𝑑𝑥
∫ = − ∫ 𝑥+3 + 3 ∫ 𝑥−2
𝑥 2 +𝑥−6
= − ln(𝑥 + 3) + 3 ln(𝑥 − 2) + 𝐶
(𝑥−2)3
= ln +𝐶
𝑥+3

46
5 sin 𝜃 cos 𝜃 𝑑𝜃
2. ∫ 𝑠𝑖𝑛2 𝜃+3 sin 𝜃−4

Solution:

5 sin 𝜃 cos 𝜃 𝑑𝜃 5 sin 𝜃 cos 𝜃 𝑑𝜃


∫ 𝑠𝑖𝑛2 𝜃+3 sin 𝜃−4 = ∫ (sin 𝜃−1)(sin 𝜃+4)
5 sin 𝜃 cos 𝜃 𝐴 𝐵
= sin 𝜃−1 + sin 𝜃+4 multiplying both sides by (sin 𝜃 − 1)(sin 𝜃 + 4)
(sin 𝜃−1)(sin 𝜃+4)
5 sin 𝜃 cos 𝜃 = 𝐴(sin 𝜃 + 4) + 𝐵(sin 𝜃 − 1)

If sin 𝜃 = 1: If sin 𝜃 = −4
5(1) cos 𝜃 = 𝐴(1 + 4) + 𝐵(1 − 1) 5(−4) cos 𝜃 = 𝐴(−4 + 4) + 𝐵(−4 − 1)
5 cos 𝜃 = 5𝐴 −20 cos 𝜃 = −5𝐵
𝐴 = cos 𝜃 𝐵 = 4 cos 𝜃

5 sin 𝜃 cos 𝜃 cos 𝜃 4 cos 𝜃


= sin 𝜃−1 + sin 𝜃+4
(sin 𝜃−1)(sin 𝜃+4)
5 sin 𝜃 cos 𝜃 𝑑𝜃 cos 𝜃 𝑑𝜃 cos 𝜃 𝑑𝜃
∫ 𝑠𝑖𝑛2 𝜃+3 sin 𝜃−4 = ∫ sin 𝜃−1 + 4 ∫ sin 𝜃+4
= ln(sin 𝜃 − 1) + 4 ln(sin + 4) + 𝐶

Repeated Linear Factor

If a linear factor ax+b occurs n times as a factor of the denominator, there corresponds
𝐴1 𝐴2 𝐴𝑛
to this n partial fractions 𝑎𝑥+𝑏 + (𝑎𝑥+𝑏)2 + ⋯ + (𝑎𝑥+𝑏)𝑛 , where 𝐴1, 𝐴2 … 𝐴𝑛 is constant and

not equal to 0.

Examples:
𝑑𝑥
1. ∫ 𝑥(𝑥+2)2

Solution:
𝑑𝑥 𝐴 𝐵 𝐶
∫ 𝑥(𝑥+2)2 = 𝑥 + 𝑥+2 + (𝑥+2)2

1 𝐴 𝐵 𝐶
= 𝑥 + 𝑥+2 + (𝑥+2)2 multiplying both sides by 𝑥(𝑥 + 2)2
𝑥(𝑥+2)2
1 = 𝐴(𝑥 + 2)2 + 𝐵[𝑥(𝑥 + 2)] + 𝐶𝑥

If x = 0: If x = -2 By coefficient of x:
1 = 𝐴(2)2 1 = −2𝐶 1 = 𝐴(𝑥 2 + 4𝑥 + 4) + 𝐵𝑥 2 + 2𝐵𝑥 + 𝐶𝑥
1 1
𝐴= 4 𝐶 = −2 0 = 4𝐴 + 2𝐵 + 𝐶
1 1
4𝐴+𝐶 4( )− 1
4 2
𝐵= = =−
2 −2 4

47
1 1 1
1 4 4 2
= − −
𝑥(𝑥+2)2 𝑥 𝑥+2 (𝑥+2)2
𝑑𝑥 1 𝑑𝑥 1 𝑑𝑥 1 𝑑𝑥
∫ 𝑥(𝑥+2)2 = 4 ∫ − 4 ∫ 𝑥+2 − 2 ∫ (𝑥+2)2
𝑥
1 1 1
= 4 ln 𝑥 − 4 ln(𝑥 + 2) − 2 ∫(𝑥 + 2)−2 𝑑𝑥
1 1 1
= 4 ln 𝑥 − 4 ln(𝑥 + 2) + 2(𝑥+2) + 𝐶

(5𝑦−4)𝑑𝑦
2. ∫ 𝑦 3 +4𝑦 2

Solution:
(5𝑦−4)𝑑𝑦 (5𝑦−4)𝑑𝑦
∫ 𝑦 3 +4𝑦 2
=∫ 𝑦 2 (𝑦+4)

(5𝑦−4) 𝐴 𝐵 𝐶
= 𝑦 + 𝑦 2 + 𝑦+4 multiplying both sides by 𝑦 2 (𝑦 + 4)
𝑦 2 (𝑦+4)
5𝑦 − 4 = 𝐴𝑦(𝑦 + 4) + 𝐵(𝑦 + 4) + 𝐶𝑦 2
5𝑦 − 4 = 𝐴𝑦 2 + 4𝐴𝑦 + 𝐵𝑦 + 4𝐵 + 𝐶𝑦 2

Finding for the value of the constants A, B and C using coefficients:

By coefficient of 𝑦 0 (constants): By coefficient of 𝑦1 or 𝑦: By coefficient of 𝑦 2 :


−4 = 4𝐵 5 = 4𝐴 + 𝐵 0=𝐴+𝐶
3
𝐵 = −1 5 = 4𝐴 − 1 0= 2+𝐶
3
6 = 4𝐴 𝐶 = −2
3
𝐴= 2

3 3
(5𝑦−4)𝑑𝑦 𝑑𝑦 𝑑𝑦 𝑑𝑦
2 2
∫ =∫ − ∫ 𝑦 2 − ∫ 𝑦+4
𝑦 3 +4𝑦 2 𝑦
3 𝑑𝑦 𝑑𝑦 3 𝑑𝑦
= 2∫ − ∫ 𝑦 2 − 2 ∫ 𝑦+4
𝑦
3 1 3
= 2 ln 𝑦 + 𝑦 − 2 ln(𝑦 + 4) + 𝐶

Distinct Quadratic Factor

If the factors in the denominator of the form 𝑎𝑥 2 + 𝑏𝑥 + 𝑐 in which 𝑏 2 − 4𝑎𝑐 < 0, assume
the partial fraction:

𝐴(2𝑎𝑥+𝑏)+𝐵
where A & B are to be determined
𝑎𝑥 2 +𝑏𝑥+𝑐

48
Examples:
𝑥𝑑𝑥
1. ∫ 𝑥 2 +6𝑥+13

Solution:

𝑥 𝐴(2𝑥+6)+𝐵
= multiply both sides by 𝑥 2 + 6𝑥 + 13
𝑥 2 +6𝑥+13 𝑥 2 +6𝑥+13
𝑥 = 𝐴(2𝑥 + 6) + 𝐵
𝑥 = 2𝐴𝑥 + 6𝐴 + 𝐵

Finding for the value of the constants A and B using coefficients:

By coefficient of 𝑥1 or 𝑥: By coefficient of 𝑥 0 (constants):


1 = 2𝐴 0 = 6𝐴 + 𝐵
1 1
𝐴=2 0 = 6 (2) + 𝐵
𝐵 = −3
𝑥 2𝑥+6 3
= 2(𝑥 2+6𝑥+13) − 𝑥 2 +6𝑥+13
𝑥 2 +6𝑥+13
𝑥𝑑𝑥 1 (2𝑥+6)𝑑𝑥 𝑑𝑥
∫ 𝑥 2 +6𝑥+13 = 2 ∫ 𝑥 2 +6𝑥+13 − 3 ∫ 𝑥 2 +6𝑥+13
1 𝑑𝑥 solve the denominator of the last term
= 2 ln(𝑥 2 + 6𝑥 + 13) − 3 ∫ (𝑥+3)2 +4 by completing the square method
1 3 𝑥+3
= 2 ln(𝑥 2 + 6𝑥 + 13) − 2 𝐴𝑟𝑐𝑡𝑎𝑛 +𝐶
2

4𝑑𝑥
2. ∫ 𝑥 3 −4𝑥 2 +8𝑥

Solution:

4 𝐴 𝐵(2𝑥−4)+𝐶
=𝑥+ multiply both sides by 𝑥(𝑥 2 − 4𝑥 + 8)
𝑥(𝑥 2 −4𝑥+8) 𝑥 2 −4𝑥+8
2
4 = 𝐴(𝑥 − 4𝑥 + 8) + 𝐵𝑥(2𝑥 − 4) + 𝐶𝑥
4 = 𝐴𝑥 2 − 4𝐴𝑥 + 8𝐴 + 2𝐵𝑥 2 − 4𝐵𝑥 + 𝐶𝑥

Finding for the value of the constants A,B and C using coefficients:

By coefficient of 𝑥 0 (constants): By coefficient of 𝑥 2 : By coefficient of 𝑥1 or 𝑥:


4 = 8𝐴 0 = 𝐴 + 2𝐵 0 = −4𝐴 − 4𝐵 + 𝐶
1 1 1 1
𝐴= 2 0 = 2 + 2𝐵 0 = −4 (− 2) − 4 (− 4) + 𝐶
1
𝐵 = −4 0 = −2 + 1 + 𝐶
𝐶=1
1 1
4 (2𝑥−4) 1
2 4
= − + 𝑥 2 −4𝑥+8
𝑥(𝑥 2 −4𝑥+8) 𝑥 𝑥 2 −4𝑥+8
49
4𝑑𝑥 1 𝑑𝑥 1 (2𝑥−4)𝑑𝑥 𝑑𝑥
∫ 𝑥 3 −4𝑥 2+8𝑥 = 2 ∫ − 4 ∫ 𝑥 2 −4𝑥+8 + ∫ 𝑥 2 −4𝑥+8
𝑥
1 1 𝑑𝑥
= 2 ln 𝑥 − 4 ln(𝑥 2 − 4𝑥 + 8) + ∫ (𝑥−2)2 +4
1 1 1 𝑥−2
= 2 ln 𝑥 − 4 ln(𝑥 2 − 4𝑥 + 8) + 2 𝐴𝑟𝑐𝑡𝑎𝑛 +𝐶
2

Repeated Quadratic Factor

If a quadratic factor 𝑎𝑥 2 + 𝑏𝑥 + 𝑐 occurs n times as a factor of the denominator, a


corresponding factor to this form is:

𝐴(2𝑎𝑥+𝑏)+𝐵 𝐶(2𝑎𝑥+𝑏)+𝐷 𝐾(2𝑎𝑥+𝑏)+𝐿


+ + ⋯ + (𝑎𝑥 2 +𝑏𝑥+𝑐)𝑛 where A, B, C, D … K, L are to be determined
𝑎𝑥 2 +𝑏𝑥+𝑐 (𝑎𝑥 2 +𝑏𝑥+𝑐)2

Examples:
𝐴𝑟𝑐𝑡𝑎𝑛 𝑦 𝑑𝑦
1. ∫ 𝑦3

Solution:
𝐴𝑟𝑐𝑡𝑎𝑛 𝑦 𝐴 𝐵 𝐶
= 𝑦 + 𝑦2 + 𝑦3
𝑦3

𝑑𝑦
Let u = 𝐴𝑟𝑐𝑡𝑎𝑛 𝑦; du = 𝑦 2+1
dv = 𝑦 −3 𝑑𝑦; v = ∫ 𝑦 −3 𝑑𝑦
𝑦 −2
v=− 2
1
v = − 2𝑦 2 + 𝐶

𝐴𝑟𝑐𝑡𝑎𝑛 𝑦 𝑑𝑦 1 1 𝑑𝑦
∫ = − 2𝑦 2 𝐴𝑟𝑐𝑡𝑎𝑛 𝑦 + 2 ∫ 𝑦 2(𝑦 2+1) using integration by-parts and partial fraction
𝑦3
1
= − 2𝑦 2 𝐴𝑟𝑐𝑡𝑎𝑛 𝑦 + 𝑝𝑎𝑟𝑡𝑖𝑎𝑙 𝑓𝑟𝑎𝑐𝑡𝑖𝑜𝑛 𝑜𝑓 𝑡ℎ𝑒 2𝑛𝑑 𝑡𝑒𝑟𝑚

1 𝐴 𝐵 𝐶(2𝑦)+𝐷
= 𝑦 + 𝑦2 + multiply both sides by 𝑦 2 (𝑦 2 + 1)
𝑦 2 (𝑦 2 +1) 𝑦 2 +1
1 = 𝐴𝑦(𝑦 2 + 1) + 𝐵(𝑦 2 + 1) + 𝐶𝑦 2 (2𝑦) + 𝐷𝑦 2
1 = a𝐴𝑦 3 + 𝐴𝑦 + 𝐵𝑦 2 + 𝐵 + 2𝐶𝑦 3 + 𝐷𝑦 2

Finding for the value of the constants A,B,C and D using coefficients:

By coefficient of 𝑦 0 : By coefficient of 𝑦 2 : By coefficient of 𝑦: By coefficient of 𝑦 3 :


𝐵=1 0=𝐵+𝐷 𝐴=0 0 = 𝐴 + 2𝐶
0=1+𝐷 0 = 0 + 2𝐶
𝐷 = −1 𝐶=0

50
𝐴𝑟𝑐𝑡𝑎𝑛 𝑦 𝑑𝑦 1 1 𝑑𝑦 𝑑𝑦
∫ = − 2𝑦 2 𝐴𝑟𝑐𝑡𝑎𝑛 𝑦 + 2 [∫ 𝑦 2 − ∫ 𝑦 2+1]
𝑦3
1 1
= − 2𝑦 2 𝐴𝑟𝑐𝑡𝑎𝑛 𝑦 − 2𝑦 − 𝐴𝑟𝑐𝑡𝑎𝑛 𝑦 + 𝐶
1 1
= − 2𝑦 − 𝐴𝑟𝑐𝑡𝑎𝑛 (2𝑦 2 + 1) + 𝐶

𝑑𝑥
2. ∫ (𝑥 2 +2𝑥+10)2

Solution:

1 𝐴(2𝑥+2)+𝐵 𝐶(2𝑥+2)+𝐷
= +
(𝑥 2 +2𝑥+10)2 𝑥 2 +2𝑥+10 (𝑥 2 +2𝑥+10)2
1 = 𝐴(2𝑥 + 2)(𝑥 + 2𝑥 + 10) + 𝐵(𝑥 2 + 2𝑥 + 10) + 𝐶(2𝑥 + 2) + 𝐷
2

1 = 𝐴[2𝑥 3 + 4𝑥 2 + 20𝑥 + 2𝑥 2 + 4𝑥 + 20] + 𝐵𝑥 2 + 2𝐵𝑥 + 10𝐵 + 2𝐶𝑥 + 2𝐶 + 𝐷


1 = 2𝐴𝑥 3 + 6𝐴𝑥 2 + 24𝐴𝑥 + 20𝐴 + 𝐵𝑥 2 + 2𝐵𝑥 + 10𝐵 + 2𝐶𝑥 + 2𝐶 + 𝐷

Finding for the value of the constants A,B,C and D using coefficients:

By coefficient of 𝑥 3 : By coefficient of 𝑥 2 : By coefficient of 𝑥: By coefficient of 𝑥 0 :


0 = 2𝐴 0 = 6𝐴 + 𝐵 0 = 24𝐴 + 2𝐵 + 2𝐶 1 = 20𝐴 + 10𝐵 + 2𝐶 + 𝐷
𝐴=0 𝐵=0 𝐶=0 𝐷=1

𝑑𝑥 𝑑𝑥
∫ (𝑥 2+2𝑥+10)2 = ∫ (𝑥 2 +2𝑥+10)2
𝑑𝑥
= ∫ [(𝑥+1)2+9]2

using trigonometric substitution


let 𝑥 − 1 = 3 tan 𝜃; dx = 3 𝑠𝑒𝑐 2 𝜃 𝑑𝜃

3 𝑠𝑒𝑐 2 𝜃 𝑑𝜃
= ∫ [3 tan 𝜃2+9]2
𝑠𝑒𝑐 2 𝜃 𝑑𝜃
= 3 ∫ [9 𝑡𝑎𝑛2 𝜃+9]2
3 𝑠𝑒𝑐 2 𝜃 𝑑𝜃
= 81 ∫ (𝑠𝑒𝑐 2𝜃)2
1 𝑑𝜃
= 27 ∫ 𝑠𝑒𝑐 2𝜃
1
= 27 ∫ 𝑐𝑜𝑠 2 𝜃 𝑑𝜃
1 1
= 27 ∫ 2 (1 + cos 2𝜃)𝑑𝜃
1 1
= 54 ∫ 𝑑𝜃 + 54 ∫ cos 2𝜃 𝑑𝜃
1 1
= 54 𝜃 + 108 sin 2𝜃 + 𝐶

51
From 𝑥 + 1 = 3 tan 𝜃

𝑥+1
x+1 √(𝑥 + 1)2 + 32 tan 𝜃 = 3

𝜃
3
1 1
= 54 𝜃 + 108 (2 sin 𝜃 cos 𝜃) + 𝐶
1 1
= 54 𝜃 + 54 sin 𝜃 cos 𝜃 + 𝐶
1 𝑥+1 1 𝑥+1 3
= 54 𝐴𝑟𝑐𝑡𝑎𝑛 + 54 [√𝑥2 ] [√𝑥 2 ]+𝐶
3 +2𝑥+10 +2𝑥+10
1 𝑥+1 1 𝑥+1
= 54 𝐴𝑟𝑐𝑡𝑎𝑛 + 18 (𝑥 2+2𝑥+10) + 𝐶
3

Rationalizing Substitution

From the lessons above, we have discussed a few techniques in integrating functions.
Now, we will learn a few substitutions that will allow us to convert or transform some types
of integrals that involves rational functions.

Examples:

𝑥𝑑𝑥
1. Evaluate the integral ∫ .
√𝑥+2

Solution:

Let 𝑢 = √𝑥 + 2; 𝑢2 = 𝑥 + 2; 𝑥 = 𝑢2 − 2; 𝑑𝑥 = 2𝑢𝑑𝑢

𝑥𝑑𝑥 (𝑢2 −2)2𝑢 𝑑𝑢


∫ √𝑥+2 = ∫ 𝑢
2
= ∫(2𝑢 − 4)𝑑𝑢
= 2 ∫ 𝑢2 𝑑𝑢 − 4 ∫ 𝑑𝑢
2𝑢3
= − 4𝑢 + 𝐶
3
2(√𝑥+2)3
= − 4(√𝑥 + 2) + 𝐶 substituting the original value of u to the resulting equation
3
2
2(√𝑥+2)
= √𝑥 + 2 ( − 4) + 𝐶 simplify and factor common terms
3
2(𝑥+2)
= √𝑥 + 2 ( 3
− 4) + 𝐶
2(𝑥+2)−12
= √𝑥 + 2 ( )+𝐶
3
2𝑥−8
= √𝑥 + 2 ( )+𝐶
3
2(𝑥−4)
= √𝑥 + 2 ( )+𝐶
3
52
𝑑𝑥
2. ∫ 3
1+ √𝑥+4

Solution:

3
Let 𝑢 = √𝑥 + 4; 𝑢3 = 𝑥 + 4; 𝑥 = 𝑢3 − 4; 𝑑𝑥 = 3𝑢2 𝑑𝑢

𝑑𝑥 3𝑢2 𝑑𝑢
∫ 1+ 3 𝑥+4 = ∫ 1+𝑢

𝑢2 𝑑𝑢
= 3∫ solve the quotient of the polynomials by long division
1+𝑢
1
= 3 ∫(𝑢 − 1 + 𝑢+1)𝑑𝑢
𝑑𝑢
= 3 ∫ 𝑢𝑑𝑢 − 3 ∫ 𝑑𝑢 + 3 ∫ 𝑢+1
3𝑢2
= − 3𝑢 + 3 ln(𝑢 + 1) + 𝐶
2
3
3( √𝑥+4)2 3 3
= − 3( √𝑥 + 4) + 3 ln( √𝑥 + 4 + 1) + 𝐶 substituting the original value of u
2

Improper Integrals

𝑏
In the previous discussion, we know that a definite integral ∫𝑎 𝑓(𝑥)𝑑𝑥 requires that the
interval [a, b] be finite. However, in improper integral, it deals with the procedure for
evaluating integrals that do not satisfy the requirements of a definite integral usually
because either one or both of the limits of integration are infinite or because 𝑓 has a finite
number of infinite discontinuities in the interval [a, b].

A function 𝑓 is said to have an infinite discontinuity at c when, from the right or left,
lim 𝑓(𝑥) = ∞ or lim 𝑓(𝑥) = −∞.
𝑥→𝑐 𝑥→𝑐

Definition of Improper Integrals with Infinite Integration Limits


1. If 𝑓 is continuous on the interval [a, ∞], then
∞ ℎ
∫ 𝑓(𝑥)𝑑𝑥 = lim ∫ 𝑓(𝑥)𝑑𝑥
𝑎 ℎ→∞ 𝑎

2. If 𝑓 is continuous on the interval (-∞, b), then


𝑏 𝑏
∫ 𝑓(𝑥)𝑑𝑥 = lim ∫ 𝑓(𝑥)𝑑𝑥
−∞ ℎ→∞ ℎ

3. If 𝑓 is continuous on the interval (-∞, ∞), then


∞ 𝑐 ℎ
∫ 𝑓(𝑥)𝑑𝑥 = lim ∫ 𝑓(𝑥)𝑑𝑥 + lim ∫ 𝑓(𝑥)𝑑𝑥
−∞ ℎ→−∞ ℎ ℎ→∞ 𝑐

53
where c is any real number.

If the limit exists, the improper integral is said to be convergent and if the limit fails to
exist, the improper integral is divergent.

Example 1:

𝑏 𝑑𝑥
Evaluate ∫1 .
𝑥

Solution:

𝑏 𝑑𝑥 𝑏 𝑑𝑥
∫1 = lim ∫1 take limit as 𝑏 → ∞
𝑥 𝑏→∞ 𝑥
= lim [ln 𝑥] 𝑏1
𝑏→∞
= lim [ln 𝑏 − 0]
𝑏→∞
=∞

Since the limit does not exist, therefore, you can conclude that the improper integral
diverges.

Example 2:


Evaluate ∫0 𝑒 −𝑥 𝑑𝑥.

Solution:

∞ 𝑏
∫0 𝑒 −𝑥 𝑑𝑥 = lim ∫0 𝑒 −𝑥 𝑑𝑥
𝑏→∞
= lim [−𝑒 −𝑥 ] 𝑏0
𝑏→∞
= lim (−𝑒 −𝑏 + 1)
𝑏→∞
=1

Therefore, you can conclude that the improper integral is convergent.

Self-Help: You can also refer to the sources below to help you further
understand the lesson.

* Stewart, J. (2000). Calculus: Concepts and Contexts. Australia: Cengage.

54
* Downing, Douglas (2009). Mathematics Terms 3rd Edition. New York: Barron’s
Educational Series Inc.
* Feliciano and Uy, F. (1983). Differential and Integral Calculus. Manila: Merriam and
Webster Bookstore, Inc.
* Peterson, Thurman. (1964). Calculus with Analytic Geometry. Philippines: Harper and
Row Publishers, Incorporated.

Let’s Check

Activity 1. Evaluate the following definite integrals by using the integration techniques
that you’ve learned from the lesson above.

𝑦 2 𝑑𝑦 2 √9−𝑥2
1. ∫ (𝑦 2+16)3/2 4. ∫1 𝑑𝑥
𝑥4

(𝑥 2 −16)3/2 𝑑𝑥
2. ∫ 𝑑𝑥 5. ∫ 3
𝑥3 (𝑥 2 +4)4

3. ∫ 𝑥 3 (𝑎2 + 𝑥 2 )1/2 𝑑𝑥 4
6. ∫ 𝑥 4 −1 𝑑𝑥

Activity 2. Solve for the improper integrals below and determine whether it diverges or
converges.
4 𝑑𝑥 2 𝑑𝑥
7. ∫0 9. ∫0
√𝑥 (𝑥−1)2
4 𝑑𝑥 0
8. ∫3 (𝑥−3)3/2
10. ∫−∞ 𝑒 3𝑥 𝑑𝑥

Let’s Analyze

Activity 1. Evaluate the following integrals. Show your solutions neatly and legibly.

√2 𝑑𝑥 4𝑥 2 +3𝑥+2
1. ∫0 6. ∫ 𝑑𝑥
𝑥 2 √4−𝑥 2 𝑥 2 (𝑥+1)

7𝑥 2 +15𝑥+22 3𝑥 3 +6𝑥 2 +17𝑥−5


2. ∫ (𝑥−3)(𝑥 2 +4𝑥+5) 𝑑𝑥 7. ∫ 𝑑𝑥
𝑥 2 +2𝑥+5

√𝑥 𝑑𝑥
3. ∫ 1+𝑥 𝑑𝑥 8. ∫ 𝑥 4 +16𝑥 2

√𝑡 𝑥 3/2
4. ∫ 4 𝑑𝑡
9. ∫ 3 𝑑𝑥
2
4 + √𝑡 √𝑥+1

6𝑥+8 1
5. ∫ (𝑥+3)(𝑥+5)(𝑥−2) 𝑑𝑥 10. ∫ 3 𝑑𝑥
√𝑥+ √𝑥

55
In a Nutshell

The importance of learning different techniques in integration is very important in helping


you solve complicated problems. In this portion of the unit, you will be required to state
your arguments or synthesis relevant to the topics presented. I will supply the first two
items and you will continue the rest.

3. The integration by parts is the simplest technique to use.


4. Not all techniques can make the integration process easy.

Your Turn:

3. __________________________________________________________________________

4. __________________________________________________________________________

5. __________________________________________________________________________

Q&A List

Questions/Issues Answer
1.
2.
3.
4.
5.

Keywords Index

This section lists down the keywords that help students for recall. Keywords are similar
to Metalanguage but without the definitions and descriptions. This section includes
concepts (a word or phrase), ideas, theories, names of people, and other vital terms to
remember. Technically speaking, all those included in the Metalanguage should be part
of the Keywords. However, Keywords can also include other important concepts or ideas
not stipulated in the Metalanguage. This section also helps in the review.

• Integration by Trigonometric • Integration by Rational Functions


Substitution • Improper Integrals
• Partial Fraction
• Rational Function

56
Big Picture

Week 10-12: Unit Learning Outcomes (ULO): At the end of the unit, you are expected
to

a. Determine the plane area, area between curves and volumes of solids
of revolution through integration,

Big Picture in Focus: ULOa. Determine the plane area, area between
curves and volumes of solids of revolution through integration.

Metalanguage

In this section, we will now be applying all the techniques that you have learned
from the past lesson. Here you are expected to have learned all the techniques in
integration to solve the following problems in this lesson. Please refer to these definitions
(terms/symbols) in case you will encounter difficulty in understanding educational
concepts.

Plane Area - A plane is a 2-dimensional space. A plane region is, well, a region on a
plane, as opposed to, for example, a region in a 3-dimensional space. We'll calculate the
area A of a plane region bounded by the curve that's the graph of a function f continuous
on [a, b] where a < b, the x-axis, and the vertical lines x = a and x = b.

Area Under A Curve - We will be computing the area A of the region bounded by 2
curves that are the graphs of the functions f and g and the vertical lines x = a and x = b,
where a < b and f and g are continuous on [a, b].

Polar Area - To understand the area inside of a polar curve r = f(θ), we start with the area
of a slice of pie. If the slice has angle θ and radius r, then it is a fraction θ/2π of the entire
pie.

Solids of Revolution - To get a solid of revolution we start out with a function, y = f(x) on
an interval [a, b]. We then rotate this curve about a given axis to get the surface of the
solid of revolution. For purposes of this discussion let us rotate the curve about the x-axis,
although it could be any vertical or horizontal axis. Doing this for the curve above gives
the following three-dimensional region.

Essential Knowledge
To perform the aforesaid big picture (unit learning outcomes) for the next two (2)
weeks of the course, you need to review the concepts and theorems of definite integral
that will be laid down in the succeeding pages. Please note that you are not limited
57
to exclusively refer to these resources. Thus, you are expected to utilize other books
and other resources that are available in the university’s library e.g. ebrary,
search.proquest.com etc.

1.1 Plane Area

Figure 1

We will be attempting to get the area of the region S that is under the curve y = f (x) from
a to b. This means that S, illustrated in F1, is bounded by the graph of a continuous
function f [ where f (x) ≥ 0 ], the vertical lines x = a and x=b, and the x axis. In solving this
problem, we will be going to approximate the region S by rectangles and then we take the
limit of the areas of the rectangles as we increase the number of rectangles.

Examples 1:

Estimate the area under the parabola 𝑦 = 𝑥 2 from 0 to 1.

Solution:

We can say that the area of S must be somewhere between 0 to 1 because S is


contained in a square with side length 1. Suppose we divide S in to four strips which will
result in figure 3.

58
1 1 1 3
Each rectangle has a width 4 and the heights are (4)2, (2)2, (4)2 and (1)2. Let R4 be the sum
of the areas of these rectangles
𝑅4 = 0. 46875

We see that the A of S is less than R4, so A < 0. 46875. If we use the smaller rectangles
in figure 4. The sum of the areas of these approximating rectangles is

𝐿4 = 0. 21875

We can now have lower and upper estimates for A:

0.21875 < A > 0.46875

As we repeat the procedure with a large number of strips, we could obtain better estimates
and will show it in the table. A good estimate of A ≈ 0.3333335.

1.2 Areas between Two Curves

The area A of the region bounded by the curves y = f(x), y = g (x) , and the lines x = a, x
= b, where f and g are continuous and f (x) ≥ g (x) for all x [ a, b], is
𝑏
𝐴 = ∫ [ 𝑓 (𝑥) − 𝑔 (𝑥)]𝑑𝑥
𝑎

59
Example 1:

Find the area of the region bounded above by 𝑦 = 𝑒 𝑥 , bounded by 𝑦 = 𝑥, and bounded
on the sides by x= 0 and x = 1.

Solution:

Example 2:

Find the area of the region enclose by the parabola y = x2 and y = 2x - x2 .

Solution:

Solving the equations simultaneoulsy will find the points of intersection of the parabolas
at ( 0, 0) and ( 1, 1). With the figure we see that the top and bottom boundaries are : yT =
2x – x2 and yB = x2. The area of rectangle is :

Example 3:

Find the approximate area of the region bounded by the curves 𝑦 = 𝑥/√𝑥 2 + 1 and 𝑦 =
𝑥 4 − 𝑥.

60
Solution:

1.3 Polar Areas

To solve the area of the shaded part, the formula below is to be used.
1 𝛽
1. A= 2 ∫𝛼 𝑟 2 𝑑𝜃

1 𝛽
2. A= 2 ∫𝛼 (𝑟0 2 − 𝑟𝑖 2 )𝑑𝜃

61
Example:

1. Determine the area of the inner loop of r = 2 + 4cosθ.

Solution:

First, we must determine if there is a symmetry. For this case, the curve is symmetrical
with respect to the x axis. It is wise to plot the curve from 0⁰ to 180⁰.

2𝜋 4𝜋
cos ɵ= -1/2, ɵ= 120⁰ = , 240 ⁰ =
3 3

ɵ 0 30 45 60 90 120 150 180


r 6 5.46 4.83 4 2 0 1.46 -2

4𝜋
1
A =∫2𝜋3 2 (2 + 4𝑐𝑜𝑠𝜃)2 𝑑𝜃
3
4𝜋
3 1
=∫ 2𝜋 (4 + 16𝑐𝑜𝑠𝜃 + 16𝑐𝑜𝑠 2 𝜃) 𝑑𝜃
2
3
4𝜋
3 1
=∫ 2𝜋 (4 + 16𝑐𝑜𝑠𝜃 + 16𝑐𝑜𝑠 2 𝜃) 𝑑𝜃
2
3
4𝜋
3
= ∫ (2 + 8𝑐𝑜𝑠𝜃 + 4(1 + 𝑐𝑜𝑠2𝜃)) 𝑑𝜃
2𝜋
3
4𝜋
3
= ∫ (6 + 8𝑐𝑜𝑠𝜃 + 4𝑐𝑜𝑠2 𝜃)𝑑𝜃
2𝜋
3
4𝜋
3
= 6𝜃 + 8𝑠𝑖𝑛𝜃 + 2𝑠𝑖𝑛2𝜃 𝑙 2𝜋
3
= 4π - 6√3 = 2.174 sq. units

2. Determine the area that lies inside r= 3+ 2sin𝜃 and outside r=2.
With this curve , symmtery with respect to the y axis is being estabished.

𝜃 0 30⁰ 45⁰ 60⁰ 90⁰ 270⁰ 300⁰ 330⁰


R 0 4 4.41 4.73 5 1 1.27 2

To determine where they interect equate the 2 equations.

62
2= 3+2sin 𝜃
7𝜋 11𝜋
sin 𝜃 = -1/2 ,𝜃= = 210⁰ , 𝜃 = = 330⁰
6 6

7𝜋
1
A = ∫−𝜋6 2 ((3 + 2𝑠𝑖𝑛ɵ)2 − 22 ) 𝑑
6
7𝜋
6 1
=∫ −𝜋 (9 + 12𝑠𝑖𝑛ɵ + 4𝑠𝑖𝑛2 ɵ − 22 ) 𝑑𝜃
2
6
7𝜋
6 1
=∫ −𝜋 (5 + 12𝑠𝑖𝑛ɵ + 4𝑠𝑖𝑛2 ɵ) 𝑑𝜃
2
6
7𝜋
6 1
=∫ −𝜋 (7 + 12𝑠𝑖𝑛ɵ − 2𝑐𝑜𝑠 2ɵ) 𝑑𝜃
2
6
7𝜋
1
= (7ɵ − 12𝑐𝑜𝑠ɵ − 𝑠𝑖𝑛2ɵ)𝑙−𝜋
6
2 6
11√3 14𝜋
= +
2 3
= 24.187 units

2 Volumes

2.1 Cylindrical disk method

Let S be a solid that lies between 𝑥 = 𝑎 and 𝑥 = 𝑏. If the corss-sectional are of S in the
plane Px through x and perpendicular to the x-axis, is A(x), where A is a continous
function, the the volume of S is

Example 1:

Show that the volume of a sphere of radius r is


63
4 3
𝑉= 𝜋𝑟
3

Solution:

The plane Px intersects the sphere in a circle whose radius is 𝑦 = √𝑟 2 + 𝑏 2 and the cross-
sectional area is . With a= -r and b = r, we have.

Example 2:

Find the volume of the solid obtained by rotating the region bounded by 𝑦 = 𝑥 3 , 𝑦 = 8,
and 𝑥 = 0 about the y-axis.

Solution:

64
The area of the cross section through y is and the volume of the
approximating cylinder is , Since the solid lies between 𝑦 = 0 and 𝑦 = 8,
its volume is

Example 3:

The region bounded R enclosed by the curves 𝑦 = 𝑥 and 𝑦 = 𝑥 3 is rotated about the x-
axis. Find the volume of the resulting solid.

Solution:

The cross-sectional area is , and the volume is

The solid in example are called solids of revolution because they are obtained by
revolving a region about a line. In general, solid of revolution by using the basic defining
formula

And we find the cross-sectional area A (x) or A(y) in one of the following ways:
• If the cross-sectional area is a disk, we find the radius of the disk and use
𝐴 = 𝜋(𝑟𝑎𝑑𝑖𝑢𝑠)2
• If the cross-section is a washer, we find the inner radius rin and outer radius rout
and compute the area of the washer by subtracting the inner disk from the outer
disk.

65
2.2 Cylindrical Shell Method

Example:

Find the volume of the solid obtained by rotating about the 𝑦 − 𝑎𝑥𝑖𝑠 the region bounded
by the curve 𝑦 = 2𝑥 2 − 𝑥 3 and the 𝑥 − 𝑎𝑥𝑖𝑠.

Solution:

Instead of slicing, we approximate the solid using cylindrical shells. In the figure a typical
approximating rectangle with width ∆x. If we rotate this rectangle about the 𝑦 = 𝑎𝑥𝑖𝑠, we
get a cylindrical shell whose average radius is xi, the midpoint of the ith subinterval.

66
2. 3 Washer Method

If the region we revolve to generate a solid does not border on or cross the axis of
revolution, the solid has a hole in it like in the figure. The cross-section perpendicular to

the axis of revolution are washers instead of disks. The dimensions of a typical washer
are

Outer radius ∶ 𝑅(𝑥)


Inner radius ∶ 𝑟(𝑥)

The washer’s area is

Consequently, the definition of volume in this case gives

Volume by Washers for Rotation About the x-axis

This method for calculating the volume of a solid of revolution is called the washer
method because a thin slab of the solid resembles a circular washer of radius R(x) and
inner radius r(x).

Example 1:

The region bounded by the curve 𝑦 = 𝑥 2 + 1 and the line 𝑦 = −𝑥 + 3 is revolved about
the 𝑥 − 𝑎𝑥𝑖𝑠 to generate a solid. Find the volume of the solid.

67
Outer radius is : 𝑅(𝑥) = −𝑥 + 3
Inner radius: 𝑟(𝑥) = 𝑥 2 + 1
The limits if integration is

The volume integral is,

Example 2:

The region bounded by the parabola 𝑦 = 𝑥 2 and the line 𝑦 = 2𝑥 in the first quadrant is
revolved about the y-axis to generate a solid. Find the volume of the solid.
68
Solution:

Self-Help: You can also refer to the sources below to help you further
understand the lesson.

* Stewart, J. (2000). Calculus: Concepts and Contexts. Australia: Cengage.


* Downing, Douglas (2009). Mathematics Terms 3rd Edition. New York: Barron’s
Educational Series Inc.
* Feliciano and Uy, F. (1983). Differential and Integral Calculus. Manila: Merriam and
Webster Bookstore, Inc.
* Peterson, Thurman. (1964). Calculus with Analytic Geometry. Philippines: Harper and
Row Publishers, Incorporated.

69
Let’s Check

Activity 1. Evaluate the following definite integrals by using the fundamental theorem of
Integral Calculus.

1. Find the area of the region bounded above by the curve 𝑦 = 2𝑒 −𝑥 + 𝑥, below by
𝑥
the curve 𝑦 = 𝑒2 , on the left by 𝑥 = 0, and on the right by 𝑥 = 1.

2. Find the area of the region enclosed by the parabola 𝑦 = 2 − 𝑥 2 and the line 𝑦 =
−𝑥.

3. Find the area of the region in the first quadrant that is bounded above by 𝑦 = √𝑥
and below the x- axis and thr line 𝑦 = 𝑥 − 2.

4. Find the volume of the solid generated by revolving the region between the 𝑦 −
𝑎𝑥𝑖𝑠 and the curve 𝑥 = 2/𝑦, 𝑎 = 1, 𝑏 = 4, about the y-axis.

5. Find the volume of the solid generated by revolving the region between the
parabola 𝑥 = 𝑦 2 + 1 and the line 𝑥 = 3 about the line 𝑥 = 3.

6. Find the area bounded by the curve x=3+cos∅, y=sin∅

7. Find the area bounded by the parabola y^2=4x and the line 2x-4

Let’s Analyze

Activity 1. Evaluate the definite integral. Show your solutions neatly and legibly.

1. Find the area bounded by the curve 𝑦 = 𝑥^2, the x- axis, and the ordinates 𝑥 = 1
and 𝑥 = 3.

2. Find the smaller area cut from the circle 𝑥 2 + 𝑦 2 = 25 by the line 𝑥 = 3

3. Find the area bounded by the curve 𝑥 = 3 + cos ∅, 𝑦 = sin ∅.

4. The region bounded by the parabola 𝑦 = 𝑥 2 and the line 𝑦 = 2𝑥 in the first
quadrant is revolved about the 𝑦 − 𝑎𝑥𝑖𝑠 to generate a solid. Find the volume of
the solid.

In a Nutshell.

70
Derive the formula for the volume of a sphere using the concepts discussed.

Q&A List

Questions/Issues Answer
1.
2.
3.
4.
5.

Keywords Index

This section lists down the keywords that help students for recall. Keywords are like
Metalanguage but without the definitions and descriptions. This section includes
concepts (a word or phrase), ideas, theories, names of people, and other vital terms to
remember. Technically speaking, all those included in the Metalanguage should be part
of the Keywords. However, Keywords can also include other important concepts or ideas
not stipulated in the Metalanguage. This section also helps in the review.

Plane Area Polar Area


Area Under the Curve Cylindrical shell method
Solids of revolution Washer Method
Cylindrical disk method

Big Picture

Week 13-14: Unit Learning Outcomes (ULO): At the end of the unit, you are expected
to

a. Determine the length of a curve and the surface of revolution through


integration, and;
b. Determine the centroids of area and solids of revolution through
integration.

Big Picture in Focus: ULOa. Determine the length of a curve and the
surface of revolution through integration.

Metalanguage
71
In this section, we will now be applying all the techniques that you have learned
from the past lesson. Here you are expected to have learned all the techniques in
integration to solve the following problems in this lesson. Please refer to these definitions
(terms/symbols) in case you will encounter difficulty in understanding educational
concepts.

Length of a Curve - Arc length is the distance between two points along a section of a
curve.

Surface of Revolution - A surface of revolution is a surface in Euclidean space created


by rotating a curve (the generatrix) around an axis of rotation.

Essential Knowledge
To perform the aforesaid big picture (unit learning outcomes) for the next two (2)
weeks of the course, you need to review the concepts and theorems of definite integral
that will be laid down in the succeeding pages. Please note that you are not limited
to exclusively refer to these resources. Thus, you are expected to utilize other books
and other resources that are available in the university’s library e.g. ebrary,
search.proquest.com etc.

Length of a Curve

Another application involving the use of definite integrals is to solve the length of a curve.
Arc length would be the distance you are traversing though a path of a curve. Examples
would be calculating the distance travelled through a parabolic path. Or, if you are going
through a map and find a curved road, you might want to measure the distance on how
far you would travel it.

We begin by first defining x and y, then we will examine the curve as functions of y and
x, respectively.

Suppose that we want to find the length of the curve given in the figure from P1 to P2.

72
Now we take a point (P) in the curve and draw a tangent line between the curve and that
point naming the tangent line as dl.

The idea here is that dl is ridiculously small so that we could approximate a part of the
curve that we need to measure. Zooming in on the graph we have this.

The red line would be the part where we would approximate using dl. Now, drawing two
more lines with respect to dl, we would get a triangle. The dx would represent the small
change in the amount of x and dy would be for the small change in the amount of y. With
that, we now have a right triangle. Using the Pythagorean Theorem, we can now solve
for dl.
𝑑𝑙 2 = 𝑑𝑥 2 + 𝑑𝑦 2

Taking the square root of both sides,

√𝑑𝑙 2 = √𝑑𝑥 2 + 𝑑𝑦 2
𝑑𝑙 = √𝑑𝑥 2 + 𝑑𝑦 2

For the arc length L, taking the integral of both sides,

∫ 𝑑𝑙 = ∫ √𝑑𝑥 2 + 𝑑𝑦 2

𝐴𝑟𝑐 𝐿𝑒𝑛𝑔𝑡ℎ (𝐿) = ∫ 𝑑𝑙 = ∫ √𝑑𝑥 2 + 𝑑𝑦 2

73
Now since our curve is in the function of x, 𝑦 = 𝑓(𝑥), so we take the arc length formula
and factor out the dx term outside the square root so that our integral would be with
𝑑𝑥 2
respect to x. To factor it out, we need to multiply dy2 with 𝑑𝑥 2 ,

𝑑𝑥 2
𝐿 = ∫ 𝑑𝑙 = ∫ √𝑑𝑥 2 + 𝑑𝑦 2 ( )
𝑑𝑥 2

𝑑𝑦 2
𝐿 = ∫ 𝑑𝑙 = ∫ √𝑑𝑥 2 (1 + )
𝑑𝑥 2

𝑑𝑦 2
𝐿 = ∫ 𝑑𝑙 = ∫ √(1 + ) 𝑑𝑥
𝑑𝑥 2

Now for the limits, we go back to the big picture and look for the x-values for P1 and P2.
Since P1 is at a and P2 is at b, therefore the limits of the integral would be from a to b.

𝑏
𝑑𝑦 2
𝐿 = ∫ 𝑑𝑙 = ∫ √[1 + ( ) ] 𝑑𝑥
𝑎 𝑑𝑥

This would mean that when for every curve described with 𝑦 = 𝑓(𝑥), the equation that we
would be using is,

𝑏
𝑑𝑦 2

𝐿 = ∫ 𝑑𝑙 = ∫ [1 + ( ) ] 𝑑𝑥
𝑎 𝑑𝑥

𝑑𝑦
Where L is the arc length and 𝑑𝑥 is the first derivative with respect to x of the equation of
the curve.

Similarly, for every curve described by 𝑥 = 𝑓(𝑦), we just must do the same procedure but
instead of dx being factored out, now it would be dy. As for the limits, you just need to find
the values for P1 and P2. For P1 is c and for P2 is d. Hence, the equation would become,

𝑑
𝑑𝑥 2

𝐿 = ∫ 𝑑𝑙 = ∫ [1 + ( ) ] 𝑑𝑦
𝑐 𝑑𝑦

Example 1:
1 1
Calculate the arc length of the curve 𝑓(𝑥) = 𝑥 3 + 2 𝑥 −1 from x=1 to x =2.
6

Solution:

74
First, you need to plot the curve,

Next, choose the right equation to use for the problem, since the function is given by
1 1
𝑓(𝑥) = 6 𝑥 3 + 2 𝑥 −1 , therefore we will be using,

𝑏
𝑑𝑦 2
𝐿 = ∫ 𝑑𝑙 = ∫ √[1 + ( ) ] 𝑑𝑥
𝑎 𝑑𝑥

𝑑𝑦
Next, the equation calls that we first need to find . Thus,
𝑑𝑥

1 1
𝑓(𝑥) = 𝑥 3 + 𝑥 −1
6 2

1 3−1
1
𝑓 (𝑥) = (3)𝑥 + (−1)𝑥 −1−1
6 2
1 1
𝑓 ′ (𝑥) = 𝑥 2 − 𝑥 −2
2 2

Substituting it to the equation for arc length with limits from 1 to 2, we get,

2 2
1 1
𝐿 = ∫ √[1 + ( 𝑥 2 − 𝑥 −2 ) ] 𝑑𝑥
1 2 2
2
1 1 1
𝐿 = ∫ √[1 + 𝑥 4 − + 𝑥 −4 ] 𝑑𝑥
1 4 2 4

Simplification of the equation would give us,

2
1 1 1
𝐿 = ∫ √[ 𝑥 4 + + 𝑥 −4 ] 𝑑𝑥
1 4 2 4

75
2
1 2 1 −2 2

𝐿 = ∫ ( 𝑥 + 𝑥 ) 𝑑𝑥
1 2 2
2
1 1
𝐿 = ∫ ( 𝑥 2 + 𝑥 −2 ) 𝑑𝑥
1 2 2

Applying integration techniques,


2
1 𝑥 2+1 1 𝑥 −2+1
𝐿=| ( )+ ( )|
2 2+1 2 −2 + 1 1
2
1 𝑥3 1 𝑥 −1
𝐿 =| ( )+ ( )|
2 3 2 −1 1
2
1 𝑥3 1
𝐿 = | ( ) − (𝑥 −1 )|
2 3 2 1

Substituting the limits,

1 23 1 1 13 1
𝐿 = [ ( ) − (2−1 )] − [ ( ) − (1−1 )]
2 3 2 2 3 2
𝟏𝟕
𝑳= 𝒖𝒏𝒊𝒕𝒔 𝒂𝒏𝒔.
𝟏𝟐

Example 2:
2
Calculate the arc length of the function 𝑥 = 𝑦 3 over the interval of y=1 and y=5.

Solution:

This problem is an example wherein the given function is 𝑥 = 𝑓(𝑦). Therefore, using the
other equation,

𝑑
𝑑𝑥 2
𝐿 = ∫ 𝑑𝑙 = ∫ √[1 + ( ) ] 𝑑𝑦
𝑐 𝑑𝑦

𝑑𝑥
Again, we need to find the 𝑑𝑦 of the function,

𝑑𝑥 2 2−1
= 𝑦3
𝑑𝑦 3

76
𝑑𝑥 2
= 3
𝑑𝑦 3 √𝑦

Then, substituting it to the equation with limits from 1 to 5, we get,

5 2
2
𝐿 = ∫ √[1 + ( 3 ) ] 𝑑𝑦
1 3 √𝑦
5
4
𝐿 = ∫ √[1 + 3
] 𝑑𝑦
1 9 √𝑦 2

Applying integration techniques to solve the integral,

5
4
𝐿 = ∫ √[1 + 3
] 𝑑𝑦
1 9 √𝑦 2

For this integral, we will try to use the substitution technique, so letting
1 2
𝑢 = 𝑦3 𝑢2 = 𝑦 3

1 −2
𝑑𝑢 = 𝑦 3 𝑑𝑦
3
1
𝑑𝑢 = 2 𝑑𝑦
3𝑦 3
𝑑𝑦 = 3𝑢2 𝑑𝑢

Substituting u and dy to the equation, we will have,

5
4
𝐿 = ∫ √[1 + ] 3𝑢2 𝑑𝑢
1 9𝑢2

Simplifying the equation with algebra,

5
9𝑢2 4
𝐿 = ∫ √[ 2 + 2 ] 3𝑢2 𝑑𝑢
1 9𝑢 9𝑢
5
9𝑢2 + 4
𝐿=∫ √ 3𝑢2 𝑑𝑢
1 9𝑢2
5
√9𝑢2 + 4 2
𝐿=∫ 3𝑢 𝑑𝑢
1 3𝑢
5
𝐿 = ∫ 𝑢√9𝑢2 + 4 𝑑𝑢
1
77
Then we will be again using another substitution method letting 𝑣

𝑣 = 9𝑢2 + 4
𝑑𝑣 = 18𝑢𝑑𝑢
1
𝑑𝑢 = 𝑑𝑣
18𝑢

Substituting v and du to the equation would yield,


5
1
𝐿 = ∫ 𝑢 √𝑣 𝑑𝑣
1 18𝑢

Simplification would leave us with,

1 5
𝐿= ∫ √𝑣 𝑑𝑣
18 1

Then using the power formula, we would get

3 5
1 𝑣2
𝐿= | |
18 3
2 1
1 35
𝐿= |𝑣 2 |
27 1

2
Substituting back 𝑣 = 9𝑢2 + 4, then 𝑢2 = 𝑦 3 ,

1 2 3 5
𝐿= |(9𝑦 + 4)2 |
3
27 1

Then applying the limits,


3 3
1 2 2 2 2
𝐿= {[(9(5)3 + 4) ] − [(9(1)3 + 4) ]}
27

Using your calculators, you would get,

𝑳 ≈ 𝟒. 𝟒𝟒𝟔𝟐𝟓𝟕𝟑𝟔𝟑 𝐮𝐧𝐢𝐭𝐬. 𝐚𝐧𝐬.

78
Length of a Polar Curve

For a length of a polar curve, the equation that you would be using would be,

𝛽
𝑑𝑟 2
𝐿 = ∫ 𝑑𝐿 = ∫ √𝑟 2 + ( ) 𝑑𝜃
𝛼 𝑑𝜃

Example:

Find the length of the polar curve 𝑟 = 1 + 𝑐𝑜𝑠𝜃

Solution:

You have to note that for a polar curve it is best to graph first the equation so that we
could know the limits that we would be using. Also, graphing would be a good tool to find
any symmetry so that our calculations would be simpler. In graphing polar curves, it is
best that you do a review on that. This link would be helpful in your review.
https://www.youtube.com/watch?v=p98hm1Dksew

After graphing the polar curve, we get this figure,

As you can see in the graph, the polar curve is symmetrical with respect to horizontal
axis, therefore our limits would be 0 to π in radians or 0 to 180⁰ degrees. If you can find
any symmetry in the graph, you can opt to solve one side and just multiply it by how many
symmetries it has. Therefore, for this function since it has a symmetry, one above the
horizontal and one below, so we just multiply our equation by 2.
𝑑𝑟
Solving first for 𝑑𝜃,

𝑟 = 1 + 𝑐𝑜𝑠𝜃
𝑑𝑟
= −𝑠𝑖𝑛𝜃𝑑𝜃
𝑑𝜃
79
Using the equation given, the data gathered from the graph and dθ, we would then get
this equation,
𝜋
𝐿 = 2 ∫ √(1 + 𝑐𝑜𝑠𝜃)2 + (−𝑠𝑖𝑛𝜃)2 𝑑𝜃
0

Simplifying the equation,


𝜋
𝐿 = 2 ∫ √1 + 2𝑐𝑜𝑠𝜃 + 𝑐𝑜𝑠 2 𝜃 + 𝑠𝑖𝑛2 𝜃𝑑𝜃
0

Using trigonometric identities to simplify further,


𝜋
𝐿 = 2 ∫ √1 + 2𝑐𝑜𝑠𝜃 + 1𝑑𝜃 𝑠𝑖𝑛𝑐𝑒 𝑠𝑖𝑛2 𝜃 + 𝑐𝑜𝑠 2 𝜃 = 1
0
𝜋
𝐿 = 2 ∫ √2 + 2𝑐𝑜𝑠𝜃 𝑑𝜃
0
𝜋
𝐿 = 2 ∫ √2(1 + 𝑐𝑜𝑠𝜃) 𝑑𝜃
0

Using Half-angle identities, we will get,

𝜋
2
𝐿 = 2 ∫ √2(1 + 𝑐𝑜𝑠𝜃) ∗ 𝑑𝜃
0 2
𝜋
4(1 + 𝑐𝑜𝑠𝜃)
𝐿 = 2∫ √ 𝑑𝜃
0 2
𝜋
(1 + 𝑐𝑜𝑠𝜃)
𝐿 = 4∫ √ 𝑑𝜃
0 2
𝜋
𝜃
𝐿 = 4 ∫ 𝑐𝑜𝑠 𝑑𝜃
0 2

Integrating,
𝜃𝜋
𝐿 = 4 |2𝑠𝑖𝑛 |
20

Applying the limits,

𝜋 0
𝐿 = 8 (𝑠𝑖𝑛 − 𝑠𝑖𝑛 )
2 2
𝜋
𝐿 = 8 (𝑠𝑖𝑛 )
2
𝑳 = 𝟖 𝒖𝒏𝒊𝒕𝒔. 𝒂𝒏𝒔.
80
Surfaces of Revolution

A surface of revolution is the surface area when an arc of a curve is revolved around a
certain axis. Say we have a curve 𝑦 = 𝑓(𝑥) at the intervals (a,b) in the x-axis.

Photo grabbed from Paul’s Online Notes

Then we will try to rotate it in the x-axis. The figure would be like this.

Photo grabbed from Paul’s Online Notes

Afterwards, we are to calculate the surface area of the surface projected. To do that, we
need to take a representative arc of the curve and call it as dL. Previously, we defined dL
as the differential arc length of a curve. When dL would be rotated at the x-axis, it would
look like a thin cylinder with no covers. Also, at any point within the dL curve, the distance
would be y from the x-axis.

81
When we cut the dL strip, we made it would be just like a band of string. To solve for the
area of the strip, we just must multiply the length and the width of a band. This is because
the band would be just like a rectangle with sides dL and 2πy. If you would why 2πy, it is
because that side of the band is the circumference of a circle.

Therefore solving for the area of the strip made, we would get,

𝐴𝑟𝑒𝑎 𝑜𝑓 𝑆𝑡𝑟𝑖𝑝 = 𝐿𝑒𝑛𝑔𝑡ℎ 𝑥 𝑊𝑖𝑑𝑡ℎ


𝐴𝑟𝑒𝑎 𝑜𝑓 𝑆𝑡𝑟𝑖𝑝 = 2𝜋𝑦 ∗ 𝑑𝐿

The Area of the strip would be the differential surface area generated when revolved
about the x-axis, when we evaluate that one from a to b, therefore we get,

𝐴𝑟𝑒𝑎 𝑜𝑓 𝑆𝑡𝑟𝑖𝑝 = 𝑑𝑆 (𝑑𝑖𝑓𝑓𝑒𝑟𝑒𝑛𝑡𝑖𝑎𝑙 𝑠𝑢𝑟𝑓𝑎𝑐𝑒 𝑎𝑟𝑒𝑎)


𝑏
𝑑𝑆 = ∫ 2 𝜋𝑦 𝑑𝐿
𝑎

If you remember from the previous lesson, we derived the formula of dL which is,

𝑏
𝑑𝑦 2

𝐿 = ∫ 𝑑𝑙 = ∫ [1 + ( ) ] 𝑑𝑥
𝑎 𝑑𝑥

Therefore the formula for the Surface Area, dS, would be,

𝑏
𝑑𝑦 2
𝑆𝐴 = ∫ 𝑑𝑆 = ∫ 2𝜋𝑦√[1 + ( ) ] 𝑑𝑥
𝑎 𝑑𝑥
Wherein 𝑦 = 𝑓(𝑥), 𝑠𝑜

𝑏
𝑑𝑦 2

𝑆𝐴 = ∫ 𝑑𝑆 = ∫ 2𝜋𝑓(𝑥) [1 + ( ) ] 𝑑𝑥
𝑎 𝑑𝑥

82
For the case when the curve is given by the formula, 𝑥 = 𝑓(𝑦), the equation for the
surface area would be,
𝑑
𝑑𝑥 2
𝑆𝐴 = ∫ 𝑑𝑆 = ∫ 2𝜋𝑓(𝑦)√[1 + ( ) ] 𝑑𝑦
𝑐 𝑑𝑦

Example 1:

Calculate the surface area generated by revolving the arc of 𝑦 = 𝑥 3 from x=-1 to x=2 about
the x-axis.

Solution:

To solve for this we will be using the formula,

𝑏
𝑑𝑦 2
𝑆𝐴 = ∫ 𝑑𝑆 = ∫ 2𝜋𝑓(𝑥)√[1 + ( ) ] 𝑑𝑥
𝑎 𝑑𝑥

𝑑𝑦
since the function is in the for of y=f(x). Therefore, we need to solve first for then input
𝑑𝑥
limits fro -1 to 2.

𝑦 = 𝑥3
𝑑𝑦
= 3𝑥 2
𝑑𝑥

Input this to the equation, we will have,


2
𝑆𝐴 = ∫ 𝑑𝑆 = ∫ 2𝜋(𝑥 3 )√[1 + (3𝑥 2 )2 ]𝑑𝑥
−1
2
𝑆𝐴 = 2𝜋 ∫ 𝑥 3 √1 + 9𝑥 4 𝑑𝑥
−1

Then using substitution method, letting u=1+9x4 and du=36x3dx,


2
1
𝑆𝐴 = 2𝜋 ∫ 𝑥 3 √𝑢 𝑑𝑢
−1 36𝑥 3
2𝜋 2
𝑆𝐴 = ∫ √𝑢 𝑑𝑢
36 −1

Then using power formula,

83
3 2
𝜋 𝑢2
𝑆𝐴 = | |
18 3
2 −1

Then substituting back u=1+9x4,

𝜋 2 3
𝑆𝐴 = |(1 + 9𝑥 4 )2 |2−1
18 3

Applying the limits,

𝜋 3 3
𝑆𝐴 = {[1 + 9(2)4 ]2 − [1 + 9(−1)4 ]2 }
27
𝑺𝑨 = 𝟏𝟗𝟗. 𝟒𝟖𝟎𝟒 𝒔𝒒. 𝒖𝒏𝒊𝒕𝒔 𝒂𝒏𝒔.

Revolved about y-axis

When a curve 𝑦 = 𝑓(𝑥)is revolved about the y-axis, the equation for the surface area of
the curved revolved would be,

𝑏 𝑑𝑦 2
𝑆𝐴 = ∫ 𝑑𝑆 = ∫𝑎 2𝜋|𝑥|√[1 + (𝑑𝑥 ) ] 𝑑𝑥
or
𝑑 𝑑𝑥 2
𝑆𝐴 = ∫ 𝑑𝑆 = ∫𝑐 2𝜋|𝑥|√[1 + (𝑑𝑦) ] 𝑑𝑦

Example 2:

Find the surface area when the curve 𝑦 = 𝑥 2 from x = 0 to x= 1 when it is revolved about
y-axis.
𝑏
𝑑𝑦 2
𝑆𝐴 = ∫ 𝑑𝑆 = ∫ 2𝜋|𝑥|√[1 + ( ) ] 𝑑𝑥
𝑎 𝑑𝑥

To solve this problem we will be using again the formula,

𝑏
𝑑𝑦 2
𝑆𝐴 = ∫ 𝑑𝑆 = ∫ 2𝜋|𝑥|√[1 + ( ) ] 𝑑𝑥
𝑎 𝑑𝑥

𝑑𝑦
since the function is in the for of y=f(x). Therefore, we need to solve first for then input
𝑑𝑥
limits fro 0 to 1.

𝑦 = 𝑥2

84
𝑑𝑦
= 2𝑥
𝑑𝑥

Input this to the equation, we will have,


1
𝑆𝐴 = ∫ 𝑑𝑆 = ∫ 2𝜋|𝑥|√[1 + (2𝑥)2 ]𝑑𝑥
0
1
𝑆𝐴 = 2𝜋 ∫ 𝑥√1 + 4𝑥 2 𝑑𝑥
0

Using substitution method, letting u=1+x2 and du = 2xdx,


1
𝑑𝑢
𝑆𝐴 = 2𝜋 ∫ 𝑥 √𝑢
0 8𝑥
𝜋 1
𝑆𝐴 = ∫ √𝑢 𝑑𝑢
4 0

Using Power formula, we get

3 1
𝜋 𝑢2
𝑆𝐴 = | |
4 3
2 0

Substituting back u,

𝜋2 3 1
𝑆𝐴 = |(1 + 4𝑥 2 )2 |
43 0

Applying the limits,

𝜋 3 3
𝑆𝐴 = {[1 + 9(1)2 ]2 − [1 + 9(0)2 ]2 }
6
𝑺𝑨 = 𝟓. 𝟑𝟑 𝒔𝒒. 𝒖𝒏𝒊𝒕𝒔 𝒂𝒏𝒔.

Self-Help: You can also refer to the sources below to help you further
understand the lesson.

* Stewart, J. (2000). Calculus: Concepts and Contexts. Australia: Cengage.


* Downing, Douglas (2009). Mathematics Terms 3rd Edition. New York: Barron’s
Educational Series Inc.

85
* Feliciano and Uy, F. (1983). Differential and Integral Calculus. Manila: Merriam and
Webster Bookstore, Inc.
* Peterson, Thurman. (1964). Calculus with Analytic Geometry. Philippines: Harper and
Row Publishers, Incorporated.

Let’s Check

Activity 1. Find the arc length of the following.


3
1. 𝑓(𝑥) = 2(𝑥 − 1)2 from x=1 to x =5
3
2
2. 𝑓(𝑥) = (𝑥 2 + 1)2 from x= 1 to x = 4
3
𝑥2
3. 𝑓(𝑥) = − 𝑙𝑛𝑥 from x= 1 to x = 2
8

Activity 2. Find the surface area generated by the following by rotating at the given axis

4. 𝑓(𝑥) = ln 𝑥 from x=1 to x=3 about x-axis


3
1
5. 𝑓(𝑦) = (𝑦 2 + 2)2 from y=1 to y=2
3

Let’s Analyze

Activity 1. Find the arc length of the following. Show your solutions neatly and legibly.

1. 𝑟 = −4𝑠𝑖𝑛𝜃 from 0 to π
2. 𝑓(𝑦) = 3(2 + 𝑦)2 from y=1 to y=4

Activity 2. Find the surface area generated by the following by rotating at the given axis
3. 𝑓(𝑦) = √𝑏 2 − 𝑦 2 from y= 0 to y= b/2

In a Nutshell
Activity 1. Find the arc length of the following. Show your solutions neatly and legibly.

1. 𝑓(𝑥) = √𝑥 + 4 from x= 1 to x=7 using


𝑏 𝑑𝑦 2
a. 𝐿 = ∫ 𝑑𝑙 = ∫𝑎 √[1 + (𝑑𝑥 ) ] 𝑑𝑥
𝑑 𝑑𝑥 2
b. 𝐿 = ∫ 𝑑𝑙 = ∫𝑐 √[1 + (𝑑𝑦) ] 𝑑𝑦

Activity 1. Find the arc length of the following. Show your solutions neatly and legibly.

2. Given the ellipse with an equation


86
𝑥2 𝑦2
+ =1 𝑎>𝑏
𝑐 2 𝑑2
When rotated about the x-axis it would form what we call an ellipsoid. From the given
equation derive the formula of the surface area of an ellipsoid.

Q&A List

Questions/Issues Answer
1.
2.
3.
4.
5.

Keywords Index

This section lists down the keywords that help students for recall. Keywords are similar
to Metalanguage but without the definitions and descriptions. This section includes
concepts (a word or phrase), ideas, theories, names of people, and other vital terms to
remember. Technically speaking, all those included in the Metalanguage should be part
of the Keywords. However, Keywords can also include other important concepts or ideas
not stipulated in the Metalanguage. This section also helps in the review.

Length of Curve Surface of Revolution


Polar Length of Curve Revolved about x
Revolved about y

Big Picture in Focus: ULOb. Determine the centroids of area and


solids of revolution through integration.

Metalanguage

In this section, we will now be applying all the techniques that you have learned
from the past lesson. Here you are expected to have learned all the techniques in
integration to solve the following problems in this lesson. Please refer to these definitions
(terms/symbols) in case you will encounter difficulty in understanding educational
concepts.

Centroids- is the center of mass of a geometric object of uniform density.

Essential Knowledge
To perform the aforesaid big picture (unit learning outcomes) for the next two (2)
weeks of the course, you need to review the concepts and theorems of definite integral
87
that will be laid down in the succeeding pages. Please note that you are not limited
to exclusively refer to these resources. Thus, you are expected to utilize other books
and other resources that are available in the university’s library e.g. ebrary,
search.proquest.com etc.

Centroid of a Region

Principle 1: Symmetry
If the region has an axis of symmetry, then the centroid (𝑥, ̅ 𝑦̅)lies somewhere along that
axis. In particular, if the region has a center, then the center is centroid.

Principle 2: Additivity

If the region, having area A, consists of a finite number of pieces with areas 𝐴1 … … … 𝐴𝑛
and centroids (𝑥̅̅̅̅
1 , ̅̅̅)………,
𝑦1 (𝑥
̅̅̅̅
𝑛 , ̅̅̅),
𝑦𝑛 then

𝑥̅ 𝐴 = 𝑥̅1 𝐴1 + ⋯ + 𝑥̅𝑛 𝐴𝑛 𝑎𝑛𝑑 𝑦̅𝐴 = 𝑦̅1 𝐴1 + ⋯ + 𝑦̅𝑛 𝐴𝑛

𝑏 𝑏
1
𝑥̅ 𝐴 = ∫ 𝑥𝑓(𝑥)𝑑𝑥, 𝑦̅𝐴 = ∫ [𝑓(𝑥)]2 𝑑𝑥
𝑎 𝑎 2

Example 1:

Find the Centroid of the quarter disc shown in the figure below.

88
Solution:

The quarter disc is symmetric about the line 𝑥 = 𝑦. We know therefore that 𝑥̅ = 𝑦̅. Here,

𝑟 𝑟 𝑟
1 2
1 2 1 2 𝑥3 1
𝑦̅𝐴 = ∫ [𝑓(𝑥)] 𝑑𝑥 = ∫ (𝑟 − 𝑥 )𝑑𝑥 = [𝑟 𝑥 − ] = 𝑟 3
2
0 2 0 2 2 3 0 3
𝑓(𝑥) = √𝑟 2 − 𝑥 2

1
Since 𝐴 = 4 𝜋𝑟 2 ,
1
𝑦̅𝐴 = 𝑟 3
3

1 3
𝑟 4𝑟
𝑦̅ = 3 =
1 2 3𝜋
4 𝜋𝑟
𝑟 𝑟
𝑥̅ 𝐴 = ∫ 𝑥𝑓(𝑥)𝑑𝑥 = ∫ 𝑥√𝑟 2 − 𝑥 2 𝑑𝑥
0 0
𝑙𝑒𝑡 𝑢 = (𝑟 2 − 𝑥 2 ), 𝑑𝑢 = −2𝑥𝑑𝑥
1 0 1
= − ∫ 𝑢2 𝑑𝑢
2 𝑟2
1 2 3 0 1
= − [ 𝑢2 ] = 𝑟 3
2 3 𝑟2 3
1 3
𝑥̅ 𝐴 = 𝑟
3
1 3
𝑟 4𝑟
𝑥̅ = 3 =
1 2 3𝜋
4 𝜋𝑟
4𝑟 4𝑟
( , )
3𝜋 3𝜋

89
In the figure shown, the region Ω between the graphs of two continuous functions 𝑓 and
g/in this case, if Ω has area A and centroid (𝑥,
̅ 𝑦̅),then

𝑏 𝑏
1
𝑥̅ 𝐴 = ∫ 𝑥[𝑓(𝑥) − 𝑔(𝑥)]𝑑𝑥, 𝑦̅𝐴 = ∫ {[𝑓(𝑥)]2 − [𝑔(𝑥)]2 } 𝑑𝑥
𝑎 𝑎 2

Example 2:

Find the centroid of the region shown in the figure.

Solution:

Here there is no symmetry we can appeal to. We must carry out the
calculation

2 2
2
1 3 2 4
2
𝐴 = ∫ [𝑓(𝑥) − 𝑔(𝑥)]𝑑𝑥 = ∫ (2𝑥 − 𝑥 )𝑑𝑥 = [𝑥 − 𝑥 ] =
0 0 3 0 3
𝑏 2 2 2
2 1 4
𝑥̅ 𝐴 = ∫ 𝑥[𝑓(𝑥) − 𝑔(𝑥)]𝑑𝑥 = ∫ 𝑥(2𝑥 − 𝑥 2 )𝑑𝑥 = ∫ (2𝑥 2 − 𝑥 3 )𝑑𝑥 = [ 𝑥 3 − 𝑥 4 ] =
𝑎 0 0 3 4 0 3
2 2 2
1 1 1
𝑦̅𝐴 = ∫ {[𝑓(𝑥)]2 − [𝑔(𝑥)]2 } 𝑑𝑥 = ∫ {[2𝑥]2 − [𝑥 2 ]2 } 𝑑𝑥 = ∫ (4𝑥 2 − 𝑥 4 )𝑑𝑥
0 2 0 2 2 0
2
1 4 1 32
= [ 𝑥3 − 𝑥5] =
2 3 5 0 15
4 32
3 15 8
Therefore, 𝑥̅ = 4 = 1; 𝑦̅ = 4 =5
3 3

Pappus’s Theorem on Volumes

All the formulas that we have derived for volumes of solids of revolution are simple
corollaries to an observation made by a brilliant ancient Greek, Pappus of Alexandria
(circa 300 A.D.)
90
Theorem:

“A plane region is revolved about an axis that lies in its plane. If the region does not
cross the axis, then the volume of the resulting solid of revolution is the area of the
region multiplied by the circumference of the circle described by the centroid of the
region:
𝑉 = 2𝜋𝑅̅ 𝐴

Where A is the area of the region and 𝑅̅ is the distance from the axis to the centroid of
the region.

Basically, we have derived only two formulas for the volumes of solids of revolution:

1. The Washer Method Formula. If the region Ω is revolved about x-axis, the
resulting solid has volume
𝑏
𝑉𝑥 = ∫ 𝜋{[𝑓(𝑥)]2 − [𝑔(𝑥)]2 }𝑑𝑥
𝑎

2. The Shell Method Formula. If the region Ω is revolved about y-axis, the resulting
solid has volume
3.
𝑏
𝑉𝑦 = ∫ 2𝜋𝑥[𝑓(𝑥) − 𝑔(𝑥)]𝑑𝑥
𝑎
Note that

𝑏 𝑏
1
𝑉𝑥 = ∫ 𝜋{[𝑓(𝑥)] −2 [𝑔(𝑥)]2 }𝑑𝑥 = 2𝜋 ∫ {[𝑓(𝑥)]2 − [𝑔(𝑥)]2 } 𝑑𝑥 = 2𝜋𝑦̅𝐴 = 2𝜋𝑅̅ 𝐴
𝑎 𝑎 2
𝑏
𝑉𝑦 = ∫ 2𝜋𝑥[𝑓(𝑥) − 𝑔(𝑥)]𝑑𝑥 = 2𝜋𝑥̅ 𝐴 = 2𝜋𝑅̅ 𝐴
𝑎

The Centroid of Solid Revolution (Using Pappus’s Theorem)

for solid that revolves along x axis:

91
𝑏
𝑥̅ 𝑉𝑥 = ∫ 𝜋𝑥{[𝑓(𝑥)]2 − [𝑔(𝑥)]2 }𝑑𝑥
𝑎
𝑏
𝑦̅𝑉𝑦 = ∫ 𝜋𝑥[𝑓(𝑥) − 𝑔(𝑥)]2 𝑑𝑥
𝑎

Note that if g(x) is the x-axis, then g(x)=0.

Example 1:

Locate the centroid of a Hemisphere of radius r.

Solution:

When you revolve the quarter of a disc with respect


to y-axis it forms a hemisphere. Since hemisphere is
symmetrical along xy plane then 𝑥̅ = 0
𝑏
𝑦̅𝑉𝑦 = ∫ 𝜋𝑥[𝑓(𝑥) − 𝑔(𝑥)]2 𝑑𝑥
𝑎

Solving for volume of solid revolution with respect to


y-axis.
𝑟
𝑉𝑦 = ∫ 2𝜋𝑥 [√𝑟 2 − 𝑥 2 − 0] 𝑑𝑥
0

𝑟
𝑉𝑦 = 2𝜋 ∫ 𝑥 √𝑟 2 − 𝑥 2 𝑑𝑥
0

𝑟
= 2𝜋 ∫ 𝑥√𝑟 2 − 𝑥 2 𝑑𝑥
0
𝑙𝑒𝑡 𝑢 = (𝑟 2 − 𝑥 2 ), 𝑑𝑢 = −2𝑥𝑑𝑥
2𝜋 0 1
= − ∫ 𝑢2 𝑑𝑢
2 𝑟2
0
2 3 2𝜋 3
= −𝜋 [ 𝑢2 ] = 𝑟 𝑐𝑢. 𝑢𝑛𝑖𝑡𝑠
3 𝑟2 3
𝑟 2 𝑟
𝑦̅𝑉𝑦 = ∫ 𝜋𝑥 [√𝑟 2 − 𝑥 2 − 0] 𝑑𝑥 = ∫ 𝜋𝑥(𝑟 2 − 𝑥 2 )𝑑𝑥
0 0

𝑟 𝑟
2 3 )𝑑𝑥
𝑟 2𝑥2 𝑥4 𝜋𝑟 4
= ∫ 𝜋(𝑟 𝑥 − 𝑥 = 𝜋[ − ] =
0 2 4 0 4

92
𝜋𝑟 4
3
𝑦̅ = 4 = 𝑟
2𝜋 3 8
3 𝑟

Self-Help: You can also refer to the sources below to help you further
understand the lesson.

* Stewart, J. (2000). Calculus: Concepts and Contexts. Australia: Cengage.


* Downing, Douglas (2009). Mathematics Terms 3rd Edition. New York: Barron’s
Educational Series Inc.
* Feliciano and Uy, F. (1983). Differential and Integral Calculus. Manila: Merriam and
Webster Bookstore, Inc.
* Peterson, Thurman. (1964). Calculus with Analytic Geometry. Philippines: Harper and
Row Publishers, Incorporated.

Let’s Check

Activity 1. Find the centroid of the bounded region determined by the following curves.

1. 𝑦 = 6𝑥 − 𝑥 2 , 𝑦 = 𝑥
2. 𝑦 2 = 2𝑥, 𝑦 = 𝑥 − 𝑥 2
3. √𝑥 + √𝑦 = √𝑎, 𝑥 = 0, 𝑦 = 0

Let’s Analyze

Activity 1. Locate the Centroid of the given volumes of solid of revolution.

1. A solid cone of base radius r and height h.

2. The solid generated by revolving the region below the graph of f(x)=√x, from x=0
to x=1 about x-axis.

In a Nutshell

Locate the centroid of the solid generated by revolving about the x-axis the first quadrant
region bounded by the ellipse b2x2 + a2y2=a2b2 and the coordinate axis.

93
Q&A List

Questions/Issues Answer
1.
2.
3.
4.
5.

Keywords Index

This section lists down the keywords that help students for recall. Keywords are similar
to Metalanguage but without the definitions and descriptions. This section includes
concepts (a word or phrase), ideas, theories, names of people, and other vital terms to
remember. Technically speaking, all those included in the Metalanguage should be part
of the Keywords. However, Keywords can also include other important concepts or ideas
not stipulated in the Metalanguage. This section also helps in the review.

Centroid Washer Method


Pappu’s Theorem Shell Method

Big Picture

Week 15-16: Unit Learning Outcomes (ULO): At the end of the unit, you are expected
to

a. recall and apply Pappu’s Theorem, solve for the moment of inertia, work
and hydrostatic pressure and force;

Big Picture in Focus: ULOa. Recall and apply Pappus’s Theorem,


solve for the moment of inertia, work and hydrostatic pressure and
force.

Metalanguage

In this section, the concepts learned from the previous discussion will be applied
in the in-depth discussion of the Pappus theorem and the demonstration of ULOa. Here
you are expected to have learned all the techniques in integration to solve the following
problems in this lesson.

94
Essential Knowledge
To perform the aforesaid big picture (unit learning outcomes) for the next two (2)
weeks of the course, you need to review the concepts and theorems of definite integral
that will be laid down in the succeeding pages. Please note that you are not limited
to exclusively refer to these resources. Thus, you are expected to utilize other books
and other resources that are available in the university’s library e.g. ebrary,
search.proquest.com etc.

The Pappus Theorems

Although Pappus of Alexandria is known mainly for his very informed


commentaries on the work of earlier Greek geometries, he also proved some original
theorems. Here is an example:

Theorem. Let A be a region in the upper half plane with boundary curve 𝐶, Let 𝐸 be the
solid of revolution formed by rotating 𝐴 about the x-axis, and let 𝐹 be the surface of
revolution formed by rotating the boundary curve 𝐶 about the same axis. Let b(𝐴) and
b(𝐶) denote the centers of mass for 𝐴 and 𝐶 respectively (assuming constant density),
and let 𝑎∗ and 𝑐 ∗ be the distances from b(𝐴) and b(𝐶) to the x-axis. Then one has the
following formulas:
o The volume of 𝐸 is equal to the area of 𝐴 multiplied by 2𝜋𝑎∗ .
o The surface area of 𝐹 is equal to the length of 𝐶 multiplied by 2𝜋𝑐 ∗ .

From a physical viewpoint, both results are what one would expect, for they say
that the mass of the solid/surface of revolution is the same as that of a ring which
(𝑖) contains the centroid of the object,
(𝑖𝑖) has a center on the coordinate axis, and
(𝑖𝑖𝑖) has a constant density equal to the mass of the cross section 𝐴 or 𝐶 per unit
length

For the volume formula, we shall assume that 𝐴 is bounded by the curves 𝑥 = 𝑔(𝑦)
on the left 𝑦 = 𝑝 on the top, and 𝑦 = 𝑞 ≥ 0 on the bottom. In general one can cut most
figures up into pieces that each satisfy versions of this statement for suitable choices of
boundary curves, but we shall not attempt to justify this here.

Proof of the volume formula. The Shell Method for computing volumes yields the
formula

𝑃
𝑉𝑂𝐿𝑈𝑀𝐸(𝐸) = 2𝜋 ∫ 𝑦[𝑓(𝑦) − 𝑔(𝑦)]𝑑𝑦
𝑄

and also has the basic formula

95
𝑝
𝐴𝑅𝐸𝐴(𝐴) = ∫ [𝑓(𝑦) − 𝑔(𝑦)]𝑑𝑦
𝑞

On the other hand, the 𝑦 − 𝑐𝑜𝑜𝑟𝑑𝑖𝑛𝑎𝑡𝑒 for the centroid of 𝐴 is given by the formula
𝑝
𝑎∗ ∙ 𝐴𝑅𝐸𝐴(𝐴) = ∫ 𝑦[𝑓(𝑦) − 𝑔(𝑦)]𝑑𝑦
𝑞

and we substitute this into the first equation we obtain the conclusion of Pappus’ Theorem
for volumes.

Proof of the area formula. In this case we think of the curve 𝐶 as being given by 𝑦 =
𝑓(𝑥) ≥ 0 for 𝑝 ≤ 𝑥 ≤ 𝑞. The standard formula for the area of the surface of revolution 𝐹 is
𝑝
𝐴𝑅𝐸𝐴(𝐹) = 2𝜋 ∫ 𝑓(𝑥)√1 + 𝑓′(𝑥)2 𝑑𝑥
𝑞

and the length of the curve is given by


𝑝
𝐿𝐸𝑁𝐺𝑇𝐻(𝐶) = ∫ √1 + 𝑓′(𝑥)2 𝑑𝑥
𝑞

On the other hand, the 𝑦-coordinate for the centroid of 𝐶 is given by the formula
𝑝
𝑐 ∗ ∙ 𝐿𝐸𝑁𝑇𝐺𝑇𝐻(𝐶) = ∫ √1 + 𝑓′(𝑥)2 𝑑𝑥
𝑞

and if we substitute this into the first equation we obtain the conclusion of Pappus’
Theorem for surface areas.

Moments of Inertia for Areas

We want to find the moment of inertia, 𝐼𝑦 of the given area, which is around the
𝑦 − 𝑎𝑥𝑖𝑠. Each typical rectangle indicated has with 𝑑𝑥 and height 𝑦2 − 𝑦1 , so its area is
(𝑦2 − 𝑦1 )𝑑𝑥. If 𝑘 is the mass per unit area, then each typical rectangle has mass [𝑘(𝑦2 −
𝑦1 )𝑑𝑥]𝑥 2 , since each rectangle is x units from the 𝑦 − 𝑎𝑥𝑖𝑠. We can add the moments of
inertia for all the typical rectangles making up the area using integration:
96
𝑏
𝐼𝑦 = 𝑘 ∫ 𝑥 2 (𝑦2 − 𝑦1 )𝑑𝑥
𝑎

The radius of gyration 𝑅𝑦 is given by:

𝐼𝑦
𝑅𝑦 = √
𝑚

where 𝑚 is the mass of the area.

Example. For the first quadrant area bounded by the curve 𝑦 = 1 − 𝑥2 , find (a) the
moment of inertia with respect to the y-axis (𝐼𝑦 ) (b) the mass of the area (c) the radius of
gyration.

As usual, first we sketch the part of the curve in the first quadrant. It’s a parabola, passing
through (1,1) and (0,1). A typical rectangle is shown.

In this example, 𝑦2 = 1 − 𝑥 2 , and 𝑦1 = 0, 𝑎 = 0 𝑎𝑛𝑑 𝑏 = 1

(a) Finding 𝐼𝑦 :

𝑏
𝐼𝑦 = 𝑘 ∫𝑎 𝑥 2 (𝑦2 − 𝑦1 )𝑑𝑥
1
= 𝑘 ∫0 𝑥 2 [(1 − 𝑥 2 ) − 0]𝑑𝑥

1
= 𝑘 ∫0 [𝑥 2 − 𝑥 4 ]𝑑𝑥
1
𝑥3 𝑥5
= 𝑘[3 − ]
5 0
1 1
= 𝑘 (3 − 5)
2𝑘
= 15
97
(b) The mass of the area, 𝑚.

Now 𝑚 = 𝑘𝐴, where 𝐴 is the area .


1
𝐴 = ∫0 (1 − 𝑥 2 )𝑑𝑥

1
𝑥3
= [𝑥 − ]
3 0

1 2 2𝑘
=1−3=3 so 𝑚 = 𝑘𝐴 = 3

(c) The radius of gyration:

𝐼𝑦 2𝑘⁄15 1
𝑅𝑦 = √𝑚 = √ 2𝑘⁄3 = √5 ≈ 0.447

2𝑘
This means that if a mass of 3 was placed 0.447 units from the y-axis, this would have
the same moment of inertia as the original shape.

Rotation about the x-axis

For rotation about the x-axis, the moment of inertia formulae become:
𝑑
𝐼𝑥 = 𝑘 ∫ 𝑦 2 (𝑥2 − 𝑥1 )𝑑𝑦
𝑐

and

𝐼𝑥
𝑅𝑥 = √
𝑚

Example. Find the moment of inertia and the radius of gyration for the area 𝑦 = 𝑥2 + 1
from 𝑥 = 1 to 𝑥 = 2, and 𝑦 > 1, when rotated around the 𝑥 − 𝑎𝑥𝑖𝑠. The mass per unit area
is 3 kg m-2.

We sketch the parabola. The shaded area is the part that’s rotating around the 𝑥 − 𝑎𝑥𝑖𝑠,
and we have indicated a typical rectangle.

98
In this example, we are rotating the area around the x-axis. We need to express our
function in terms of 𝑦. Since we started with 𝑦 = 𝑥2 + 1, we solve for 𝑥 and obtain:

𝑥 = √𝑦 − 1

We take the positive case only, as we are dealing with the first quadrant. So the required
values which we can use in the formula are:
𝑥2 = 2 (the curve furthest from the y-axis)
𝑥1 = √𝑦 − 1 (the curve closest to the y-axis)
𝑐 = 1 and 𝑑 = 5; and 𝑘 = 3.

(a) Finding 𝐼𝑥
𝑑
𝐼𝑥 = 𝑘 ∫𝑐 𝑦 2 (𝑥2 − 𝑥1 )𝑑𝑦

5
= 3 ∫1 𝑦 2 (2 − √𝑦 − 1)𝑑𝑦

5
= 3 ∫1 (2𝑦 2 − 𝑦 2 √𝑦 − 1)𝑑𝑦

= 45.5 kg m2

(b) Now to find the mass of the area, 𝑚. 𝑚 = 𝑘𝐴, where 𝐴 is the area.
2
𝐴 = ∫1 (𝑥 2 + 1)𝑑𝑥 − 1 (we need to subtract the area of the 1x1=1 square below the shaded area)
2
𝑥3 8 1 7
= [ 3 + 𝑥] − 1 = (3 + 2) − (3 + 1) − 1 = 3
1
7
So 𝑚 = 𝑘𝐴 = 3𝑥 3 = 7 𝑘𝑔

99
(c) The radius of gyration:

𝐼 45.5
𝑅𝑥 = √𝑚𝑥 = √ ≈ 2.6 m
7

This means that if a single mass of 7 kg was placed 2.6 m from the x-axis, this would
have the same moment of inertia as the original shape when rotating around the x-axis.

Work by Integration

Work is defined as the amount of energy required to perform a physical task. When
force is constant, work can simply be calculated using the equation

𝑊 =𝐹∙𝑑

where 𝑊 is work, 𝐹 is a constant force, and 𝑑 is the distance through which the force
acts. The units of work are commonly Newton-meters, 𝑁𝑚; 𝐽; or foot-pound, 𝑓𝑡 − 𝑙𝑏.
Frequently, the force is not constant and will change over time. In order to solve for work
with a variable force, the following integral equation must be used
𝑥=𝑏
𝑊=∫ 𝑓(𝑥)𝑑𝑥
𝑥=𝑎

where 𝑊 is work, 𝑓(𝑥) is force as a function of distance, and 𝑥 is distance.

1. Finding the work required to stretch a spring

If an ideal spring is stretched or compressed x units beyond its natural length, then
Hooke’s Law, 𝑓⃗(𝑥) = 𝑘𝑥⃗ tells us the force the spring is exerting to resist that actin. The
proportionality constant k depends on the stiffness of the spring and is determined
through empirical testing.

Example. A spring has a natural length of 1 meter. A force of 25 Newtons stretches the
spring by 1⁄4 of a meter. Determine how much work is done by stretching the spring.

a) 2 meters beyond its natural length


b) From a length of 1.5 meters to 2.5 meters

We first determine the spring constant, k. Because the force is 25 N when x is 0.25 m, we
can use Hooke’s law to determine k.
𝑓(𝑥) = 𝑘𝑥
1
25𝑁 = 𝑘 ( 𝑚)
4
𝑁
∴ 𝑘 = 100 𝑎𝑛𝑑 𝑓(𝑥) = 100𝑥
𝑚

100
Hence, to find the work done by stretching the spring from its rest position to 2 meters
beyond that resting position, we do the following:

𝑏 2
100 2 2
𝑊 = ∫ 𝑓(𝑥)𝑑𝑥 = ∫ 100𝑥𝑑𝑥 = 𝑥 ] = 50(2)2 − 50(0)2 = 200𝑁𝑚 𝑜𝑟 200𝐽
𝑎 0 2 0

Similarly, to find the work done by stretching the spring from a length of 1.5 m to 2.5 m:

𝑏 1.5
100 2 1.5
𝑊 = ∫ 𝑓(𝑥)𝑑𝑥 = ∫ 100𝑥𝑑𝑥 = 𝑥 ] = 50(1.5)2 − 50(0.5)2 = 100𝑁𝑚 𝑜𝑟 100𝐽
𝑎 0.5 2 0.5

Note: In this type of problem, we need to pay attention to boundaries. If the spring is not
stretched, no matter what the length of it is, the lower boundary must be zero. If the spring
is stretched to a certain length, then we need to subtract the natural length of it from that
value. This then will determine the appropriate boundaries of the integral.

2. Finding the work required to wind a wire around a drum

When winding up a wire, a force has to be applied to pull the mass up against
gravity. Therefore, gravitational weight of the mass is equal to the required force to wind
the wire. The potential energy gained by the mass is the same as the work done by
winding up the wire.

Example. Find the work generated from one revolution of the pictured massless pulley
and massless wire system. The mass of the block is 3 kg.

For one rotational revolution of the pulley, the distance the weight
is raised is equal to one circumference; therefore, 2𝜋𝑟 = 2𝜋 ∙ 2 =
4𝜋. The gravitational force on 3 kg is determined by 𝑚𝑔 = (3𝑘𝑔) ∙
𝑚
(9.8 𝑠2 ) = 29.4 𝑁. Therefore,

4𝜋 4𝜋
𝑊=∫ 3 ∙ 9.8𝑑𝑥 = 29.4 ∫ 𝑑𝑥 = 29.4𝑥]4𝜋
0 = 29.4 ∙ 4𝜋 − 29.4 ∙ 0 = 369𝐽
0 0
(rounded to 3 significant
digits)

101
Example. We are given a fully extended cable of 150 feet weighing 2.00 lb/ft. How much
work is done after winding 50 feet of cable?
𝑙𝑏
𝐹 = 2 𝑓𝑡 (150 − 𝑥)𝑓𝑡 [why (150-x)? At the beginning there will
be 150 feet of hanging cable. As the cable is wound up (by x feet)
the cable becomes shorter and shorter, weighing less and less.
We stop after x=50 feet are wound, i.e., when there is only 100
feet of cable left hanging.]

50 50
𝑊 = ∫ 2(150 − 𝑥)𝑑𝑥 = ∫ (300 − 2𝑥)𝑑𝑥 = 300 ∙ 50 − 502
0 0
= 1.25 𝑥 104 𝑓𝑡 ∙ 𝑙𝑏

3. Finding work required to pump liquid from a tank

Pumping liquid out of the top of a tank requires work because the liquid is moving
against gravity. To calculate this, we imagine the work required to lift small disks of liquid
up and out of the tank. So we are lifting a series of masses against gravity and allowing
the liquid to spill out once the top is reached. We are asked to calculate the work
performed in all of this activity. Recall that weight = mass times the force of gravity (𝑊 =
𝑓𝑡 𝑚
𝑚𝑔), where W is weight, m is mass,and g is the gravitational constant, 32𝑠2 or 9.8𝑠2 .
Sometimes the weight is provided and sometimes the mass is. What varies in these
systems is the distance each disc needs to be lifted, measured by taking the total height,
𝐻, and subtracting from this the present height of the remaining liquid, 𝑥, and the volume
of each disc.

Example. Find the work done by pumping out water from the top of a cylindrical tank 3.00
𝑙𝑏
ft in radius and 10 ft tall, if the tank is initially full. (The density of water is 62.4𝑓𝑡 3)

𝐹 = [𝐷𝑒𝑛𝑠𝑖𝑡𝑦] ∙ [𝐴𝑟𝑒𝑎 𝑜𝑓 𝐷𝑖𝑠𝑐] ∙ [𝐻𝑒𝑖𝑔ℎ𝑡 𝑜𝑓 𝐷𝑖𝑠𝑐]


= [𝜌] ∙ [𝜋𝑟 2 ] ∙ [𝑑𝑥]

Since 𝑊𝑜𝑟𝑘 = (𝐹 ∙ 𝑑𝑖𝑠𝑡𝑎𝑛𝑐𝑒) and the distance is H-x,


Then
10
𝑊 = ∫ (𝜌)(𝜋𝑟 2 )(𝐻 − 𝑥)𝑑𝑥
0
10
= ∫0 (62.4)(𝜋 ∙ 32 )(10 − 𝑥)𝑑𝑥
10
= 561. 𝜋 ∫0 (10 − 𝑥)𝑑𝑥

102
1 10
= 561.6π [10𝑥 − 2 𝑥2 ]
0
1
= 561.6𝜋 [100 − 2 (10)2 ] = 8.82 ∙ 104 𝑓𝑡 ∙ 𝑙𝑏

Example. Find the work done by pumping out molasses from a conical tank filled to 2 ft
from the top of the tank. The tank has a maximum radius of 3 ft and a height of 10 ft.
𝑙𝑏
Molasses weighs 100𝑓𝑡 3.

The added difficulty here results from the fact


that as the height through which each disc is
lifted changes, so does the radius change. We
would prefer to integrate with respect to a single
variable; therefore, we seek a relationship
between the variables h and r.

ℎ 10
Using similar triangles: =
𝑟 3
3
𝑟 = 10 ℎ

𝐹 = [𝐷𝑒𝑛𝑠𝑖𝑡𝑦] ∙ [𝐴𝑟𝑒𝑎 𝑜𝑓 𝐷𝑖𝑠𝑐] ∙ [𝐻𝑒𝑖𝑔ℎ𝑡 𝑜𝑓 𝐷𝑖𝑠𝑐] = [𝜌] ∙ [𝜋𝑟 2 ] ∙ [𝑑ℎ]

8 8 2 8
3
𝑊 = ∫ (𝜌)( 𝜋𝑟 )(𝐻 − ℎ)𝑑ℎ = ∫ (100) [𝜋 ( ℎ) (10 − ℎ)𝑑ℎ = 9𝜋 ∫ ℎ2 (10 − ℎ)𝑑ℎ]
2
0 0 10 0

8
8 10ℎ3 ℎ4 10∙83 84
= 9𝜋 ∫0 (10ℎ2 − ℎ3 )𝑑ℎ = 9𝜋 [ − ] = 9𝜋 [ − ]
3 4 0 3 4
= 1.930 𝑥 104 𝑓𝑡 ∙ 𝑙𝑏

4. Finding the work required to move two particles

𝑘𝑞 𝑞
Coulomb’s Law states that 𝐹 = 𝑟12 2; like charges repel, opposite charges attract, where
k, the proportionality constant is approximately equal to 9𝑥109 𝑁𝑚2 𝐶 −2 . Therefore, the
energy required to bring two particles of charge q1 and q2 from infinity to a distance, R,
is:

103
𝑅 𝑅 𝑅
𝑘𝑞1 𝑞2 1 1 1
𝑊 = ∫ 𝐹 ∙ 𝑑𝑟 = ∫ 𝑑𝑟 = 𝑘𝑞1 𝑞2 ∫ 𝑑𝑟 = 𝑘𝑞1 𝑞2 [− − (− )]
∞ ∞ 𝑟2 ∞ 𝑟
2 𝑅 ∞
𝑘𝑞1 𝑞2
= − 𝑅

Notice the similarity between this and Newton’s Law of Gravitation.

Example. Find the work required to bring together a 0.8 nC negative charge and a 0.6
nC negative charge from a distance of 200 mm (0.2 m) to a distance of 100 mm (0.1 m).
0.1 0.1 (9 0.1
𝑘𝑞1 𝑞2 ∙ 109 )(−0.8 ∙ 10−9 )(−0.6 ∙ 10−9 ) −9
1
𝑊=∫ 2
𝑑𝑟 = ∫ 2
𝑑𝑟 = 4.32 ∙ 10 ∫ 2
𝑑𝑟
0.2 𝑟 0.2 𝑟 0.2 𝑟

1 0.1 1 1
= 4.32 ∙ 10−9 [− 𝑟 ] = 4.32 ∙ 10−9 [− 0.1 − (− 0.2)] = −2.16 ∙ 10−8 𝐽
0.2

Force due to Hydrostatic Pressure

When a dam is built, it is imperative to for engineers to understand how much force
water will exert against the face of the dam. The first thing we realize is the force exerted
by the fluid is related to the natural concept of pressure. The pressure a force exerts on
a region is measured in units of force per unit of area: for example, the air pressure in a
tire is often measured in pounds per square inch (PSI). Hence, we see that the general
relationship is given by
𝐹
𝑃 = , 𝑜𝑟 𝐹 = 𝑃 ∙ 𝐴,
𝐴

where P represents pressure, F represents force, and A the area of the region being
considered. Of course, in the equation F = PA, we assume that the pressure is constant
over the entire region A.

Most people know from experience that the deeper one dives underwater while
swimming, the greater the pressure that is exerted by the water. This is due to the fact
that the deeper one dives, the more water there is right on top of the swimmer: it is the
force that “column” of water exerts that determines the pressure the swimmer
experiences. To get water pressure measured in its standard units (pounds per square
foot), we say that the total water pressure is found by computing the total weight of the
column of water that lies above a region of area 1 square foot at a fixed depth. Such a
rectangular column with a 1 × 1 base and a depth of d feet has volume V = 1 · 1 · d ft 3,
and thus the corresponding weight of the water overhead is 62.4d. Since this is also the
amount of force being exerted on a 1 square foot region at a depth d feet underwater, we
see that P = 62.4d (lbs/ft2) is the pressure exerted by water at depth d.

The understanding that P = 62.4d will tell us the pressure exerted by water at a
depth of d, along with the fact that F = PA, will now enable us to compute the total force
that water exerts on a dam, as we see in the following example.

104
Example. Consider a trapezoid-shaped dam that is 60 feet wide at its base and 90 feet
wide at its top, and assume the dam is 25 feet tall with water that rises to within 5 feet of
the top of its face. Water weighs 62.5 pounds per cubic foot. How much force does the
water exert against the dam?

First, we sketch a picture of the dam, as shown in the figure below. Note that, as
in problems involving the work to pump out a tank, we let the positive x-axis point down.

It is essential to use the fact that pressure is constant at a fixed depth. Hence, we
consider a slice of water at constant depth on the face, such as the one shown in the
figure. First, the approximate area of this slice is the area of the pictured rectangle. Since
the width of that rectangle depends on the variable 𝑥 (which represents the how far the
slice lies from the top of the dam), we find a formula for the function 𝑦 = 𝑓(𝑥) that
determines one side of the face of the dam. Since 𝑓 is linear, it is straightforward to find
3
that 𝑦=𝑓(𝑥) = 45 − 5𝑥. Hence, the approximate area of a representative slice is

3
𝐴𝑠𝑙𝑖𝑐𝑒 = 2 ∙ 𝑓(𝑥) ∙ Δ𝑥 = 2 (45 − 𝑥) ∙ Δ𝑥
5

At any point on this slice, the depth is approximately constant, and thus the
pressure can be considered constant. In particular, we note that since 𝑥 measures the
distance to the top of the dam, and because the water rises to within 5 feet of the top of
the dam, the depth of any point on the representative slice is approximately (𝑥−5) . Now,
since pressure

The figure is a trapezoidal dam that is 25


feet tall, 60 feet wide at its base, 90 feet
wide at its top, with the water line 5 feet
down from the top of its face.

is given by 𝑃=62.4𝑑 , we have that at


any point on the representative slice

𝑃𝑠𝑙𝑖𝑐𝑒 = 62.4 ∙ (𝑥 − 5)

Knowing both the pressure and area, we


can find the force the water exerts on the
slice. Using 𝐹=𝑃𝐴 , it follows that
3
𝐹𝑠𝑙𝑖𝑐𝑒 = 𝑃𝑠𝑙𝑖𝑐𝑒 ∙ 𝐴𝑠𝑙𝑖𝑐𝑒 = 62.4(𝑥 − 5) ∙ (2) (45 − 𝑥) (Δ𝑥)
5

Finally, we use a definite integral to sum the forces over the appropriate range of 𝑥 values.
Since the water rises to within 5 feet of the top of the dam, we start at 𝑥 = 5 and slice all
the way to the bottom of the dam, where 𝑥 = 30. Hence,

105
𝑥=30
3
𝐹=∫ 62.4 ∙ (𝑥 − 5) ∙ (2) (45 − 𝑥) 𝑑𝑥
𝑥=5 5

𝐹 ≈ 1.248 ∙ 106

Self-Help: You can also refer to the sources below to help you further
understand the lesson.

* Stewart, J. (2000). Calculus: Concepts and Contexts. Australia: Cengage.


* Downing, Douglas (2009). Mathematics Terms 3rd Edition. New York: Barron’s
Educational Series Inc.
* Feliciano and Uy, F. (1983). Differential and Integral Calculus. Manila: Merriam and
Webster Bookstore, Inc.
* Peterson, Thurman. (1964). Calculus with Analytic Geometry. Philippines: Harper and
Row Publishers, Incorporated.

Let’s Check

Activity 1. Solve the following problems below:

1. Find the moment of inertia of the area bounded by the curve 𝑥2 = 4𝑦, the line y=1
and the y-axis on the first quadrant with respect to y-axis.
2. A spring has a natural length of 250 cm. A force of 18 Newtons stretches the spring
to a length of 5 meters. Determine how much work is done by stretching the spring
a. 150 cm beyond its natural length
b. from a length of 3.5 meters to 5.0 meters
3. The end of a tank containing water is vertical and has the shape of the bottom of
a half circle or radius 10 m. Determine the hydrostatic force against the end of the
tank.

Let’s Analyze

Activity 1. Solve the following problems below:

1. Find the moment of inertia of the area bounded by the curve 𝑥2 = 8𝑦, the line x=4
and the x-axis on the first quadrant with respect to x-axis.
2. Find the work done winding 10 feet of a 25-ft cable that weighs 4.00 lb/ft when
there is a 50 lb mass that hangs on the end.
3. The end of a tank containing water is vertical and has the shape of a triangle with
base 4 ft and height 6 ft. The tank is full to a level of 4 ft. Determine the Force on
this end.

106
In a Nutshell

Q&A List

Questions/Issues Answer
1.
2.
3.
4.
5.

Keywords Index

This section lists down the keywords that help students for recall. Keywords are like
Metalanguage but without the definitions and descriptions. This section includes
concepts (a word or phrase), ideas, theories, names of people, and other vital terms to
remember. Technically speaking, all those included in the Metalanguage should be part
of the Keywords. However, Keywords can also include other important concepts or ideas
not stipulated in the Metalanguage. This section also helps in the review.

The Pappus Theorem Spring


Moments of Inertia for Area Hydrostatic Pressure
Work by Integration Force Due to Hydrostatic Pressure

Big Picture

Week 17-18: Unit Learning Outcomes (ULO): At the end of the unit, you are expected
to

a. recall and apply knowledge of the definite integrals and apply integration
techniques to solve multiple integrals, and;
b. demonstrate deep learning about surface tracing more specifically integration of
different types of surfaces.

Big Picture in Focus: ULOa. Recall and apply knowledge of the


definite integrals and apply integration techniques to solve multiple
integrals.

107
Metalanguage

In this section, the concept of definite integral in multiple integrals will be applied
and discussed to demonstrate ULOa. Please refer to these definitions in case you will
encounter difficulty in understanding educational concepts.

Double Integral. A definite integral that are extended to functions that includes more than
one variable to integrate a two-dimensional region.

Triple Integral. It is an integration that includes three variables to integrate three-


dimensional region.

Essential Knowledge

To perform the aforesaid big picture (unit learning outcomes) for the last two weeks
of this course, you need to review and apply the concepts and knowledge of definite
integral that were discussed from the previous weeks. Please note that you are not limited
to exclusively refer to these resources. Thus, you are expected to utilize other books,
research articles and other resources that are available in the university’s library e.g.
ebrary, search.proquest.com etc.

Double Integrals

This type of integral includes two or more variables in an integral function. It is also defined
as the limit of the sums and takes the form of ∬ 𝑓(𝑥, 𝑦)𝑑𝑦𝑑𝑥. In this case the function will
be completely integrated first with respect to y and then with respect to x. That being said,

the integration process depends on the order of how the derivative appears in the given
equation.

Examples:
3 4
1. Evaluate ∫1 ∫1 (𝑥 2 − 𝑦)𝑑𝑦𝑑𝑥.

Solution:

1st step: Take the inner limit first and integrate completely the function with respect to y,
keeping the other variable as constant.

3 4 3 4
∫1 ∫1 (𝑥 2 − 𝑦)𝑑𝑦𝑑𝑥 = ∫1 [ ∫1 (𝑥 2 − 𝑦)𝑑𝑦𝑑𝑥]
3 4
= ∫1 [∫1 (𝑥 2 − 𝑦)𝑑𝑦] 𝑑𝑥
3 𝑦2 4
= ∫1 [𝑥 2 𝑦 − ] 𝑑𝑥
2 1

108
3 (4)2 (1)2
= ∫1 {[𝑥 2 (4) − ] − [𝑥 2 (1) − ]} 𝑑𝑥
2 2
3 15
= ∫1 (3𝑥 2 − ) 𝑑𝑥
2

2nd step: Integrate the resulting function with respect to x.


3𝑥3 15𝑥3
= [ 3 − 2 ]1
= 11

3 4
Therefore, ∫1 ∫1 (𝑥 2 − 𝑦)𝑑𝑦𝑑𝑥 is equal to 11.

2 4
2. Evaluate ∫0 ∫0 (𝑥 + 1)𝑑𝑦𝑑𝑥.

Solution:

2 4 2 4
∫0 ∫0 (𝑥 + 1)𝑑𝑦𝑑𝑥 = ∫0 [∫0 (𝑥 + 1)𝑑𝑦] 𝑑𝑥
2
= ∫0 [𝑥𝑦 + 𝑦] 40 𝑑𝑥
2
= ∫0 [(4𝑥 + 4) − 0]𝑑𝑥
4𝑥 2
=[ + 4𝑥] 20
2
4(2)2
= [( + 4(2)) − 0]
2

= 16

2 4
Therefore, ∫0 ∫0 (𝑥 + 1)𝑑𝑦𝑑𝑥 is equal to 16.
4 2𝑦
3. ∫0 ∫𝑦 (8𝑥 + 𝑒 𝑦 ) 𝑑𝑥𝑑𝑦

Solution:

4 2𝑦 4 2𝑦
∫0 ∫𝑦 (8𝑥 + 𝑒 𝑦 ) 𝑑𝑥𝑑𝑦 = ∫0 [∫𝑦 (8𝑥 + 𝑒 𝑦 )𝑑𝑥] 𝑑𝑦
4 8𝑥 2
= ∫0 [ + 𝑥𝑒 𝑦 ] 2𝑦
𝑦
𝑑𝑦
2
4
= ∫0 [4𝑥 2 + 𝑥𝑒 𝑦 ] 2𝑦
𝑦
𝑑𝑦
4
= ∫0 [4(2𝑦)2 + (2𝑦)𝑒 𝑦 ] − [4(𝑦)2 + (𝑦)𝑒 𝑦 ]𝑑𝑦
4
= ∫0 [16𝑦 2 + 2𝑦𝑒 𝑦 − 4𝑦 2 − 𝑦𝑒 𝑦 ]𝑑𝑦
4
= ∫0 (12𝑦 2 + 𝑦𝑒 𝑦 )𝑑𝑦

From 𝑦𝑒 𝑦 , let u = y; du = dy
dv = 𝑒 𝑦 𝑑𝑦; v = 𝑒 𝑦

109
12𝑦 3
=[ + 𝑦𝑒 𝑦 − ∫ 𝑒 𝑦 𝑑𝑦] 40
3
= [4𝑦 3 + 𝑦𝑒 𝑦 − 𝑒 𝑦 ] 40
= [4(4)3 + 4𝑒 4 − 𝑒 4 ] − [4(0)3 + (0)𝑒 0 − 𝑒 0 ]
= 256 + 4𝑒 4 − 𝑒 4 + 1
= 257+3𝑒 4
= 420.79

4 2𝑦
Therefore, ∫0 ∫𝑦 (8𝑥 + 𝑒 𝑦 ) 𝑑𝑥𝑑𝑦 is equal to 420.79.

Evaluation by Inversion of Order

There are some integral functions that can’t be solved using elementary functions. Thus,
the order of integration must be first changed and will then yield to the elementary
methods.

Example 1:
1 1 2
Evaluate ∫0 ∫𝑥 𝑒 𝑦 𝑑𝑦 𝑑𝑥 .

Solution:

The integrand cannot be solved with respect to y. Therefore, we need to invert the order
of integration and integrating the function with respect to x. To do that, the limits will also
be changed, accordingly.
1=𝑥 1=𝑦
2
∫ ∫ 𝑒 𝑦 𝑑𝑦 𝑑𝑥
0=𝑥 𝑥=𝑦

Since the first order of integration is with respect to y, therefore, the inner limits are also
equal to y and the outer limits are respectively equal to x.

If we graph the limits in a Cartesian plane, with x = 1, x = 0, y = 1 and y = x, we form a


triangle that is bounded by x = 0, y = x and y = 1. Therefore, the new limits are:
1 1 2 1 𝑦 2
∫0 ∫𝑥 𝑒 𝑦 𝑑𝑦 𝑑𝑥 = ∫0 ∫0 𝑒 𝑦 𝑑𝑥 𝑑𝑦
1 2
= ∫0 [𝑥𝑒 𝑦 ] 𝑦0 𝑑𝑦
1 2 2
= ∫0 [(𝑦)𝑒 𝑦 − (0)𝑒 𝑦 ]𝑑𝑦
1 2
= ∫0 𝑦𝑒 𝑦 𝑑𝑦
2
𝑦2𝑒 𝑦
=[ ] 10
2
2
𝑒𝑦
= −0
2

110
2
𝑒𝑦
=[ ] 10
2
1
= 2 (𝑒 − 1)

1 1 2 1
Therefore, ∫0 ∫𝑥 𝑒 𝑦 𝑑𝑦 𝑑𝑥 is equal to 2 (𝑒 − 1) ≈ 0.86

Triple Integrals

Triple integrals follow the same rules in integrating the integrands as double integrals.
However, the function already involves three variables and therefore requires to be
integrated three times.

Examples:

1 6 5 2𝑧
1. Evaluate ∫0 ∫3 ∫2 (𝑥 2 + 2𝑦 3 + ) 𝑑𝑥𝑑𝑦𝑑𝑧.
3

Solution:

1 6 5 2𝑧 1 6 𝑥3 52𝑥𝑧
∫0 ∫3 ∫2 (𝑥 2 + 2𝑦 3 + 3
) 𝑑𝑥𝑑𝑦𝑑𝑧 = ∫0 ∫3 [ 3 + 2𝑥𝑦 3 +2 3
] 𝑑𝑦𝑑𝑧
1 6 (5)3 2(5)𝑧 (2)3 2(2)𝑧
= ∫0 ∫3 {[ + 2(5)𝑦 3 + ] −[ + 2(2)𝑦 3 + ]} 𝑑𝑦𝑑𝑧
3 3 3 3
1 6 125 10𝑧 8 4𝑧
= ∫0 ∫3 {[ + 10𝑦 3 + ] − [3 + 4𝑦 3 + ]} 𝑑𝑦𝑑𝑧
3 3 3
1 6
= ∫0 ∫3 (39 + 6𝑦 3 + 2𝑧)𝑑𝑦𝑑𝑧
1 6𝑦 4
= ∫0 [39𝑦 + + 2𝑦𝑧] 63 𝑑𝑧
4
1 6(6)4 6(3)4
= ∫0 {[39(6) + + 2(6)𝑧] − [39(3) + + 2(3)𝑧]} 𝑑𝑧
4 4
1 472
= ∫0 {[2178 + 12𝑧] − [ + 6𝑧]} 𝑑𝑧
2
1 3879
= ∫0 [ + 6𝑧] 𝑑𝑧
2
3879 6𝑧 2 1
=[ 𝑧+ ]
2 2 0
3879 6(1)2
= {[ (1) + ] − [0]}
2 2
3885
= 2

1 6 5 2𝑧 3885
Therefore, ∫0 ∫3 ∫2 (𝑥 2 + 2𝑦 3 + ) 𝑑𝑥𝑑𝑦𝑑𝑧 is equal to .
3 2

6 3𝑦 𝑦+2
2. Evaluate ∫−2 ∫0 ∫𝑥 (6𝑥𝑦𝑧 𝑑𝑧𝑑𝑦𝑑𝑥).

Solution:
6 3𝑦 𝑦+2 6 3𝑦 6𝑥𝑦𝑧 2 𝑦+2
∫−2 ∫0 ∫𝑥 (6𝑥𝑦𝑧 𝑑𝑧𝑑𝑦𝑑𝑥) = ∫−2 ∫0 [ ] 0 𝑑𝑦𝑑𝑥
2

111
6 3𝑦 6𝑥𝑦(𝑦+2)2 6𝑥𝑦(𝑥)
= ∫−2 ∫0 {[ ]−[ ]} 𝑑𝑦𝑑𝑥
2 2
6 3𝑦 6𝑥𝑦 2 +12𝑥𝑦 6𝑥 2 𝑦
= ∫−2 ∫0 [ 2
− 2 ] 𝑑𝑦𝑑𝑥
6 3𝑦
= ∫−2 ∫0 [3𝑥𝑦 2 + 6𝑥𝑦 − 3𝑥 2 𝑦]𝑑𝑦𝑑𝑥
6 3𝑥𝑦 3 6𝑥𝑦 2 3𝑥 2 𝑦 2 3𝑦
= ∫−2 [ 3 + 2 − 2 ] 0 𝑑𝑥
6 3𝑥 2 𝑦 2 3𝑦
= ∫−2 [𝑥𝑦 3 + 3𝑥𝑦 2 − 2 ] 0 𝑑𝑥
6 3𝑥 2 (3𝑦)2
= ∫−2 {[𝑥(3𝑦)3 + 3𝑥(3𝑦)2 − 2
] − [0]} 𝑑𝑥
6 27
= ∫−2(27𝑥𝑦 3 + 27𝑥𝑦 2 − 2 𝑥 2 𝑦 2 ) 𝑑𝑥
27𝑥 2 𝑦 3 27𝑥 2 𝑦 2 27 𝑥 3 6
= [ 2 + 2 − 2 ( 3 ) 𝑦 2 ] −2
27(6)2 𝑦 3 27(6)2 𝑦 2 27 63
=[ + − ( 3 ) 𝑦2]
2 2 2
27(−2)2 𝑦 3 27(−2)2 𝑦 2 27 (−2)3
-[ + − ( ) 𝑦2]
2 2 2 3
= [486𝑦 3
+ 486𝑦 2 − 972𝑦 − [54𝑦 3 + 54𝑦 2 + 36𝑦] 2]

= [486𝑦 3 − 486𝑦 2 ] − [54𝑦 3 + 90𝑦 2 ]


= 432𝑦 3 − 576𝑦 2

6 3𝑦 𝑦+2
Therefore, ∫−2 ∫0 ∫𝑥 (6𝑥𝑦𝑧 𝑑𝑧𝑑𝑦𝑑𝑥) is equal to 432𝑦 3 − 576𝑦 2 .

𝜋/2 cos 𝜃 𝑟 cos 𝜃


3. Evaluate ∫0 ∫0 ∫𝑟 2 𝑟 𝑑𝑧𝑑𝑟𝑑𝜃

Solution:

𝜋/2 cos 𝜃 𝑟 cos 𝜃 𝜋/2 cos 𝜃 𝑟 cos 𝜃


∫0 ∫0 ∫𝑟 2 𝑟 𝑑𝑧𝑑𝑟𝑑𝜃 = ∫0 ∫0 [∫𝑟 2 𝑟 𝑑𝑧] 𝑑𝑟𝑑𝜃
𝜋/2 cos 𝜃
= ∫0 ∫0 [𝑟𝑧] 𝑟 cos
𝑟2
𝜃
𝑑𝑟𝑑𝜃
𝜋/2 cos 𝜃
= ∫0 ∫0 [𝑟(𝑟 cos 𝜃)] − [𝑟(𝑟 2 )] 𝑑𝑟𝑑𝜃
𝜋/2 cos 𝜃
= ∫0 ∫0 (𝑟 2 cos 𝜃 − 𝑟 3 ) 𝑑𝑟𝑑𝜃
𝜋/2 𝑟 3 cos 𝜃 𝑟 4 cos 𝜃
= ∫0 [ −
] 0 𝑑𝜃
3 4
𝜋/2 (cos 𝜃)3 cos 𝜃 (cos 𝜃)4
= ∫0 [ 3
− 4
] 𝑑𝜃
4
𝜋/2 𝑐𝑜𝑠 𝜃 4
𝑐𝑜𝑠 𝜃
= ∫0 ( 3 − 4 )𝑑𝜃
𝜋/2 1
= ∫0 (12 𝑐𝑜𝑠 4 𝜃) 𝑑𝜃
𝜋/2 1 1+cos 2𝜃 2
= ∫0 [ ] 𝑑𝜃
12 2
1 𝜋/2
= ∫ (1 + 2 cos 2𝜃 + 𝑐𝑜𝑠 2 2𝜃) 𝑑𝜃
48 0

112
1 𝜋/2 1+cos 4𝜃
= 48 ∫0 [1 + 2 cos 2𝜃 + ( )] 𝑑𝜃
2
1 1 1
= 48 [𝜃 + sin 2𝜃 + 2 𝜃 + 8 sin 4𝜃] 𝜋/2
0
1 3 1
= 48 [2 𝜃 + sin 2𝜃 + 8 sin 4𝜃] 𝜋/2
0
1 3 𝜋 𝜋 1 𝜋
= 48 [2 (2 ) + sin 2 (2 ) + 8 sin 4 ( 2 )] − [0]
𝜋
= 64

𝜋/2 cos 𝜃 𝑟 cos 𝜃 𝜋


Therefore, ∫0 ∫0 ∫𝑟 2 𝑟 𝑑𝑧𝑑𝑟𝑑𝜃 is equal to 64.

Self-Help: You can also refer to the sources below to help you further
understand the lesson.

* Feliciano and Uy, F. (1983). Differential and Integral Calculus. Manila: Merriam and
Webster Bookstore, Inc.
* Love, C. and Rainville, E. (19811). Differential and Integral Calculus 6th Edition. The
Macmillan Company, New York.

Let’s Check
Activity 1. Evaluate the following multiple and apply some integration techniques (if
needed) that you’ve learned from the previous lessons.

2 2𝑦 1 1+𝑥 2
1. ∫1 ∫0 (𝑥 2 + 𝑦 2 ) 𝑑𝑥𝑑𝑦 2. ∫0 ∫1 (𝑦 + 2𝑥)−1/2 𝑑𝑦𝑑𝑥

2 4 𝜋/2 𝜋/2
3. ∫0 ∫2𝑥 𝑒 −𝑥−2𝑦 𝑑𝑦𝑑𝑥 4. ∫0 ∫𝑦 𝑦 sin 2𝑥 𝑑𝑥𝑑𝑦

1 1 1 𝑦 sin 𝑥
5. ∫0 ∫𝑥 𝑥(𝑥 2 + 𝑦 2 )2/3 𝑑𝑦𝑑𝑥 6. ∫0 ∫𝑦√ 𝑑𝑥𝑑𝑦
𝑥

Let’s Analyze

Activity 1. Evaluate the following triple integrals. Show your solutions neatly and
legibly.
2 𝑥 𝑥+𝑦 1 1 1
1. ∫0 ∫0 ∫0 𝑒 𝑥 (𝑦 + 2𝑧) 𝑑𝑧 𝑑𝑦 𝑑𝑥 2. ∫0 ∫0 ∫0 𝑎(𝑥 2 + 𝑦 2 + 𝑧 2 ) 𝑑𝑧 𝑑𝑦 𝑑𝑧

1 𝑧2 3 2 1 1
3. ∫0 ∫0 ∫0 𝑦 cos(𝑧 5 )𝑑𝑥 𝑑𝑦 𝑑𝑧 4. ∫0 ∫0 ∫𝑦 sinh(𝑧 2 ) 𝑑𝑧 𝑑𝑦 𝑑𝑥

113
2 4 2 3 2 √4−𝑥 2 1
5. ∫0 ∫0 ∫𝑧 𝑦𝑧𝑒 𝑥 𝑑𝑥 𝑑𝑦 𝑑𝑧 6. ∫−2 ∫−√4−𝑥2 ∫(𝑥 2 +𝑦 2)2 𝑥 2 𝑑𝑧 𝑑𝑦 𝑑𝑥

In a Nutshell

The importance of learning multiple integrals is very important in helping you solve
application-based problem involving areas and volumes of a region. In this portion of the
unit, you will be required to state your arguments or synthesis relevant to the topics
presented. I will supply the first two items and you will continue the rest.

1. The integration of double integrals applies the concepts of definite integral.


2. In performing integration of multiple integrals, you must integrate according to the
sequence of integration required in the problem.

Your Turn:

3. __________________________________________________________________________

4. __________________________________________________________________________

5. __________________________________________________________________________

Q&A List

Questions/Issues Answer
1.
2.
3.
4.
5.

Keywords Index

This section lists down the keywords that help students for recall. Keywords are like
Metalanguage but without the definitions and descriptions. This section includes
concepts (a word or phrase), ideas, theories, names of people, and other vital terms to
remember. Technically speaking, all those included in the Metalanguage should be part
of the Keywords. However, Keywords can also include other important concepts or ideas
not stipulated in the Metalanguage. This section also helps in the review.

Multiple Integrals Definite Integral


Double Integrals Inversion of Order
Triple Integrals

114
Big Picture in Focus: ULOb. demonstrate deep learning about
surfacing tracing more specifically integration of
different types of surfaces.

Metalanguage

In this section, the concept of definite integral in multiple integrals will be applied
and discussed to demonstrate ULOb. Please refer to these definitions in case you will
encounter difficulty in understanding surface integral.

1.Surface integral. surface integral is a generalization of multiple integrals to


integration over surfaces
2. Parameterization. Parameterization is the process of finding parametric
equations of a curve, a surface, or, more generally, a manifold or a
variety, defined by an implicit equation.
3.Cylindrical Surface. A cylindrical surface is a surface consisting of all the
points on all the lines which are parallel to a given line and which pass through a
fixed plane curve in a plane not parallel to the given line.
4. Plane surface. A plane surface is a flat surface which extends in all directions
5. Spherical surface. is a geometrical object in three-dimensional space that is
the surface of a ball.

Essential Knowledge

To perform the aforesaid big picture (unit learning outcomes) for the last two weeks
of this course, you need to review and apply the concepts and knowledge of definite
integral that were discussed from the previous weeks. Please note that you are not limited
to exclusively refer to these resources. Thus, you are expected to utilize other books,
research articles and other resources that are available in the university’s library e.g.
ebrary, search.proquest.com etc.

Surface Integrals

This section is talking about integrating functions over some surface, S, in three-
dimensional space. Let us start off with a sketch of the surface S since the notation can
get a little confusing once we get into it.

115
Figure 1. Parametric Surface

The region S will lie in some region D that lies in the xy-plane that shown in figure
1. We used a rectangle here, but it does not have to be of course. Also, note that we could
just as easily looked at a surface S that was in front of some region D in the yz-plane or
the xz-plane. Do not get so locked into the xy-plane that you cannot do problems that
have regions in the other two planes.

Now, how we evaluate the surface integral will depend upon how the surface is
given to us. There are essentially two separate methods that you can apply.

First Method

First, let’s look at the surface integral in which the surface S is given by z=g(x,y).
In this case, the surface integral is
∂g 2 ∂g 2
∬ 𝑓(𝑥, 𝑦, 𝑧)𝑑𝑆 = ∬ 𝑓(𝑥, 𝑦, 𝑔(𝑥, 𝑦))√( ) + ( ) + 1 𝑑𝐴
∂x ∂y
𝑆 𝐷

Note:
Now, we need to be careful here as both of these looks like standard double
integrals. The integral on the right is a standard double integral. The integral on the left
however is a surface integral. The way to tell them apart is by looking at the differentials.
The surface integral will have a dS while the standard double integral will have a dA.

To evaluate a surface integral, we will substitute the equation of the surface in


for z in the integrand and then add on the often messy square root. After that, the integral
is a standard double integral and by this point, we should be able to deal with that.

Second Method

The second method for evaluating a surface integral is for those surfaces that are
given by the parameterization.

⃗⃗
𝑟⃗ (𝑢, 𝑣) = 𝑥(𝑢, 𝑣)𝑖⃗ + 𝑦(𝑢, 𝑣)𝑗⃗ + 𝑧(𝑢, 𝑣)𝑘

116
In this case, the surface integral is,

∬ 𝑓(𝑥, 𝑦, 𝑧)𝑑𝑆 = ∬ 𝑓(𝑟⃗ (𝑢, 𝑣))||𝑟⃗⃗⃗⃗⃗


𝑢 𝑥 ⃗⃗⃗⃗
𝑟𝑣 || 𝑑𝐴
𝑆 𝐷

Where D is the range of the parameters that trace out the surface S.

Let’s notice that since we can parameterize a surface given by z=g(x,y) as,

⃗⃗
𝑟⃗ (𝑢, 𝑣) = 𝑥𝑖⃗ + 𝑦 𝑗⃗ + 𝑧𝑘
we can always use this form for these kinds of surfaces as well. It can be shown that,

∂g 2 ∂g 2
||𝑟⃗⃗⃗⃗⃗ 𝑟𝑣 || = √(∂x) + (∂y) + 1
𝑢 𝑥 ⃗⃗⃗⃗

for these kinds of surfaces. You might want to verify this for the practice of computing
these cross products.

Examples:

Plane Surface

1. Evaluate ∬ 6𝑥𝑦 𝑑𝑆 where S is the portion of the plane x + y + z


𝑆
= 1 that lies in front of the yz − plane

Solution:
Since we are looking for the portion of the plane that lies in front of the yz-plane
we are going to need to write the equation in the form x=g(y, z).
This is easy enough to do
x=1–y–z
Next, we need to determine just what D is. This is a sketch of the surface S.

Here is a sketch of the region D.


117
Notice that the axes are labeled differently than we are used to seeing in the
sketch of D. This was to keep the sketch consistent with the sketch of the surface. We
arrived at the equation of the hypotenuse by setting x equal to zero in the equation of
the plane and solving for z. Here are the ranges for y and z.

0<y< 1 0<z< 1−𝑦

Now, because the surface is not in the form z=g(x,y) we can’t use the formula
above. However, as noted above we can modify this formula to get one that will work
for us.
∂g 2 ∂g 2
∬ 𝑓(𝑥, 𝑦, 𝑧)𝑑𝑆 = ∬ 𝑓(𝑔(𝑦, 𝑧), 𝑦, 𝑧)√1 + ( ) + ( ) 𝑑𝐴
∂y ∂z
𝑆 𝐷

The changes made to the formula should be the somewhat obvious changes.
So, let’s do the integral.

∬ 6𝑥𝑦 𝑑𝑆 = ∬ 6(1 − 𝑦 − 𝑧)𝑦√(1) + (−1)2 + (−1)2 𝑑𝐴


𝑆 𝐷

Notice that we plugged in the equation of the plane for the x in the integrand. At
this point, we’ve got a fairly simple double integral to do. Here is that work.
∬ 6𝑥𝑦 𝑑𝑆 = ∬ 6(1 − 𝑦 − 𝑧) 𝑦 √3 𝑑𝐴
𝑆 𝐷
1 1−𝑦
= ∫0 ∫0 6(𝑦 − 𝑦 2 𝑦𝑧)√3 𝑑𝑧𝑑𝑦
1 1−𝑦
= 6√3 ∫0 ∫0 (𝑦 − 𝑦 2 − 𝑦𝑧) 𝑑𝑧𝑑𝑦
1 1
= 6√3 ∫0 (𝑦𝑧 − 𝑦 2 𝑧 − 2 𝑦𝑧 2 ) |1−𝑦
0
𝑑𝑦
1 1
= 6√3 ∫0 [𝑦(1 − 𝑦) − 𝑦 2 (1 − 𝑦) − 2 𝑦(1 − 𝑦)2 ] − 0 𝑑𝑦
1 1 1
= 6√3 ∫0 (2 𝑦 − 𝑦 2 + 2 𝑦 3 ) 𝑑𝑦
1 1 1
=6√3 (4 𝑦 2 − 3 𝑦 3 + 8 𝑦 4 )|10
1 1 1
= 6√3[4 (1)2 − 3 (1)3 + 8 (1)4 ] − (0)
6−8+3 √3
=6√3 ( ) =
24 4
118
Spherical Surface
2. Evaluate ∬ 𝑧 𝑑𝑆 where S is the upper half of a sphere of radius 2.
𝑆
Solution:
We need to know the parameterization of a sphere. Here is the parameterization
for this sphere.

⃗⃗
𝑟⃗ ( 𝜃 , 𝜑) = 2𝑠𝑖𝑛 𝜑𝑐𝑜𝑠 𝜃 𝑖⃗ + 2𝑠𝑖𝑛 𝜑𝑠𝑖𝑛𝜃 𝑗⃗ + 2𝑐𝑜𝑠𝜑𝑘

Since we are working on the upper half of the sphere here are the limits on the
parameters.
𝜋
0 < 𝜃 < 2𝜋 0<𝜑< 2

Next, we need to determine ⃗⃗⃗⃗


𝑟𝜃 𝑥 ⃗⃗⃗⃗⃗
𝑟𝜑 . Here are the two individual vectors.

𝜃 ( 𝜃 , 𝜑) = −2𝑠𝑖𝑛 𝜑𝑠𝑖𝑛 𝜃 𝑖⃗ + 2𝑠𝑖𝑛 𝜑𝑐𝑜𝑠𝜃 𝑗⃗


𝑟⃗⃗⃗⃗
⃗⃗
𝑟𝜑 ( 𝜃 , 𝜑) = 2𝑐𝑜𝑠 𝜑𝑐𝑜𝑠 𝜃 𝑖⃗ + 2𝑐𝑜𝑠 𝜑𝑠𝑖𝑛𝜃 𝑗⃗ − 2𝑠𝑖𝑛𝜑𝑘
⃗⃗⃗⃗⃗

Now let’s take cross product.

𝑖⃗ 𝑗⃗ ⃗⃗
𝑘
𝑟𝜃 𝑥 ⃗⃗⃗⃗⃗
⃗⃗⃗⃗ 𝑟𝜑 = [−2𝑠𝑖𝑛 𝜑𝑠𝑖𝑛 𝜃 2𝑠𝑖𝑛 𝜑𝑐𝑜𝑠𝜃 0 ]
2𝑐𝑜𝑠 𝜑𝑐𝑜𝑠 𝜃 2𝑐𝑜𝑠 𝜑𝑠𝑖𝑛𝜃 −2𝑠𝑖𝑛𝜑

= −4𝑠𝑖𝑛2 𝜑 𝑐𝑜𝑠 𝜃 ⃗⃗ − 4𝑠𝑖𝑛2 𝜑 𝑠𝑖𝑛 𝜃 𝑗⃗ −


𝑖⃗ − 0𝑗⃗ − 4𝑠𝑖𝑛𝜑𝑐𝑜𝑠𝜑 𝑠𝑖𝑛2 𝜃𝑘
4𝑠𝑖𝑛𝜑𝑐𝑜𝑠𝜑𝑐𝑜𝑠 2 𝜃 ⃗⃗
𝑘
= −4𝑠𝑖𝑛2 𝜑 𝑐𝑜𝑠 𝜃 ⃗⃗
𝑖⃗ − 4𝑠𝑖𝑛2 𝜑 𝑠𝑖𝑛 𝜃 𝑗⃗ − 4𝑠𝑖𝑛𝜑𝑐𝑜𝑠𝜑(𝑠𝑖𝑛2 𝜃 + 𝑐𝑜𝑠 2 𝜃 ) 𝑘
= −4𝑠𝑖𝑛2 𝜑 𝑐𝑜𝑠 𝜃 2
𝑖⃗ − 4𝑠𝑖𝑛 𝜑 𝑠𝑖𝑛 𝜃 𝑗⃗ − 4𝑠𝑖𝑛𝜑𝑐𝑜𝑠𝜑𝑘⃗⃗

Finally, we need the magnitude of this,

= √(−4𝑠𝑖𝑛2 𝜑 𝑐𝑜𝑠 𝜃 )2 + (−4𝑠𝑖𝑛2 𝜑 𝑠𝑖𝑛 𝜃 )2 + (−4𝑠𝑖𝑛𝜑𝑐𝑜𝑠𝜑)2


= √16𝑠𝑖𝑛4 𝜑 𝑐𝑜𝑠 2 𝜃 + 16𝑠𝑖𝑛4 𝜑 𝑠𝑖𝑛2 𝜃 + 16𝑠𝑖𝑛2 𝜑𝑐𝑜𝑠 2 𝜑
= √16𝑠𝑖𝑛4 𝜑( 𝑐𝑜𝑠 2 𝜃 + 𝑠𝑖𝑛2 𝜃) + 16𝑠𝑖𝑛2 𝜑𝑐𝑜𝑠 2 𝜑
= √16𝑠𝑖𝑛4 𝜑 + 16𝑠𝑖𝑛2 𝜑𝑐𝑜𝑠 2 𝜑
= √16𝑠𝑖𝑛2 𝜑( 𝑠𝑖𝑛2 𝜑 + 𝑐𝑜𝑠 2 𝜑)
= √16𝑠𝑖𝑛2 𝜑
= 4𝑠𝑖𝑛 𝜑

119
We can drop the absolute value bars in the sine because sine is positive in the
range of 𝜑 that we are working with. The surface integral is then

∬ 𝑧 𝑑𝑆 = ∬ 2𝑐𝑜𝑠 𝜑(4𝑠𝑖𝑛𝜑)𝑑𝐴
𝑆 𝐷

Don’t forget that we need to plug in for x, y and/or z in these as well, although in
this case we just needed to plug in z. Here is the evaluation for the double integral.
𝜋
2𝜋
2
∬ 𝑧 𝑑𝑆 = ∫ ∫ 2𝑐𝑜𝑠𝜑 (4𝑠𝑖𝑛𝜑) 𝑑𝜑𝑑𝜃
0 0
𝑆
𝜋
2𝜋
2
=∫ ∫ (4)2𝑐𝑜𝑠𝜑 𝑠𝑖𝑛𝜑 𝑑𝜑𝑑𝜃
0 0
𝜋
2𝜋
2
=∫ ∫ 4𝑠𝑖𝑛2𝜑 𝑑𝜑𝑑𝜃
0 0
𝜋
2𝜋
= ∫0 4(−2 cos(2𝜑)) |02 d𝜃
𝜋
2𝜋 𝜋
= ∫0 4 [−2 cos (2 ( 2 ))] − 4[−2 cos(2(0))] |02 d𝜃
2𝜋
= ∫0 4 d𝜃
2𝜋
= 4𝜃|
0
= 4 (2𝜋) − 4(0)
= 8𝜋

Cylindrical Surface

Example:

Evaluate ∬𝑆 𝑦 + 𝑧 𝑑𝑆 where S is the surface whose side is the cylinder x 2 + y 2 = 3,


whose bottom is the disk x 2 + y 2 < 3 in the xy-plane and whose top is the plane z=4-y.

Solution:

We are putting a top and bottom on the surface this time. Let’s first start with a sketch
of the surface.

120
There is more to this sketch than the actual surface itself. We’re going to let 𝑆1 be
the portion of the cylinder that goes from the xy-plane to the plane. In other words, the
top of the cylinder will be at an angle. We’ll call the portion of the plane that lies inside
(i.e. the cap on the cylinder) 𝑆2 . Finally, the bottom of the cylinder is the disk of radius
√3 in the xy-plane and is denoted by 𝑆3 .

In order to do this integral, we’ll need to note that just like the standard double
integral, if the surface is split up into pieces we can also split up the surface integral.
So, for our example, we will have

∬ 𝑦 + 𝑧 𝑑𝑆 = ∬ 𝑦 + 𝑧 𝑑𝑆 + ∬ 𝑦 + 𝑧 𝑑𝑆 + ∬ 𝑦 + 𝑧 𝑑𝑆
𝑆 𝑆1 𝑆2 𝑆3
We’re going to need to do three integrals here. However, we’ve done most of the
work for the first one in the previous example so let’s start with that.

𝑺𝟏 : The Cylinder
The parameterization of the cylinder and |𝑟⃗⃗⃗⃗
𝜃 𝑥 ⃗⃗⃗⃗⃗|
𝑟𝜑 is,

⃗⃗
𝑟𝜃 𝜃) = √3𝑐𝑜𝑠𝜃𝑖⃗ + √3 sinθ𝑗⃗ + 𝑧𝑘
⃗⃗⃗⃗(𝑧, |𝑟⃗⃗⃗⃗
𝜃 𝑥 ⃗⃗⃗⃗⃗|
𝑟𝜑 = √3

This will be the limit on the parameters.

0 < 𝜃 < 2𝜋
0 < z < 4 − 𝑦 = 4 − √3 sin 𝜃

121
The upper limit for the z’s is the plane so we can just plug that in. However, since
we are on the cylinder, we know what y is from the parameterization so we will also
need to plug that in.

Here is the integral for the cylinder.

∬ 𝑦 + 𝑧 𝑑𝑆 = ∬(√3 sin 𝜃 + 𝑧) √3𝑑𝐴


𝑆1 𝐷
2𝜋 4−√3 𝑠𝑖𝑛𝜃
= √3 ∫0 ∫0 √3 sin𝜃 + 𝑧 𝑑𝑧𝑑𝜃
2𝜋 1
= √3 ∫0 (√3 𝑠𝑖𝑛 𝜃(𝑧) + 2 (𝑧 )3 |(4−√30𝑠𝑖𝑛𝜃) 𝑑𝜃
2𝜋 1 2 1
=√3 ∫0 [√3 𝑠𝑖𝑛 𝜃(4 − √3 𝑠𝑖𝑛𝜃) + 2 (4 − √3 𝑠𝑖𝑛𝜃 ) ] – [√3𝑠𝑖𝑛𝜃(0) + 2 (0 )]
2𝜋 1
= √3 ∫0 [4√3 𝑠𝑖𝑛 𝜃 − 3𝑠𝑖𝑛𝜃 + 2 (16 − 8√3𝑠𝑖𝑛 𝜃 + 3 sin2 𝜃)]𝑑𝜃
2𝜋 3
= √3 ∫0 (4√3 𝑠𝑖𝑛 𝜃 − 3𝑠𝑖𝑛𝜃 + 4 − 4√3𝑠𝑖𝑛 𝜃 + 2 sin2 𝜃)𝑑𝜃
2𝜋 3
= √3 ∫0 (8 + 2 sin2 𝜃)𝑑𝜃
2𝜋 3
= √3 ∫0 (8 + 4 (1 − cos(2𝜃))𝑑𝜃
29 3
= √3( 4 𝜃 + 8 sin(2𝜃)) |2𝜋
0
29 3 29 3
= √3[ 4 (2𝜋) + 8 sin(4𝜋)]-√3[ 4 (0) + 8 sin(0)]
29√3𝜋
= 2

𝑺𝟐 : Plane on Top of the Cylinder

In this case, we don’t need to do any parameterization since it is set up to use the
formula that we gave at the start of this discussion. Remember that the plane is given by
z=4-y. Also that, for this surface. D is the disk of radius √3 centered at the origin.

Here is the integral for the plane.


∬ 𝑦 + 𝑧 𝑑𝑆 = ∬(𝑦 + 4 − 𝑦) √(0)2 + (−1)2 + 1𝑑𝐴
𝑆2 𝐷
= ∬𝐷 4 √2𝑑𝐴

Don’t forget that we need to plug in for z. Now at this point, we can proceed in one
of two ways. Either we can proceed with the integral or we can recall that ∬𝐷 𝑑𝐴 is nothing
more than the area of D and we know that D is the disk of radius √3 and so there is no
reason to do the integral.

122
∬ 𝑦 + 𝑧 𝑑𝑆 = 4√2 ∬ 𝑑𝐴
𝑆2 𝑆2
2
= 4√2 (𝜋(√3) )
= 12√2 𝜋

𝑺𝟑 : Bottom of the Cylinder

Again, this is set up to use the initial formula we gave in this section once we realize
that the equation for the bottom is given by g(x,y)=0 and D is the disk of radius √3
centered at the origin. Also, don’t forget to plug in for z.

∬ 𝑦 + 𝑧 𝑑𝑆 = ∬(𝑦 + 0) √(0)2 + (0)2 + 1𝑑𝐴


𝑆3 𝐷

∬ 𝑦 + 𝑧 𝑑𝑆 = ∬ 𝑦 𝑑𝐴
𝑆3 𝐷
2𝜋 √3
= ∫0 ∫0 𝑟 2 sin𝜃𝑑𝑟𝑑𝜃
2𝜋 1 3
= ∫0 𝑟 sin𝜃 |√3
0
𝑑𝜃
3
2𝜋 1 1
= ∫0 [3 (√3)3 sin𝜃 − 3 (0)3 sin𝜃 ]𝑑𝜃
2𝜋 1 3
= ∫0 3 [(√3) sin𝜃]𝑑𝜃
2𝜋
= ∫0 √3 sin𝜃𝑑𝜃
= −√3cos𝜃 |2𝜋 0
=0

We can now get the value of the integral that we are after.

∬ 𝑦 + 𝑧 𝑑𝑆 = ∬ 𝑦 + 𝑧 𝑑𝑆 + ∬ 𝑦 + 𝑧 𝑑𝑆 + ∬ 𝑦 + 𝑧 𝑑𝑆
𝑆 𝑆1 𝑆2 𝑆3

29√3𝜋
= +12√2 𝜋+0
2

𝜋
= (29√3+24√2 )
2

123
Self-Help: You can also refer to the sources below to help you further
understand the lesson.

* Love, C. and Rainville, E. (19811). Differential and Integral Calculus 6th Edition. The
Macmillan Company, New York.
*Dawkins, P(2007). Calculus III: Surface Integral. Retrieved from
https://tutorial.math.lamar.edu/terms.aspx

Let’s Check!

Activity 1. Answer the following problems.

1. 𝐸𝑣𝑎𝑙𝑢𝑎𝑡𝑒 ∬ 40 𝑦𝑑𝑆 𝑤ℎ𝑒𝑟𝑒 𝑆 𝑖𝑠 𝑡ℎ𝑒 𝑝𝑜𝑟𝑡𝑖𝑜𝑛 𝑜𝑓 𝑦 = 3𝑥 2 + 2𝑧 2 𝑡ℎ𝑎𝑡 𝑙𝑖𝑒𝑠 𝑏𝑒ℎ𝑖𝑛𝑑 𝑦 = 6.


𝑆

2. 𝐸𝑣𝑎𝑙𝑢𝑎𝑡𝑒 ∬ 𝑧 𝑑𝑆 𝑤ℎ𝑒𝑟𝑒 𝑆 𝑖𝑠 𝑡ℎ𝑒 𝑢𝑝𝑝𝑒𝑟 ℎ𝑎𝑙𝑓 𝑜𝑓 𝑎 𝑠𝑝ℎ𝑒𝑟𝑒 𝑜𝑓 𝑟𝑎𝑑𝑖𝑢𝑠 5.


𝑆

3. 𝐸𝑣𝑎𝑙𝑢𝑎𝑡𝑒 ∬ 3𝑦 𝑑𝑆 𝑤ℎ𝑒𝑟𝑒 𝑆 𝑖𝑠 𝑡ℎ𝑒 𝑝𝑜𝑟𝑡𝑖𝑜𝑛 𝑜𝑓 𝑦 2 + 𝑥 2 = 4 𝑏𝑒𝑡𝑤𝑒𝑒𝑛


𝑆
𝑥 = 3 𝑎𝑛𝑑 𝑥 = 3 − 𝑧.

4. 𝐹𝑖𝑛𝑑 ∬𝑆(𝑥 + 𝑦𝑧)𝑑𝑆 𝑤ℎ𝑒𝑟𝑒 𝑆 𝑖𝑠 𝑡ℎ𝑒 𝑠𝑢𝑟𝑓𝑎𝑐𝑒 𝑜𝑓 𝑧 = 12 − 4𝑥 − 3𝑦 𝑐𝑜𝑛𝑡𝑎𝑖𝑛𝑒𝑑 𝑖𝑛


𝑖𝑛 𝑡ℎ𝑒 𝑓𝑖𝑟𝑠𝑡 𝑞𝑢𝑎𝑑𝑟𝑎𝑛𝑡.

Let’s Analyze!

Activity 1. Answer the following problems.


1. 𝐸𝑣𝑎𝑙𝑢𝑎𝑡𝑒 ∬ 𝑦 𝑑𝑆 𝑤ℎ𝑒𝑟𝑒 𝑆 𝑖𝑠 𝑡ℎ𝑒 𝑝𝑜𝑟𝑡𝑖𝑜𝑛 𝑜𝑓 𝑡ℎ𝑒 𝑐𝑦𝑙𝑖𝑛𝑑𝑒𝑟 4𝑥 2 + 2𝑦 2 = 16
𝑆
𝑡ℎ𝑎𝑡 𝑙𝑖𝑒𝑠 𝑏𝑒𝑡𝑤𝑒𝑒𝑛 𝑧 = 0 𝑎𝑛𝑑 𝑧 = 7.

2. 𝐸𝑣𝑎𝑙𝑢𝑎𝑡𝑒 ∬ 𝑥 2 𝑑𝑆 𝑤ℎ𝑒𝑟𝑒 𝑆 𝑖𝑠 𝑡ℎ𝑒 𝑝𝑜𝑟𝑡𝑖𝑜𝑛 𝑜𝑓 𝑡ℎ𝑒 𝑠𝑝ℎ𝑒𝑟𝑒 𝑜𝑓 𝑟𝑎𝑑𝑖𝑢𝑠 4, 𝑐𝑒𝑛𝑡𝑒𝑟𝑒𝑑 𝑎𝑡 𝑡ℎ𝑒 𝑜𝑟𝑖𝑔𝑖𝑛,
𝑆
𝑠𝑢𝑐ℎ 𝑡ℎ𝑎𝑡 𝑥 > 0 𝑎𝑛𝑑 𝑧 > 0.

3. 𝐸𝑣𝑎𝑙𝑢𝑎𝑡𝑒 ∬ 𝑧 + 3 𝑑𝑆 𝑤ℎ𝑒𝑟𝑒 𝑆 𝑖𝑠 𝑡ℎ𝑒 𝑠𝑢𝑟𝑓𝑎𝑐𝑒 𝑜𝑓 𝑡ℎ𝑒 𝑠𝑜𝑙𝑖𝑑 𝑏𝑜𝑢𝑛𝑑𝑒𝑑 𝑏𝑦 𝑧 = 2𝑥 2 + 2𝑦 2


𝑆
−3 𝑎𝑛𝑑 𝑧 = 1.

124
4. 𝐸𝑣𝑎𝑙𝑢𝑎𝑡𝑒 ∬𝑆 𝑥 + 𝑦 2 + 𝑧 2 𝑑𝑆 𝑤ℎ𝑒𝑟𝑒 𝑆 𝑖𝑠 𝑡ℎ𝑒 𝑝𝑜𝑟𝑡𝑖𝑜𝑛 𝑜𝑓 𝑥 = 4 − 𝑦 2 − 𝑧 2 𝑡ℎ𝑎𝑡 𝑙𝑖𝑒𝑠 𝑖𝑛
𝑓𝑟𝑜𝑛𝑡 𝑜𝑓 𝑥 = −2.

5. 𝐸𝑣𝑎𝑙𝑢𝑎𝑡𝑒 ∬ 2𝑥 − 3𝑦 + 𝑧𝑑𝑆 𝑤ℎ𝑒𝑟𝑒 𝑆 𝑖𝑠 𝑡ℎ𝑒 𝑝𝑜𝑟𝑡𝑖𝑜𝑛 𝑜𝑓 𝑥 + 𝑦 + 𝑧 = 2 𝑡ℎ𝑎𝑡 𝑙𝑖𝑒𝑠 𝑖𝑛


𝑆
𝑓𝑖𝑟𝑠𝑡 𝑜𝑐𝑡𝑎𝑛𝑡.

In a Nutshell

This section shows how Double Integrals help us solve for the surfaces of the
plane, spherical, and cylindrical figures at the given limits. In this portion of the unit, you
will be required to state your arguments or synthesis relevant to the topics presented. I
will supply the first two items and you will continue the rest.

1. In dealing with surface tracing, you need to be familiar with parameterization


2. Surface integrals are also used to find the flow of material through a surface.

Your Turn:

3. __________________________________________________________________________

4. __________________________________________________________________________

5. __________________________________________________________________________

Q&A List

Questions/Issues Answer
1.
2.
3.
4.
5.

Keywords Index

This section lists down the keywords that help students for recall. Keywords are like
Metalanguage but without the definitions and descriptions. This section includes
concepts (a word or phrase), ideas, theories, names of people, and other vital terms to
remember. Technically speaking, all those included in the Metalanguage should be part
of the Keywords. However, Keywords can also include other important concepts or ideas
not stipulated in the Metalanguage. This section also helps in the review.

125
Surface Integral Cylindrical Surface
Parameterization Plane Surface
Double Integral Spherical Surface

126

You might also like